v1) i - blue valley schools chemistry/ap... · 3djh ri 3djh ri 2001 ap® chemistry...

182
DO NOT D1TACH FRONI BOOK. I.>’ t)O8 4 Li I Be 6.94 Na ,2299 PERIODIC TABLE OF THE ELEMENTS 10 20 K (‘a 391)) ‘UJi>> 21 Sc 44 ‘0 I I i’i 479(1 5 7 8 9 B C N 0 F V 50i)4 37 Rb 85.47 49 38 Sr $7.62 1 He 4,9)) 1) Nc 20.18 Is Ar 39 95 I 3 Al 26.98 24 25 26 27 28 Cr Mn Fe Co Ni 52,00 54.94 55.85 58.93 t) Y 88.91 14 Si 28.09 .10 Zr 91 22 P 30.97 41 Nb 92.91 I (>00 16 S 42.06 9 (8) 17 CI 45,45 42 43 Mo Tc 95,94 >8) 29 30 Cu Zn o3,55 o5,39 .44 Ru I 0 31 32 Ga Ge )‘) 72 72 5’) 45 46 Rh Pd 102.01 106.42 As 74.92 47 Ag 107.87 87 Fr (2231 48 Cd 12.41 4)) In I 14.82 $9 tAc 21.7.03 .L’4+U •226.02 50 Sn 118.71 103 Rf 1.4 ,>5 Sc Br 7599 7), 99 5:. 5•3 Te I 127.60 Ilo.9I 55 40 57 72.73 74 75 76 77 78 79 80 XI 82 53 54 85 86 (,s Bi ‘I a hf I a V Re Os Jr Pt Au Hg II Pb Bi Po t Rn I 0 I I >59) “54) 1s09 l8>S’ 1SOJ lO2 9” 2 19205 969” “0059 “04 ‘> 1 050’ { (>9> 9> 51 Sb 121775 105 I) b (262 36 Kr 8379) 54 Xe 131.1’) 106 107 Sg Bh (266) (264) r” Co >7’> Zri 0 2 I’D > 2 2 C) 0 0 z -4 0 -4 m z rn x -4 -ci C, m 10$ us (277> 109 110 Mt I)s (2683 (%71 III Rg (27%) Lafltianide Seres tAclinide Seriec 58 5’) Ce Pr 140.12 140.91 6)) 61 62 Nd Pm Sm 144.24 (145) 150.4 90 Th (>1 Eu 151.97 91 Pa 64 (>5 Gd Tb 457:25 •158.9:3 I >2.94 2>1.04 66 i)y 1 ((2,50 93 04 95 96 U Np Pu Am Cm 2>5.0> (237) 244 2.13 (247) 67 (‘5 (0) Ho Er Tm 164.93 167.26 164.93 V 1) 173.04 97 Bk I ti 74,97 98 Cf (251) 99 100 101 102 Es Fm Md No 252) 2’ 403 I r Handout Page 1 of 182 Handout Page 1 of 182 Handout Page 1 of 182 Updated 4/9/2014 Handout Page 1 of 182

Upload: vuongmien

Post on 12-Mar-2018

255 views

Category:

Documents


7 download

TRANSCRIPT

Page 1: V1) I - Blue Valley Schools Chemistry/AP... · 3DJH RI 3DJH RI 2001 AP® CHEMISTRY FREE-RESPONSEQUESTIONS Answer EITHER Question 2 below OR Question 3 printed on page 8

DO

NO

TD

1TA

CH

FR

ON

IB

OO

K.

I.>’

t)O8

4

Li

IB

e

6.94 Na

,2299

PE

RIO

DIC

TA

BL

EO

FT

HE

EL

EM

EN

TS

102

0

K(‘

a39

1))

‘UJi

>>

21 Sc 44‘0

II

i’i

479(

1

57

89

BC

N0

F

V50

i)4

37 Rb

85.4

7

49

38 Sr

$7.6

2

1 He

4,9)

)

1)

Nc

20.1

8Is Ar

3995

I3 Al

26.9

824

2526

2728

Cr

Mn

Fe

Co

Ni

52,0

054

.94

55.8

558

.93

t) Y88

.91

14 Si2

8.0

9

.10

Zr

9122

P30

.97

41 Nb

92.9

1

I (>0

016 S

42.0

6

9(8

)

17 CI

45,4

5

4243

Mo

Tc

95,9

4>

8)

2930

Cu

Zn

o3,5

5o5

,39

.44

Ru

I 0

3132

Ga

Ge

)‘)

7272

5’)

45‘

46

Rh

Pd

102.

0110

6.42

As

74.9

247 Ag

107.

87

87 Fr

(223

1

48 Cd

12.4

1

4))

InI

14.8

2

$9 tAc

21.7

.03

.L’4+

U

•226

.02

50 Sn 118.

71

103

Rf

1.4,>

5

ScB

r7599

7),99

5:.5•

3

Te

I12

7.60

Ilo

.9I

5540

5772.7

374

7576

7778

7980

XI

8253

5485

86

(,s

Bi

‘Ia

hf

I aV

Re

Os

JrP

tA

uH

gII

Pb

Bi

Pot

Rn

I0

II

>59)

“5

4)

1s0

9l8

>S

’1S

OJ

lO2

9”

219205

969”

“0059

“04

‘>

10

50

’{

(>9>

9>

51 Sb

1217

75

105

I)b

(262

36 Kr

8379

)

54 Xe

131.

1’)

106

107

SgB

h(2

66)

(264)

r”—

Co

>7

’>

Zri 0 2 I’D > 2 2

C) 0 0 z -4 0 -4 m z rn x -4 -ci

C, m

10$

us

(277

>

109

110

Mt

I)s

(268

3(%

71

III

Rg

(27%

)

Lafl

tianid

eS

ere

s

tAcli

nid

eS

erie

c

585’

)

Ce

Pr

140.

1214

0.91

6))

6162

Nd

Pm

Sm

144.

24(1

45)

150.

490

Th

(>1

Eu

151.

9791 Pa

64(>

5

Gd

Tb

457:

25•1

58.9

:3

I>2

.94

2>1.

04

66 i)y

1((2

,50

9304

9596

UN

pP

uA

mC

m2>

5.0>

(237)

244

2.13

(247

)

67

(‘5

(0)

Ho

Er

Tm

164.

9316

7.26

164.

93

V1)

173.

0497 Bk

Iti

74,9

7

98 Cf

(251

)

9910

010

110

2

Es

Fm

Md

No

252)

2’,

403

I r

Handout Page 1 of 182 Handout Page 1 of 182

Handout Page 1 of 182 Updated 4/9/2014 Handout Page 1 of 182

Page 2: V1) I - Blue Valley Schools Chemistry/AP... · 3DJH RI 3DJH RI 2001 AP® CHEMISTRY FREE-RESPONSEQUESTIONS Answer EITHER Question 2 below OR Question 3 printed on page 8

Fg)t2e

3L

t-’:’ 2t

Br2 lj 2 -

211324 + 2

2e

hg:

Fe

I,)) 2c

Cu + e

2c

Cu’ --e

Sn 2e

Sc 211

2FC -- 2e

Ph 2e

Sn2-’- 2e

Ni2+ 2e

Co2— 2eCd2—2e

Cr3+e

Fe + 2c

Cr + 3e

Zn2+2

2H20(1)+ 2c

Mn” +2e

3c

Bc” --2s

— 2

Na

• 2c

Sr22

+ 2e

K -c

Cs” -e

L_ Li

GO ON TO THE NEXT PAGE.

STANDARD REDUCTION POTENTiALS iN AQUEOUS SOLUTION AT 25 C

Half-reaction L (V)

I 2F

- Co

2C1

— 2H2O

—i 2Br

-4 Hg2

—4 Hg(1_r

—* 21-Igh

Fe”

— Cuc

—I Cu(sJ

—I Cu

Sn2”

—‘ Phc—f Snisi

4 Ni(

Co(s

-4 Cds

— Cr”

— Fc

—5’ Cr(s

I Znv

—I H,(g+20H

—4

— Al

— Be(.s

— Na

—I Ca(s

—5’

Ks

Co

Li(s

2.$

1.52

1.50

l.3b

123

LU”

0.92

0.85

0.80

0. “9

0.’7

0.53

0.52

0.34

0.15

0IS

0,14

0.00

—013

—0.14

—0.25

—0.28

—0.40

—0.41

-0.44

—0.74

—0.76

—0.83

—1.18

— 1.66

— 1.70

— - -..

—2.71

— Q

--2.90

—2.92

2.92

2.92

—3.05

-3-

Handout Page 2 of 182 Handout Page 2 of 182

Handout Page 2 of 182 Updated 4/9/2014 Handout Page 2 of 182

Page 3: V1) I - Blue Valley Schools Chemistry/AP... · 3DJH RI 3DJH RI 2001 AP® CHEMISTRY FREE-RESPONSEQUESTIONS Answer EITHER Question 2 below OR Question 3 printed on page 8

EQUIliBRIUM

K- H[A j- [HAl

K [0H][HWI[Bj

K = IOH ][HH — I,)>> @ 25 C—KK

pH — —log[H 1. p()H = —Iog[OFll3=pl1 pOH

[AjpH pL l0[KAj

[HW]pOH pk.. — Io,[B]

pK — —lugKa. pK;, = —logA,

K1 = K (RT).

where An — moles product gas — moles reactant gas

THERMOCHEMISTRYIKINETICS

AS - 3 products —S reactants

All — AH1 products —AH1 reactants

AG AG1 products —AG1 reactants

- \H —7_AS

—RTInK = —1303 RTIogK

-nP E

AG AG 4-RTlnQ= \G --l303RTlogQ

q — meAT

C-

yjj

CONSTANTS

U \eloct

a principal quantum numberma — mass

Eauilihrium Constants

K (Weak acid)

weak base

K atcr

K. gas pressure)

K molar concentrations)

— standard entropy

II — standard enthalpv

G — standard free energ\

E standard reduction potentialT = temperature

a moles

m — mass

q = heat

= specific heat capacity— molar heat capacity at constant pressure

activation energyk — rate constant

A frequency factor

96,500 coulomhs per moleof electrons

8.3! J mal K

0.021 [,atrn an 1 ‘K

— 62.3 L torr nal K

X.3 I s oh cauhrnh nial K

GO ON TO THE NEXT PAGE.

ADV \NCED PLACEMENT CHEMIS IRY EQUA flONS AND

ATOMIC STRUCTURE K — energy

£ — h frequency

1? 3 — waselengthp—ran

mU p momentum—11Th 10

— ))Ulea Speed or light, 3)) l0 m

Plancks constant. 6 6.63 H) J

Boltzmanns constant. = .3X lu-•’

Aogadro’s number 6.022 v 1U- mol

Electron charge. c = —1,602 10 a coulomb

I electron o1t per atom 96.5 kJ mol

I I[5] [A]

-E IInk

= R 7_I ‘‘

Farada’s constant, ±

Gas constant. R

-4-

Handout Page 3 of 182 Handout Page 3 of 182

Handout Page 3 of 182 Updated 4/9/2014 Handout Page 3 of 182

Page 4: V1) I - Blue Valley Schools Chemistry/AP... · 3DJH RI 3DJH RI 2001 AP® CHEMISTRY FREE-RESPONSEQUESTIONS Answer EITHER Question 2 below OR Question 3 printed on page 8

GASES, LIQUIDS, AND SOLUTIONS

12KE per molecule = 2mti

KE per mole RT

1

molarity. M = moles solute per liter solution

moialitv moles solute per kdocram sol ent

sT. iK, x molalit\

= iK,, Y molalit

it = iMRT

.1 = abc

P — pressure

V — volume

T temperature

a number ol rnole

D — denit

in — mass

Ii vclocit

a,,, root —niean —square \pced

KE — kinetic enera’.r — rate of etfusion

— molar rilass

it — osmotic pressure= vant Hoff factor

K, — molal freezing-point depression constant

— molal boiling-point elevation constant

A — absorbance

a = moIarahsorptivit

b - pathlength

c — concentration

Q = reaction quotient

I = current (ampercs

q = charge (coulomhs)

time (seconds)

E = standard reduction potential

K = equilibrium constant

Gas constant, R 8.31 J mol K

= 0.0821 L arm mol ‘K

= 62.4 L torr niol K

= 8,31 volt coulomb mol K

1.38 10 2, J K

=l.X6Kkgmol

K, for H20 — 0.512 Kkgmol

I atm = 60 mm H= _‘sot.’tr

GO ON TO THE NEXT PAGE.

P1’ — nRT

1’ — n/a aRT

nmle% A— P -- / X ‘.shere .V

— total niolcP. -P-P.--;’

.11

C —2 ‘

D

3kT — 3RT°

\rn M

(JXE[)ATION-REDUCTION; ELF.CTROCHEMISTRY

Q[Ci[Dlh\hBCdD[\]“[Bj

I(

R7 0059”E.. = E — lnQ K, — ‘ ‘ logQ @25 C

nP a

‘iTha k

— 0,0502

Boltzmann’s constant, k

K1 forH,O

STP — 000 C and I 0

1’aIdL’ s cun.tant. . 00.501) c- Ui’ n1b— i’ii’

at electron..

-5-

Handout Page 4 of 182 Handout Page 4 of 182

Handout Page 4 of 182 Updated 4/9/2014 Handout Page 4 of 182

Page 5: V1) I - Blue Valley Schools Chemistry/AP... · 3DJH RI 3DJH RI 2001 AP® CHEMISTRY FREE-RESPONSEQUESTIONS Answer EITHER Question 2 below OR Question 3 printed on page 8

2001 AP® CHEMISTRY FREE-RESPONSE QUESTIONS

CHEMISTRY

Section II

(Total time—90 minutes)

Part A

Time—40 minutes

YOU MAY USE YOUR CALCULATOR FOR PART A.

CLEARLY SHOW THE METHOD USED AND THE STEPS INVOLVED IN ARRIVING AT YOUR ANSWERS.It is to your advantage to do this, since you may obtain partial credit if you do and you will receive little or no credit ifyou do not. Attention should be paid to significant figures.

Be sure to write all your answers to the questions on the lined pages following each question in the booklet with thepink cover. Do NOT write your answers on the green insert.

Answer Question 1 below. The Section II score weighting for this question is 20 percent.

I. Answer the following questions relating to the solubility of the chlorides of silver and lead.

(a) At 10°C, 8.9 x 10’ g of AgCI(s) will dissolve in 100. mL of water.

(i) Write the equation for the dissociation of AgC1(s) in water.

(ii) Calculate the solubility, in mol L, of AgCl(s) in water at 10°C.

(iii) Calculate the value of the solubility-product constant, Kçp, for AgCl(s) at 10°C.

(b) At 25°C, the value of K51, for PbCI2(s) is 1.6 x l0 and the value of K,1, for AgCl(s) is 1.8 x

(i) If 60.0 mL of 0.0400 M NaCI(aq) is added to 60.0 mL of 0.0300 M Pb(N03)7(aq), will a precipitateform? Assume that volumes are additive. Show calculations to support your answer.

(ii) Calculate the equilibrium value of [Pb2(aq)J in 1.00 L of saturated PbC1, solution to which 0.250mole of NaCl(s) has been added. Assume that no volume change occurs.

(iii) If 0.100 M NaCI(aq) is added slowly to a beaker containing both 0.120 M AgNO3(aq) and 0,150 ill

Pb(N03),(aq) at 25°C, which will precipitate first, AgCl(s) or PbCl,(s)? Show calculations to

support your answer.

Cops right © 200! by College Entrance Examination Board rights reserved.Advanced Placement Program and AP are registered trademarks of the College Entrance Examination Board.

GO ON TO THE NEXT PAGE.6

Handout Page 5 of 182 Handout Page 5 of 182

Handout Page 5 of 182 Updated 4/9/2014 Handout Page 5 of 182

Page 6: V1) I - Blue Valley Schools Chemistry/AP... · 3DJH RI 3DJH RI 2001 AP® CHEMISTRY FREE-RESPONSEQUESTIONS Answer EITHER Question 2 below OR Question 3 printed on page 8

2001 AP® CHEMISTRY FREE-RESPONSE QUESTIONS

Answer EITHER Question 2 below OR Question 3 printed on page 8. Only one of these two questions will begraded. If you start both questions, be sure to cross out the question you do not want graded. The Section II scoreweighting for the question you choose is 20 percent.

2NO(g)+02(g) —* 2N0,(g) H°=—ll4.l kJ, A°=—l46.5JK

2. The reaction represented above is one that contributes significantly to the formation of photochemical smog.

(a) Calculate the quantity of heat released when 73.1 g of NO(g) is converted to NO,(g).

(b) For the reaction at 25°C, the value of the standard free-energy change, G°, is —70.4 kJ.

(i) Calculate the value of the equilibrium constant, Keq. for the reaction at 25°C.

(ii) Indicate whether the value of G° would become more negative, less negative, or remain unchangedas the temperature is increased. Justify your answer.

(c) Use the data in the table below to calculate the value of the standard molar entropy, S°, for O,(g) at 25°C.

Standard Molar Entropy, S°

(J K mol)

NO(g) 210.8

N02(g) 240.1

(d) Use the data in the table below to calculate the bond energy, in kJ moI1,of the nitrogen-oxygen bond inNO2 . Assume that the bonds in the NO2 molecule are equivalent (i.e., they have the same energy).

Bond Energy

(kJ mol)

Nitrogen-oxygen bond in NO 607

Oxygen-oxygen bond in 0., 495

Nitrogen-oxygen bond in NO2

Copyright © 200! by College Entrance Examination Board. All rights reserved.Advanced Placement Program and AP are registered trademarks of the College Entrance Examination Board.

GO ON TO THE NEXT PAGE.7

Handout Page 6 of 182 Handout Page 6 of 182

Handout Page 6 of 182 Updated 4/9/2014 Handout Page 6 of 182

Page 7: V1) I - Blue Valley Schools Chemistry/AP... · 3DJH RI 3DJH RI 2001 AP® CHEMISTRY FREE-RESPONSEQUESTIONS Answer EITHER Question 2 below OR Question 3 printed on page 8

2001 AP® CHEMISTRY FREE-RESPONSE QUESTIONS

3. Answer the following questions about acetylsalicylic acid, the active ingredient in aspirin.

(a) The amount of acetylsalicylic acid in a single aspirin tablet is 325 mg, yet the tablet has a mass of 2.00 g.Calculate the mass percent of acetylsalicylic acid in the tablet.

(b) The elements contained in acetylsalicylic acid are hydrogen. carbon, and oxygen. The combustion of 3.000 gof the pure compound yields 1.200 g of water and 3.72 L of dry carbon dioxide, measured at 750. mm Hgand 25°C. Calculate the mass, in g, of each element in the 3.000 g sample.

(c) A student dissolved 1.625 g of pure acetylsalicylic acid in distilled water and titrated the resulting solution tothe equivalence point using 88.43 mL of 0.102 Al NaOH(aq). Assuming that acetylsalicylic acid has onlyone ionizable hydrogen, calculate the molar mass of the acid.

(d) A 2.00 x iO mole sample of pure acetylsalicylic acid was dissolved in 15.00 mL of water and then titratedwith 0.100 M NaOH(aq). The equivalence point was reached after 20.00 mL of the NaOH solution hadbeen added. Using the data from the titration, shown in the table below, determine

(i) the value of the acid dissociation constant, Ka, for acetylsaLicylic acid and

(ii) the pH of the solution after a total volume of 25.00 mL of the NaOH solutionhad been added (assume that volumes are additive).

Volume of0.100MNaOH pH

Added (mL)

0.0() 2.22

5.00 2.97

10.00 3.44

15.00 3.92

20.00 8,13

25.00 7

Copyright © 2001 by College Entrance Examination Board. All rights reserved,Advanced Placement Program and AP are registered trademarks of the College Entrance Examination Board.

GO ON TO THE NEXT PAGE.8

Handout Page 7 of 182 Handout Page 7 of 182

Handout Page 7 of 182 Updated 4/9/2014 Handout Page 7 of 182

Page 8: V1) I - Blue Valley Schools Chemistry/AP... · 3DJH RI 3DJH RI 2001 AP® CHEMISTRY FREE-RESPONSEQUESTIONS Answer EITHER Question 2 below OR Question 3 printed on page 8

2001 AP® CHEMISTRY FREE-RESPONSE QUESTIONS

CHEMISTRY

Part BTime—SO minutes

NO CALCULATORS MAY BE USED FOR PART B.

Answer Question 4 below, The Section II score weighting for this question is 15 percent.

4. Write the formulas to show the reactants and the products for any FIVE of the laboratory situations describedbelow. Answers to more than five choices will not be graded. In all cases, a reaction occurs. Assume thatsolutions are aqueous unless otherwise indicated. Represent substances in solution as ions if the substances areextensively ionized. Omit formulas for any ions or molecules that are unchanged by the reaction. You need notbalance the equations.

Exapic A strip of magnesium is added to a solution of silver nitrate.

£. + +

(a) Sulfur dioxide gas is bubbled into distilled water.

(b) A drop of potassium thiocyanate solution is added to a solution of iron(lII) nitrate.

(c) A piece of copper wire is placed in a solution of silver nitrate.

(d) Solutions of potassium hydroxide and propanoic acid are mixed.

(e) A solution of iron(ll) chloride is added to an acidified solution of sodium dichromate.

(f) Chlorine gas is bubbled through a solution of potassium bromide.

(g) Solutions of strontium nitrate and sodium sulfate are mixed.

(h) Powdered magnesium carbonate is heated strongly.

Copyright © 2001 by College Entrance Examination Board. All rights reserved.Advanced Placement Program and AP are registered trademarks of the College Entrance Examination Board.

GO ON TO THE NEXT PAGE.9

Handout Page 8 of 182 Handout Page 8 of 182

Handout Page 8 of 182 Updated 4/9/2014 Handout Page 8 of 182

Page 9: V1) I - Blue Valley Schools Chemistry/AP... · 3DJH RI 3DJH RI 2001 AP® CHEMISTRY FREE-RESPONSEQUESTIONS Answer EITHER Question 2 below OR Question 3 printed on page 8

2001 AP® CHEMISTRY FREE-RESPONSE QUESTIONS

Your responses to the rest of the questions in this part of the examination will be graded on the basis of the accuracyand relevance of the information cited. Explanations should be clear and well organized. Examples and equationsmay be included in your responses where appropriate. Specific answers are preferable to broad, diffuse responses.

Answer BOTH Question 5 below AND Question 6 printed on page 11, Both of these questions will be graded. TheSection II score weighting for these questions is 30 percent (15 percent each).

Solution 1 Solution 2 Solution 3 Solution 4 Solution 5

0.lOM 0.IOM 0.1OM 0.IOM 0.IOMPb(N01), NaCI KMnO4 C2H5OH KC7H3O,

5. Answer the questions below that relate to the five aqueous solutions at 25°C shown above.

(a) Which solution has the highest boiling point? Explain.

(b) Which solution has the highest pH? Explain.

(c) Identify a pair of the solutions that would produce a precipitate when mixed together. Write the formula ofthe precipitate.

(d) Which solution could be used to oxidize the Cl(aq) ion? Identify the product of the oxidation.

(e) Which solution would be the least effective conductor of electricity? Explain.

Copy roiht © 20(11 h College Entrance Examination Board. All rights reserved.Advanced Placement Program and AP are registered trademarks of the College Entrance Examination Board.

GO ON TO THE NEXT PAGE.10

Handout Page 9 of 182 Handout Page 9 of 182

Handout Page 9 of 182 Updated 4/9/2014 Handout Page 9 of 182

Page 10: V1) I - Blue Valley Schools Chemistry/AP... · 3DJH RI 3DJH RI 2001 AP® CHEMISTRY FREE-RESPONSEQUESTIONS Answer EITHER Question 2 below OR Question 3 printed on page 8

2001 AP® CHEMISTRY FREE-RESPONSE QUESTIONS

3 1(aq) + S2O82q) I3(aq) ÷ 2 S042(aq)

6. Iodide ion, L(aq), reacts with peroxydisulfate ion, SO82(aq), according to the equation above. Assume thatthe reaction goes to completion.

(a) Identify the type of reaction (combustion, disproportionation. neutralization, oxidation-reduction,precipitation, etc.) represented by the equation above. Also, give the formula of another substance that couldconvert r(aq) to I3(aq).

(b) In an experiment, equal volumes of 0.0 120 M 1(aq) and 0.0040 MS2O82(aq) are mixed at 25°C. The

concentration of I3(aq) over the following 80 minutes is shown in the graph below.

0.0030

030 45000 80Time (minutes)

(i) Indicate the time at which the reaction first reaches completion by marking an “X” on the curve aboveat the point that corresponds to this time. Explain your reasoning.

(ii) Explain how to determine the instantaneous rate of formation of 13(aq) at exactly 20 minutes.. Drawon the graph above as part of your explanation.

(c) Describe how to change the conditions of the experiment in part (b) to determine the order of the reactionwith respect to L(aq) pj with respect toS2082(aq).

(d) State clearly how to use the information from the results of the experiments in part (c) to determine the valueof the rate constant, k, for the reaction.

(e) On the graph below (which shows the results of the initial experiment as a dashed curve), draw in a curve forthe results you would predict if the initial experiment were to be carried out at 35°C rather than at 25°C.

0.0030

0.0020 . — [13j at 25°C

00010

I0 10 20 30 40 50 60 70 80

Time (minutes)

Copyright © 200! hy College Entrance Examination Board. All rights reserved.Advanced Placement Program and .AP are registered trademarks of the College Entrance Examination Board.

GO ON TO THE NEXT PAGE.11

Handout Page 10 of 182 Handout Page 10 of 182

Handout Page 10 of 182 Updated 4/9/2014 Handout Page 10 of 182

Page 11: V1) I - Blue Valley Schools Chemistry/AP... · 3DJH RI 3DJH RI 2001 AP® CHEMISTRY FREE-RESPONSEQUESTIONS Answer EITHER Question 2 below OR Question 3 printed on page 8

2001 AP® CHEMISTRY FREE-RESPONSE QUESTIONS

Answer EITHER Question 7 below OR Question 8 printed on page 13. Only one of these two questions will begraded. If you start both questions, be sure to cross out the question you do not want graded. The Section II scoreweighting for the question you choose is 15 percent.

Wire

7. Answer the following questions that refer to the galvanic cell shown in the diagram above. (A table of standardreduction potentials is printed on the green insert and on page 4 of the booklet with the pink cover.)

(a) Identify the anode of the cell and write the half-reaction that occurs there.

(b) Write the net ionic equation for the overall reaction that occurs as the cell operates and calculate the value

of the standard cell potential, Eeii.

(c) Indicate how the value of Ei.eii would be affected if the concentration of Ni(N03)2(aq) was changed

from 1.0 M to 0.10 M and the concentration of Zn(N03),(aq) remained at 1.0 M. Justify your answer.

(d) Specify whether the value of Keq for the cell reaction is less than I, greater than 1, or equal to 1. Justify

your answer.

Cop right © 200! by College Entrance Examination Board. All rights reserved.Advanced Placement Program and AP are registered trademarks of the College Entrance Examination Board.

GO ON TO THE NEXT PAGE.

Ni

l.OMNi(N03), l.OMZn(N01),

12

Handout Page 11 of 182 Handout Page 11 of 182

Handout Page 11 of 182 Updated 4/9/2014 Handout Page 11 of 182

Page 12: V1) I - Blue Valley Schools Chemistry/AP... · 3DJH RI 3DJH RI 2001 AP® CHEMISTRY FREE-RESPONSEQUESTIONS Answer EITHER Question 2 below OR Question 3 printed on page 8

2001 AP® CHEMISTRY FREE-RESPONSE QUESTIONS

8. Account for each of the following observations about pairs of substances. In your answers, use appropriateprinciples of chemical bonding and/or intermolecular forces. In each part, your answer must include referencesto both substances.

(a) Even though NH3 and CH3 have similar molecular masses, NH3 has a much higher normal boiling point

(—33°C) than CH4 (—164°C).

(b) At 25°C and 1.0 atm, ethane (C,H6)is a gas and hexane (C6H13)is a liquid.

(c) Si melts at a much higher temperature (1,410°C) than Cl2 (—101°C).

(d) MgO melts at a much higher temperature (2,852°C) than NaF (993°C),

END OF EXAMINATION

Copyright © 2001 by College Entrance Examination Board. All rights reserved.Advanced Placement Program and AP are registered trademarks of the College Entrance Examination Board.

13

Handout Page 12 of 182 Handout Page 12 of 182

Handout Page 12 of 182 Updated 4/9/2014 Handout Page 12 of 182

Page 13: V1) I - Blue Valley Schools Chemistry/AP... · 3DJH RI 3DJH RI 2001 AP® CHEMISTRY FREE-RESPONSEQUESTIONS Answer EITHER Question 2 below OR Question 3 printed on page 8

2002 AP® CHEMISTRY FREE-RESPONSE QUESTIONS

CHEMISTRY

Section II

(Total time—90 minutes)

Part A

Time—40 minutes

YOU MAY USE YOUR CALCULATOR FOR PART A.

CLEARLY SHOW THE METHOD USED AND THE STEPS INVOLVED IN ARRIVING AT YOUR ANSWERS.It is to your advantage to do this, since you may obtain partial credit if you do and you will receive little or no credit ifyou do not. Attention should be paid to significant figures.

Be sure to write all your answers to the questions on the lined pages following each question in the booklet with thepink cover. Do NOT write your answers on the green insert.

Answer Question I below. The Section II score weighting for this question is 20 percent.

HOBr(aq) H(aq) + OBr(aq) K0 = 2.3 x 10

I. Hypobromous acid, HOBr, is a weak acid that dissociates in water, as represented by the equation above.

(a) Calculate the value of [Hi in an HOBr solution that has a pH of 4.95.

(b) Write the equilibrium constant expression for the ionization of HOBr in water, then calculate theconcentration of HOBr(aq) in an HOBr solution that has [Hj equal to 1 .8 x I 0 M,

(c) A solution of Ba(OH)2 is titrated into a solution of HOBr.

(i) Calculate the volume of 0.115 M Ba(OH)2(aq) needed to reach the equivalence point when titratedinto a 65.0 mL sample of 0.146 M HOBr(aq),

(ii) Indicate whether the pH at the equivalence point is less than 7, equal to 7, or greater than 7. Explain.

(d) Calculate the number of moles of NaOBr(s) that would have to be added to i25 mL of 0.160 M HOBrto produce a buffer solution with [Wj = 5.00 x l0 M. Assume that volume change is negligible.

(e) HOBr is a weaker acid than HBrO3.Account for this fact in terms of molecular structure.

Copyright © 2002 by College Entrance Examination Board. All rights reserved,Advanced Placement Program and AP are registered trademarks of the College Entrance Examination Board,

6 GO ON TO THE NEXT PAGE.

Handout Page 13 of 182 Handout Page 13 of 182

Handout Page 13 of 182 Updated 4/9/2014 Handout Page 13 of 182

Page 14: V1) I - Blue Valley Schools Chemistry/AP... · 3DJH RI 3DJH RI 2001 AP® CHEMISTRY FREE-RESPONSEQUESTIONS Answer EITHER Question 2 below OR Question 3 printed on page 8

2002 AP® CHEMISTRY FREE-RESPONSE QUESTIONS

Answer EITHER Question 2 below OR Question 3 printed on page 8, Only one of these two questions will begraded. If you start both questions, be sure to cross out the question you do not want graded. The Section II scoreweighting for the question you choose is 20 percent.

2. Answer parts (a) through (e) below, which relate to reactions involving silver ion, Ag.

The reaction between silver ion and solid zinc is represented by the following equation.

2 Ag(aq) + Zn(s) —> Zn2(aq) + 2 Ag(s)

(a) A 1.50 g sample of Zn is combined with 250. mL of 0.110 M AgNO3 at 25°C.

(i) Identify the limiting reactant. Show calculations to support your answer.

(ii) On the basis of the limiting reactant that you identified in part (i), determine the value of [Zn2j afterthe reaction is complete. Assume that volume change is negligible.

(b) Determine the value of the standard potential, E°, for a galvanic cell based on the reaction betweenAgNO3(aq) and solid Zn at 25°C.

Another galvanic cell is based on the reaction between Ag(aq) and Cu(s), represented by the equation below.At 25°C, the standard potential, E°, for the cell is 0.46 V.

2 Ag(aq) + Cu(s) — Cu2(aq) + 2 Ag(s)

(c) Determine the value of the standard free-energy change, tG°, for the reaction between Ag(aq) and Cu(s)at 25°C.

(d) The cell is constructed so that [Cu2j is 0.045 M and [Ag] is 0.010 M. Calculate the value of thepotential, E, for the cell.

(e) Under the conditions specified in part (d), is the reaction in the cell spontaneous? Justify your answer.

Copyright © 2002 by College Entrance Examination Board. All rights reserved.Advanced Placement Program and AP are registered trademarks of the College Entrance Examination Board.

7 GO ON TO THE NEXT PAGE.

Handout Page 14 of 182 Handout Page 14 of 182

Handout Page 14 of 182 Updated 4/9/2014 Handout Page 14 of 182

Page 15: V1) I - Blue Valley Schools Chemistry/AP... · 3DJH RI 3DJH RI 2001 AP® CHEMISTRY FREE-RESPONSEQUESTIONS Answer EITHER Question 2 below OR Question 3 printed on page 8

2002 AP® CHEMISTRY FREE-RESPONSE QUESTIONS

3. Consider the hydrocarbon pentane, C5H12 (molar mass 72.15 g).

(a) Write the balanced equation for the combustion of pentane to yield carbon dioxide and water.

(b) What volume of dry carbon dioxide, measured at 25°C and 785 mm Hg, will result from the completecombustion of 2.50 g of pentane?

(c) The complete combustion of 5.00 g of pentane releases 243 kJ of heat. On the basis of this information,calculate the value of zS.H for the complete combustion of one mole of pentane.

(d) Under identical conditions, a sample of an unknown gas effuses into a vacuum at twice the rate that a sampleof pentane gas effuses. Calculate the molar mass of the unknown gas.

(e) The structural formula of one isomer of pentane is shown below. Draw the structural formulas for the othertwo isomers of pentane. Be sure to include all atoms of hydrogen and carbon in your structures.

H H H HF!I I I I I

H—C—C—C—C—C—HI I I I I

H H H H H

Copyright © 2002 by College Entrance Examination Board, All rights reserved,Advanced Placement Program and AP are registered trademarks of the College Entrance Examination Board.

8 GO ON TO THE NEXT PAGE.

Handout Page 15 of 182 Handout Page 15 of 182

Handout Page 15 of 182 Updated 4/9/2014 Handout Page 15 of 182

Page 16: V1) I - Blue Valley Schools Chemistry/AP... · 3DJH RI 3DJH RI 2001 AP® CHEMISTRY FREE-RESPONSEQUESTIONS Answer EITHER Question 2 below OR Question 3 printed on page 8

2002 AP® CHEMISTRY FREE-RESPONSE QUESTIONS

CHEMISTRYPart B

Time—SO minutes

NO CALCULATORS MAY BE USED FOR PART B.

Answer Question 4 below. The Section II score weighting for this question is 15 percent.

4. Write the formulas to show the reactants and the products for any FIVE of the laboratory situations describedbelow. Answers to more than five choices will not be graded. In all cases, a reaction occurs. Assume thatsolutions are aqueous unless otherwise indicated. Represent substances in solution as ions if the substances areextensively ionized. Omit formulas for any ions or molecules that are unchanged by the reaction. You need notbalance the equations.

Example: A strip of magnesium is added to a solution of silver nitrate.

+ A + A

(a) A solution of sodium iodide is added to a solution of lead(II) acetate.

(b) Pure solid phosphorus (white form) is bumed in air.

(c) Solid cesium oxide is added to water.

(d) Excess concentrated hydrochloric acid is added to a 1.0 M solution of cobalt(II) chloride.

(e) Solid sodium hydrogen carbonate (sodium bicarbonate) is strongly heated.

(0 An excess of hydrochloric acid is added to solid zinc sulfide.

(g) Acidified solutions of potassium permanganate and iron(ll) nitrate are mixed together.

(h) A solution of potassium hydroxide is added to solid ammonium chloride.

Copyright © 2002 by College Entrance Examination Board. All rights reserved,Advanced Placement Program and AP are registered trademarks of the College Entrance Examination Board.

9 GO ON TO THE NEXT PAGE.

Handout Page 16 of 182 Handout Page 16 of 182

Handout Page 16 of 182 Updated 4/9/2014 Handout Page 16 of 182

Page 17: V1) I - Blue Valley Schools Chemistry/AP... · 3DJH RI 3DJH RI 2001 AP® CHEMISTRY FREE-RESPONSEQUESTIONS Answer EITHER Question 2 below OR Question 3 printed on page 8

2002 AP® CHEMISTRY FREE-RESPONSE QUESTIONS

Your responses to the rest of the questions in this part of the examination will be graded on the basis of the accuracyand relevance of the information cited. Explanations should be clear and well organized. Examples and equationsmay be included in your responses where appropriate. Specific answers are preferable to broad, diffuse responses.

Answer BOTH Question 5 below AND Question 6 printed on page 11. Both of these questions will be graded. TheSection II score weighting for these questions is 30 percent (15 percent each).

H(aq) + OW(aq) — H,O(l)

5. A student is asked to determine the molar enthalpy of neutralization, AH,ze,,t, for the reaction represented above.The student combines equal volumes of 1.0 M HC1 and 1.0 Al NaOH in an open polystyrene cup calorimeter.

The heat released by the reaction is determined by using the equation q = ,ncT.

Assume the following.

• Both solutions are at the same temperature before they are combined.

• The densities of all the solutions are the same as that of water.

• Any heat lost to the calorimeter or to the air is negligible.

• The specific heat capacity of the combined solutions is the same as that of water.

(a) Give appropriate units for each of the terms in the equation q = mcT.

(b) List the measurements that must be made in order to obtain the value of q.

(c) Explain how to calculate each of the following.

(i) The number of moles of water formed during the experiment

(ii) The value of the molar enthalpy of neutralization, tHneut, for the reaction between HCI(aq) andNaOH(aq)

(d) The student repeats the experiment with the same equal volumes as before, but this time uses 2.0 M HCJand 2.0 M NaOH.

(i) Indicate whether the value of q increases, decreases, or stays the same when compared to the firstexperiment. Justify your prediction.

(ii) Indicate whether the value of the molar enthalpy of neutralization, increases, decreases, orstays the same when compared to the first experiment. Justify your prediction.

(e) Suppose that a significant amount of heat were lost to the air during the experiment. What effect would thishave on the calculated value of the molar enthalpy of neutralization, AH, ? Justify your answer.

Copvrwht ) 2002 by College Entrance Examination Board. All dghts reserved.Advanced Placement Program and AP are registered trademarks of the College Entrance Examination Board.

10 GO ON TO THE NEXT PAGE.

Handout Page 17 of 182 Handout Page 17 of 182

Handout Page 17 of 182 Updated 4/9/2014 Handout Page 17 of 182

Page 18: V1) I - Blue Valley Schools Chemistry/AP... · 3DJH RI 3DJH RI 2001 AP® CHEMISTRY FREE-RESPONSEQUESTIONS Answer EITHER Question 2 below OR Question 3 printed on page 8

2002 AP® CHEMISTRY FREE-RESPONSE QUESTIONS

6. Use the principles of atomic structure and/or chemical bonding to explain each of the following. In each part,your answer must include references to J2th substances.

(a) The atomic radius of Li is larger than that of Be.

(b) The second ionization energy of K is greater than the second ionization energy of Ca.

(c) The carbon-to-carbon bond energy in C7H4 is greater than it is in C,H6.

(d) The boiling point of Ci, is lower than the boiling point of Br2.

Answer EITHER Question 7 below OR Question 8 printed on page 12. Only one of these two questions will begraded. If you start both questions, be sure to cross out the question you do not want graded. The Section 11 scoreweighting for the question you choose is 15 percent.

7. An environmental concern is the depletion of 03 in Earth’s upper atmosphere, where 03 is normally inequilibrium with 02 and 0. A proposed mechanism for the depletion of 03 in the upper atmosphere isshown below.

Step I 03 + Cl —> 02 + CIO

Step II C1O + 0 — Cl + 02

(a) Write a balanced equation for the overall reaction represented by Step I and Step II above.

(b) Clearly identify the catalyst in the mechanism above. Justify your answer.

(c) Clearly identify the intermediate in the mechanism above. Justify your answer.

(d) If the rate law for the overall reaction is found to be rate = k[031[ClJ, determine the following.

(i) The overall order of the reaction

(ii) Appropriate units for the rate constant, k

(iii) The rate-determining step of the reaction, along with justification for your answer

Copyright © 2002 by College Entrance Examination Board. All rights reserved.Advanced Placement Program and AP are registered trademarks of the College Entrance Examination Board.

11 GO ON TO THE NEXT PAGE.

Handout Page 18 of 182 Handout Page 18 of 182

Handout Page 18 of 182 Updated 4/9/2014 Handout Page 18 of 182

Page 19: V1) I - Blue Valley Schools Chemistry/AP... · 3DJH RI 3DJH RI 2001 AP® CHEMISTRY FREE-RESPONSEQUESTIONS Answer EITHER Question 2 below OR Question 3 printed on page 8

2002 AP® CHEMISTRY FREE-RESPONSE QUESTIONS

C(s) + C07(g) 2 CO(g)

8. Carbon (graphite), carbon dioxide, and carbon monoxide form an equilibrium mixture, as represented by theequation above.

(a) Predict the sign for the change in entropy, iS’, for the reaction. Justify your prediction.

(b) In the table below are data that show the percent of CO in the equilibrium mixture at two differenttemperatures. Predict the sign for the chaige in enthalpy, All, for the reaction. Justify your prediction.

Temperature % CO700°C 60

850°C 94

(c) Appropriately complete the potential energy diagram for the reaction by finishing the curve on the graphbelow. Also, clearly indicate All for the reaction on the graph.

Reaction Coordinate

(d) If the initial amount of C(s) were doubled, what would be the effect on the percent of CO in theequilibrium mixture? Justify your answer.

END OF EXAMINATION

Copyright © 2002 by College Entrance Examination Board All rights reserved.Advanced Placement Program and AP are registered trademarks of the College Entrance Examination Board.

12

Handout Page 19 of 182 Handout Page 19 of 182

Handout Page 19 of 182 Updated 4/9/2014 Handout Page 19 of 182

Page 20: V1) I - Blue Valley Schools Chemistry/AP... · 3DJH RI 3DJH RI 2001 AP® CHEMISTRY FREE-RESPONSEQUESTIONS Answer EITHER Question 2 below OR Question 3 printed on page 8

2002 AP® CHEMISTRY FREE-RESPONSE QUESTIONS (Form B>

CHEMISTRY

Section II

(Total time—90 minutes)

Part A

Time—40 minutes

YOU MAY USE YOUR CALCULATOR FOR PART A.

CLEARLY SHOW THE METHOD USED AND THE STEPS INVOLVED IN ARRIVING AT YOUR ANSWERS.It is to your advantage to do this, since you may obtain partiai credit if you do and you will receive little or no credit ifyou do not. Attention should be paid to significant figures.

Be sure to write all your answers to the questions on the lined pages following each question in the booklet with thegoldenrod cover. Do NOT write your answers on the lavender insert.

Answer Question 1 below. The Section II score weighting for this question is 20 percent.

HC3H5O3(aq) H(aq) + C3H503(aq)

1. Lactic acid, HC3H5O3,is a monoprotic acid that dissociates in aqueous solution, as represented by the equation

above, Lactic acid is 1.66 percent dissociated in 0.50 M HC3H503(aq) at 298 K. For parts (a) through (d)

below, assume the temperature remains at 298 K.

(a) Write the expression for the acid-dissociation constant, Ka, for lactic acid and calculate its value.

(b) Calculate the pH of 0,50 M HC3H503.

(c) Calculate the pH of a solution formed by dissolving 0.045 mole of solid sodium lactate, NaC3H5O3,

in 250. mL of 0.50 M HC3H503.Assume that volume change is negligible.

(d) A 100. mL sample of 0.10 M HC1 is added to 100. mL of 0.50 M HC3H503.Calculate the molar

concentration of lactate ion, C3H503, in the resulting solution.

Copyright © 2002 by College Entrance Examination Board, All rights reserved.Advanced Placement Program and AP are registered trademarks of the College Entrance Examination Board.

6 GO ON TO THE NEXT PAGE.

Handout Page 20 of 182 Handout Page 20 of 182

Handout Page 20 of 182 Updated 4/9/2014 Handout Page 20 of 182

Page 21: V1) I - Blue Valley Schools Chemistry/AP... · 3DJH RI 3DJH RI 2001 AP® CHEMISTRY FREE-RESPONSEQUESTIONS Answer EITHER Question 2 below OR Question 3 printed on page 8

2002 AP® CHEMISTRY FREE-RESPONSE QUESTIONS (Form B)

Answer EITHER Question 2 below OR Question 3 printed on page 8. Only one of these two questions will begraded. If you start both questions, be sure to cross out the question you do not want graded. The Section II scoreweighting for the question you choose is 20 percent.

2. A rigid 8.20 L flask contains a mixture of 2.50 moles of H2, 0.500 mole of 02, and sufficient Ar so thatthe partial pressure of Ar in the flask is 2.00 atm. The temperature is 127°C.

(a) Calculate the total pressure in the flask.

(b) Calculate the mole fraction of H2 in the flask.

(c) Calculate the density (in g L’) of the mixture in the flask.

The mixture in the flask is ignited by a spark, and the reaction represented below occurs until one of thereactants is entirely consumed.

2 H2(g) + 02(g) — 2H20(g)

(d) Give the mole fraction of all species present in the flask at the end of the reaction.

Copyright © 2002 by College Entrance Examination Board. All rights reserved.Advanced Placement Program and AP are registered trademarks of the College Entrance Examination Board.

7 GO ON TO THE NEXT PAGE.

Handout Page 21 of 182 Handout Page 21 of 182

Handout Page 21 of 182 Updated 4/9/2014 Handout Page 21 of 182

Page 22: V1) I - Blue Valley Schools Chemistry/AP... · 3DJH RI 3DJH RI 2001 AP® CHEMISTRY FREE-RESPONSEQUESTIONS Answer EITHER Question 2 below OR Question 3 printed on page 8

2002 AP® CHEMISTRY FREE-RESPONSE QUESTIONS (Form B)

3 Nitrogen monoxide, NO(g), and carbon monoxide, CO(g), are air pollutants generated by automobiles, It has

been proposed that under suitable conditions these two gases could react to form N2(g) and CO,(g), which are

components of unpolluted air.

(a) Write a balanced equation for the reaction described above. Indicate whether the carbon in CO is oxidizedor whether it is reduced in the reaction. Justify your answer.

(b) Write the expression for the equilibrium constant, K,,, for the reaction.

(c) Consider the following thermodynamic data.

NO CO CO2AGJ° (kJmol’) +86.55 —137.15 —394.36

(i) Calculate the value of AG° for the reaction at 298 K.

(ii) Given that All0 for the reaction at 298 K is —746 kJ per mole of N2(g) formed, calculate the value

of AS° for the reaction at 298 K. Include units with your answer.

(d) For the reaction at 298 K, the value of K,, is 3.3 x 10120. In an urban area, typical pressures of the

gases in the reaction are NO = 5.0 x iO- atm, P0 = 5.0 x l0- atm, N2 = 0.781 atm, and

= 3.1 x l0” atm.

(i) Calculate the value of AG for the reaction at 298 K when the gases are at the partial pressures givenabove.

(ii) In which direction (to the right or to the left) will the reaction be spontaneous at 298 K with thesepartial pressures? Explain.

Copyright © 2002 by College Entrance Examination Board, All rights reserved.Advanced Placement Program and AP are registered trademarks of the College Entrance Examination Board.

8 GO ON TO THE NEXT PAGE.

Handout Page 22 of 182 Handout Page 22 of 182

Handout Page 22 of 182 Updated 4/9/2014 Handout Page 22 of 182

Page 23: V1) I - Blue Valley Schools Chemistry/AP... · 3DJH RI 3DJH RI 2001 AP® CHEMISTRY FREE-RESPONSEQUESTIONS Answer EITHER Question 2 below OR Question 3 printed on page 8

2002 AP® CHEMISTRY FREE-RESPONSE QUESTIONS (Form B)

CHEMISTRY

Part B

Time—SO minutes

NO CALCULATORS MAY BE USED FOR PART B.Answer Question 4 below. The Section II score weighting for this question is 15 percent.

4. Write the formulas to show the reactants and the products for any FIVE of the laboratory situations describedbelow. Answers to more than five choices will not be graded. In all cases, a reaction occurs. Assume thatsolutions are aqueous unless otherwise indicated. Represent substances in solution as ions if the substances areextensively ionized. Omit formulas for any ions or molecules that are unchanged by the reaction. You need notbalance the equations.

Example: A strip of magnesium is added to a solution of silver nitrate.

+ +

(a) A sample of 1-propanol is burned in air.

(h) Solutions of sodium chromate and lead nitrate are mixed.

(c) A bar of iron metal is added to a solution of iron(ffl) chloride.

(d) Concentrated ammonia solution is added to copper(II) sulfate solution.

(e) Sulfur dioxide gas is bubbled into a beaker of water.

(0 Equal volumes of 0.1 M sodium phosphate and 0.1 M hydrochloric acid are mixed.

(g) Hydrogen chloride gas is bubbled through a solution of potassium cyanide.

(h) Liquid bromine is carefully added to a solution of potassium iodide.

Copyright © 2002 by College Entrance Examination Board. All rights reserved.Advanced Placement Program and AP are registered trademarks of the College Entrance Examination Board.

9

Handout Page 23 of 182 Handout Page 23 of 182

Handout Page 23 of 182 Updated 4/9/2014 Handout Page 23 of 182

Page 24: V1) I - Blue Valley Schools Chemistry/AP... · 3DJH RI 3DJH RI 2001 AP® CHEMISTRY FREE-RESPONSEQUESTIONS Answer EITHER Question 2 below OR Question 3 printed on page 8

2002 AP® CHEMISTRY FREE-RESPONSE QUESTIONS (Form B)

Your responses to the rest of the questions in this part of the examination will be graded on the basis of the accuracyand relevance of the information cited, Explanations should be clear and well organized. Examples and equationsmay be included in your responses where appropriate, Specific answers are preferable to broad, diffuse responses.

Answer BOTH QuestionS below AND Question 6 printed on page 11. Both of these questions will be graded. TheSection II score weighting for these questions is 30 percent (15 percent each).

5. Consider five unlabeled bottles, each containing 5.0 g of one of the following pure salts.

AgC1 BaCl2 CoC12 NaC1 NH4CI

(a) Identify the salt that can be distinguished by its appearance alone. Describe the observation that supportsyour identification.

(b) Identify the salt that can be distinguished by adding 10 mL of H20 to a small sample of each of theremaining unidentified salts. Describe the observation that supports your identification.

(c) Identify a chemical reagent that could be added to the salt identified in part (b) to confirm the salt’s identity.Describe the observation that supports your confirmation.

(d) Identify the salt that can be distinguished by adding 1.0 M Na2SO4 to a small sample of each of theremaining unidentified salts. Describe the observation that supports your identification.

(e) Identify the salt that can be distinguished by adding 1.0 M NaOH to a small sample of each of theremaining unidentified salts. Describe the observation that supports your identification.

Cop3 right t) 2002 by College Entrance Examination Board, All rights reserved,Advanced Placement Program and AP are registered trademarks of the College Entrance Examination Board,

10

Handout Page 24 of 182 Handout Page 24 of 182

Handout Page 24 of 182 Updated 4/9/2014 Handout Page 24 of 182

Page 25: V1) I - Blue Valley Schools Chemistry/AP... · 3DJH RI 3DJH RI 2001 AP® CHEMISTRY FREE-RESPONSEQUESTIONS Answer EITHER Question 2 below OR Question 3 printed on page 8

2002 AP® CHEMISTRY FREE-RESPONSE QUESTIONS (Form B)

6. Using principles of chemical bonding and molecular geometry, explain each of the following observations.Lewis electron-dot diagrams and sketches of molecules may be helpful as part of your explanations. For eachobservation, your answer must include references to gfJ substances.

(a) The bonds in nitrite ion, N02”, are shorter than the bonds in nitrate ion, N03”.

(b) The CH2F2 molecule is polar, whereas the CF4 molecule is not.

(c) The atoms in a C2H4 molecule are located in a single plane, whereas those in a C,H6 molecule are not.

(d) The shape of a PF5 molecule differs from that of an IF molecule.

(e) HC1O3 is a stronger acid than HC1O.

Copyright © 2002 by College Entrance Examination Board. All rights reserved,Advanced Placement Program and AP are registered trademarks of the College Entrance Examination Board.

11

Handout Page 25 of 182 Handout Page 25 of 182

Handout Page 25 of 182 Updated 4/9/2014 Handout Page 25 of 182

Page 26: V1) I - Blue Valley Schools Chemistry/AP... · 3DJH RI 3DJH RI 2001 AP® CHEMISTRY FREE-RESPONSEQUESTIONS Answer EITHER Question 2 below OR Question 3 printed on page 8

2002 AP® CHEMISTRY FREE-RESPONSE QUESTIONS (Form B)

Answer EITHER Question 7 below OR Question 8 printed on page 13. Only one of these two questions will begraded. If you start both questions, be sure to cross out the question you do not want graded. The Section II scoreweighting for the question you choose is 15 percent.

7. The diagram below shows the experimental setup for a typical electrochemical cell that contains two standardhalf-cells. The cell operates according to the reaction represented by the following equation.

Zn(s) + Ni2(aq) — Ni(s) + Zn2(aq)

(a) Identify M and M2 in the diagram and specify the initial concentration for M2 in solution.

(b) Indicate which of the metal electrodes is the cathode. Write the balanced equation for the reaction thatoccurs in the half-cell containing the cathode.

(c) What would be the effect on the cell voltage if the concentration of Zn2 was reduced to 0.100 M in thehalf-cell containing the Zn electrode?

(d) Describe what would happen to the cell voltage if the salt bridge was removed. Explain.

Copyright © 2002 by College Entrance Examination Board. All rights reserved.Advanced Placement Program and AP are registered trademarks of the College Entrance Examination Board.

12

Handout Page 26 of 182 Handout Page 26 of 182

Handout Page 26 of 182 Updated 4/9/2014 Handout Page 26 of 182

Page 27: V1) I - Blue Valley Schools Chemistry/AP... · 3DJH RI 3DJH RI 2001 AP® CHEMISTRY FREE-RESPONSEQUESTIONS Answer EITHER Question 2 below OR Question 3 printed on page 8

2002 AP® CHEMISTRY FREE-RESPONSE QUESTIONS (Form B)

8. The graph below shows the result of the titration of a 25 mL sample of a 0.10 Al solution of a weak acid, HA,with a strong base, 0.10 Al NaOH.

13

II

10 15 20 25 30

Milliliters of 0.10 Al NaOH Added

(a) Describe two features of the graph above that identify HA as a weak acid.

(b) Describe one method by which the value of the acid-dissociation constant forthe graph above.

HA can be determined using

(c) On the graph above, sketch the titration curve that would result if 25 mL of 0.10 M HC1 were used insteadof 0.1OM HA.

(d) A 25 mL sample of 0.10 M HA is titrated with 0.20 Al NaOH.

(i) What volume of base must be added to reach the equivalence point?

(ii) The pH at the equivalence point for this titration is slightly higher than the pH at the equivalence pointin the titration using 0.10 Al NaOH. Explain.

END OF EXAMINATION

Copyright © 2002 by College Entrance Examination Board. All rights reserved.Advanced Placement Program and AP are registered trademarks of the College Entrance Examination Board.

pH

/,

7

5,——”

—*---**-3

I0

0iEJZE

13

Handout Page 27 of 182 Handout Page 27 of 182

Handout Page 27 of 182 Updated 4/9/2014 Handout Page 27 of 182

Page 28: V1) I - Blue Valley Schools Chemistry/AP... · 3DJH RI 3DJH RI 2001 AP® CHEMISTRY FREE-RESPONSEQUESTIONS Answer EITHER Question 2 below OR Question 3 printed on page 8

2003 AP® CHEMISTRY FREE-RESPONSE QUESTIONS

CHEMISTRY

Section II

(Total time—90 minutes)

Part A

Time—40 minutes

YOU MAY USE YOUR CALCULATOR FOR PART A.

CLEARLY SHOW THE METHOD USED AND THE STEPS INVOLVED IN ARRIVING AT YOUR ANSWERS.It is to your advantage to do this, since you may obtain partial credit if you do and you will receive little or no credit ifyou do not. Attention should be paid to significant figures.

Be sure to write all your answers to the questions on the lined pages following each question in the booklet with thepink cover. Do NOT write your answers on the green insert.

Answer Question 1 below. The Section II score weighting for this question is 20 percent.

C6H5NH2(aq) + H2O(1) C6H5NH3(aq) + OH(aq)

I. Aniline, a weak base, reacts with water according to the reaction represented above.

(a) Write the equilibrium constant expression, Kb, for the reaction represented above.

(b) A sample of aniline is dissolved in water to produce 25.0 mL of a 0. 10 M solution. The pH of the solutionis 8.82. Calculate the equilibrium constant, Kb, for this reaction.

(c) The solution prepared in part (b) is titrated with 0.10 M HCI. Calculate the pH of the solution when 5.0 mLof the acid has been added.

(d) Calculate the pH at the equivalence point of the titration in part (c).

(e) The pK, values for several indicators are given below. Which of the indicators listed is most suitable for this

titration? Justify your answer.

Indicator pK

: Erythrosine 3Litmus 7

i Thymolphthalein 10

Copyright © 2003 by College Entrance Examination Board. All rights reserved.Available to AP professionals at apcentraLcollegcboard.com and to

students and parents at wwwcollegeboard.com/apstudents.

GO ON TO THE NEXT PAGE.6

Handout Page 28 of 182 Handout Page 28 of 182

Handout Page 28 of 182 Updated 4/9/2014 Handout Page 28 of 182

Page 29: V1) I - Blue Valley Schools Chemistry/AP... · 3DJH RI 3DJH RI 2001 AP® CHEMISTRY FREE-RESPONSEQUESTIONS Answer EITHER Question 2 below OR Question 3 printed on page 8

2003 AP® CHEMISTRY FREE-RESPONSE QUESTIONS

Answer EITHER Question 2 below OR Question 3 printed on page 8. Only one of these two questions will begraded. If you start both questions, be sure to cross out the question you do not want graded. The Section II scoreweighting for the question you choose is 20 percent.

2. A rigid 5.00 L cylinder contains 24.5 g of N2(g) and 28.0 g of 07(g).

(a) Calculate the total pressure, in atm, of the gas mixture in the cylinder at 298 K.

(b) The temperature of the gas mixture in the cylinder is decreased to 280 K. Calculate each of the following.

(i) The mole fraction of N7(g) in the cylinder

(ii) The partial pressure, in atm, of N2(g) in the cylinder

(c) If the cylinder develops a pinhole-sized leak and some of the gaseous mixture escapes, would the

moles of N(g) . .

ratio in the cylinder increase, decrease, or remain the same? Justify your answer.moles of °2 (g)

A different rigid 5.00 L cylinder contains 0.176 mol of NO(g) at 298 K. A 0.176 mol sample of 02(g) isadded to the cylinder, where a reaction occurs to produce N02(g).

(d) Write the balanced equation for the reaction.

(e) Calculate the total pressure, in atm, in the cylinder at 298 K after the reaction is complete.

Copyright © 2003 by College Entrance Examination Board. All rights reserved.Available to AP professionals at apcentraIcol1egeboardcom and to

students and parents at www.collegeboardcom/apstudents.

GO ON TO THE NEXT PAGE.7

Handout Page 29 of 182 Handout Page 29 of 182

Handout Page 29 of 182 Updated 4/9/2014 Handout Page 29 of 182

Page 30: V1) I - Blue Valley Schools Chemistry/AP... · 3DJH RI 3DJH RI 2001 AP® CHEMISTRY FREE-RESPONSEQUESTIONS Answer EITHER Question 2 below OR Question 3 printed on page 8

2003 AP® CHEMISTRY FREE-RESPONSE QUESTIONS

5 Br(aq) + BrO(aq) + 6 H(aq) — 3 Br,(i) + 3 H,O(1)

3. In a study of the kinetics of the reaction represented above, the following data were obtained at 298 K.

1 Rate of

. Initial [Br—I Initial [BrO3j Initial [HJ DisappearanceExperiment

(mol L) (mol L) (mol L) of Br03

(mol L’ s)

1 0.00100 0.00500 0.100 2.50 x iO

2 0.00200 0.00500 0.100 5.OOx l0

3 0.00100 0.00750 0.100 3.75x 10

4 0.00100 0.01500 0.200 3.OOx i0

(a) From the data given above, determine the order of the reaction for each reactant listed below. Show yourreasoning.

(i) Br

(ii) Br03

(iii) H

(b) Write the rate law for the overall reaction.

(c) Determine the value of the specific rate constant for the reaction at 298 K. Include the correct units.

(d) Calculate the value of the standard cell potential, E°, for the reaction using the information in the tablebelow.

Half-reaction E° (V)

Br,(I) + 2 e — 2 Br(aq) + 1.065

_BrO3(aq) ± 6H(aq) ± 5 e — - Br,(i) + 3 1-120(1) +1.52

(e) Determine the total number of electrons transferred in the overall reaction.

STOPIf you finish before time is called, you may check your work on this part only.

Do not turn to the other part of the test until you are told to do so.

Copyright © 2003 by College Entrance Examination Board. All rights reserved.Available to AP professionals at apcentral.collegeboard,com and to

students and parents at a w w collegeboard.com/apstudents.

8

Handout Page 30 of 182 Handout Page 30 of 182

Handout Page 30 of 182 Updated 4/9/2014 Handout Page 30 of 182

Page 31: V1) I - Blue Valley Schools Chemistry/AP... · 3DJH RI 3DJH RI 2001 AP® CHEMISTRY FREE-RESPONSEQUESTIONS Answer EITHER Question 2 below OR Question 3 printed on page 8

2003 AP® CHEMISTRY FREE-RESPONSE QUESTIONS

CHEMISTRY

Part B

Time—50 minutes

NO CALCULATORS MAY BE USED FOR PART B.

Answer Question 4 below. The Section II score weighting for this question is 15 percent.

4. Write the formulas to show the reactants and the products for any FIVE of the laboratory situations describedbelow. Answers to more than five choices will not be graded. In all cases, a reaction occurs, Assume thatsolutions are aqueous unless otherwise indicated. Represent substances in solution as ions if the substances areextensively ionized. Omit formulas for any ions or molecules that are unchanged by the reaction. You need notbalance the equations.

Example: A strip of magnesium is added to a solution of silver nitrate.

. + +

(a) A solution of potassium phosphate is mixed with a solution of calcium acetate.

(b) Solid zinc carbonate is added to 1.0 M sulfuric acid.

(c) A solution of hydrogen peroxide is exposed to strong sunlight.

(d) A 0.02 M hydrochloric acid solution is mixed with an equal volume of a 0.01 M calcium hydroxidesolution.

(e) Excess concentrated aqueous ammonia is added to solid silver chloride.

(0 Magnesium ribbon is burned in oxygen.

(g) A bar of strontium metal is immersed in a 1.0 ill coppertil) nitrate solution.

(h) Solid dinitrogen pentoxide is added to water.

Copyright © 2003 by College Entrance Examination Board. All rights reserved.Available to AP professionals at apcentraLcollegeboard.com and to

students and parents at wwwcollegeboard.com!apstudents.

GO ON TO THE NEXT PAGE.9

Handout Page 31 of 182 Handout Page 31 of 182

Handout Page 31 of 182 Updated 4/9/2014 Handout Page 31 of 182

Page 32: V1) I - Blue Valley Schools Chemistry/AP... · 3DJH RI 3DJH RI 2001 AP® CHEMISTRY FREE-RESPONSEQUESTIONS Answer EITHER Question 2 below OR Question 3 printed on page 8

2003 AP® CHEMISTRY FREE-RESPONSE QUESTIONS

Your responses to the rest of the questions in this part of the examination will be graded on the basis of the accuracyand relevance of the information cited. Explanations should be clear and well organized. Examples and equationsmay be included in your responses where appropriate. Specific answers are preferable to broad, diffuse responses.

Answer BOTH Question 5 below AND Question 6 printed on page 12. Both of these questions will be graded. TheSection II score weighting for these questions is 30 percent (15 percent each).

5. A student is instructed to determine the concentration of a solution of CoCI, based on absorption of light(spectrometric/colorimetric method). The student is provided with a 0.10 M solution of CoCl2 with which toprepare standard solutions with concentrations of 0.020 M, 0,040 M, 0.060 M, and 0.080 M.

(a) Describe the procedure for diluting the 0.10 M solution to a concentration of 0.020 M using distilled water,a 100 mL volumetric flask, and a pipet or buret. hiclude specific amounts where appropriate.

The student takes the 0.10 M solution and determines the percent transmittance and the absorbance at variouswavelengths. The two graphs below represent the data.

V

-I

V0

(b) Identify the optimum wavelength for the analysis.

Copyright © 2003 by College Entrance Examination Board, All rights resersed.Available to AP professionals at apccntral.collegeboard.com and to

students and parents at swv .collegeboard.comlapstudents.

GO ON TO THE NEXT PAGE.

In”t’J’J

20360

0.60

0.40

0.20

0360

-----c400 440 480 520 560 600

Wavelength (nm)400 440 480 520 560 600

Wavelength (nm)

10

Handout Page 32 of 182 Handout Page 32 of 182

Handout Page 32 of 182 Updated 4/9/2014 Handout Page 32 of 182

Page 33: V1) I - Blue Valley Schools Chemistry/AP... · 3DJH RI 3DJH RI 2001 AP® CHEMISTRY FREE-RESPONSEQUESTIONS Answer EITHER Question 2 below OR Question 3 printed on page 8

2003 AP® CHEMISTRY FREE-RESPONSE QUESTIONS

The student measures the absorbance of the 0.020 M, 0.040 M, 0.060 M, 0.080 M, and 0.10 M solutions. Thedata are plotted below.

0.60

0.50

0.40

0.30

0.20

0.100.020 0.040 0.060 0.080 0.10

Concentration (M)

(c) The absorbance of the unknown solution is 0.275. What is the concentration of the solution?

(d) Beer’s Law is an expression that includes three factors that determine the amount of light that passes througha solution, identify two of these factors.

(e) The student handles the sample container (e.g., test tube or cuvette) that holds the unknown solution andleaves fingerprints in the path of the light beam. How will this affect the calculated concentration of theunknown? Explain your answer.

(1) Why is this method of determining the concentration of CoCl2 solution appropriate, whereas using thesame method for measuring the concentration of NaCl solution would not be appropriate?

Copyright © 2003 by College Entrance Examination Board. All rights reserved.Available to AP professionals at apccntral.collegeboard.com and to

students and parents at www.collcgeboard.com!apstudents.

GO ON TO THE NEXT PAGE.

C)

0.

__

11

Handout Page 33 of 182 Handout Page 33 of 182

Handout Page 33 of 182 Updated 4/9/2014 Handout Page 33 of 182

Page 34: V1) I - Blue Valley Schools Chemistry/AP... · 3DJH RI 3DJH RI 2001 AP® CHEMISTRY FREE-RESPONSEQUESTIONS Answer EITHER Question 2 below OR Question 3 printed on page 8

2003 AP® CHEMISTRY FREE-RESPONSE QUESTIONS

6. For each of the following, use appropriate chemical principles to explain the observation. Include chemicalequations as appropriate.

(a) In areas affected by acid rain, statues and structures made of limestone (calcium carbonate) often show signsof considerable deterioration,

(b) When table salt (NaCl) and sugar (C12H2201) are dissolved in water, it is observed that

(i) both solutions have higher boiling points than pure water, and

(ii) the boiling point of 0.10 M NaCI(aq) is higher than that of 0.10 M C12H22011(aq).

(c) Methane gas does not behave as an ideal gas at low temperatures and high pressures.

(d) Water droplets form on the outside of a beaker containing an ice bath.

Copyright © 2003 by College Entrance Examination Board. All rights reserved.Available to AP professionals at apcentraLcollegeboardcom and to

students and parents at wwwcollegeboardcom/apstudents.

GO ON TO THE NEXT PAGE.12

Handout Page 34 of 182 Handout Page 34 of 182

Handout Page 34 of 182 Updated 4/9/2014 Handout Page 34 of 182

Page 35: V1) I - Blue Valley Schools Chemistry/AP... · 3DJH RI 3DJH RI 2001 AP® CHEMISTRY FREE-RESPONSEQUESTIONS Answer EITHER Question 2 below OR Question 3 printed on page 8

2003 AP® CHEMISTRY FREE-RESPONSE QUESTIONS

Answer EITHER Question 7 below OR Question 8 printed on page 14. Only one of these two questions will begraded. If you start both questions, be sure to cross out the question you do not want graded. The Section II scoreweighting for the question you choose is 15 percent.

7. Answer the following questions that relate to the chemistry of nitrogen.

(a) Two nitrogen atoms combine to form a nitrogen molecule, as represented by the following equation.

2 N(g) — N2(g)

Using the table of average bond energies below, determine the enthalpy change, MI, for the reaction.

BondAverage Bond Energy

(kJ mo1)

N—N 160

N=N 420

NN 950

(b) The reaction between nitrogen and hydrogen to form ammonia is represented below.

N,(g) + 3 H2(g) —* 2 NH3(g) = —92.2 kJ

Predict the sign of the standard entropy change, i50, for the reaction. Justify your answer.

(C) The value of AG° for the reaction represented in part (b) is negative at low temperatures but positive at hightemperatures. Explain.

(d) When N2(g) and H2(g) are placed in a sealed container at a low temperature, no measurable amount ofNH3(g) is produced. Explain.

Copyright © 2003 by College Entrance Examination Board, All rights reserved.Available to AP professionals at apcentral.collegeboard.com and to

students and parents at www.collegeboard.com/apstudents.

GO ON TO THE NEXT PAGE.13

Handout Page 35 of 182 Handout Page 35 of 182

Handout Page 35 of 182 Updated 4/9/2014 Handout Page 35 of 182

Page 36: V1) I - Blue Valley Schools Chemistry/AP... · 3DJH RI 3DJH RI 2001 AP® CHEMISTRY FREE-RESPONSEQUESTIONS Answer EITHER Question 2 below OR Question 3 printed on page 8

2003 AP® CHEMISTRY FREE-RESPONSE QUESTIONS

Compound Compound \Name Formula (kJ mo[1)

Propane CH1CH,CH3 19.0

Propanone CH3COCH3 32.0

l-propanol CH3CH,CH,OH 47.3

8. Using the information in the table above, answer the following questions about organic compounds.

(a) For propanone,

(i) draw the complete structural formula (showing all atoms and bonds);

(ii) predict the approximate carbon-to-carbon-to-carbon bond angle.

(b) For each pair of compounds below, explain why they do not have the same value for their standard heat of

vaporization, A Hi,,. (You must include specific information about J2 compounds in each pair.)

(i) Propane and propanone

(ii) Propanone and 1-propanol

(c) Draw the complete structural formula for an isomer of the molecule you drew in part (a) (i).

(d) Given the structural formula for propyne below,

H--CC-H

H

(i) indicate the hybridization of the carbon atom indicated by the arrow in the structure above;

(ii) indicate the total number of sigma (G) bonds and the total number of pi (TT) bonds in the molecule.

END OF EXAMINATION

Copyright © 2003 by College Entrance Examination Board. All rights reserved,Available to AP professionals at apcentralcollegeboard.com and to

students and parents at www.collegeboard.com/apstudents.

14

Handout Page 36 of 182 Handout Page 36 of 182

Handout Page 36 of 182 Updated 4/9/2014 Handout Page 36 of 182

Page 37: V1) I - Blue Valley Schools Chemistry/AP... · 3DJH RI 3DJH RI 2001 AP® CHEMISTRY FREE-RESPONSEQUESTIONS Answer EITHER Question 2 below OR Question 3 printed on page 8

2003 AP® CHEMISTY FREE-RESPONSE QUESTIONS (Form B)

CHEMISTRYSection II

(Total time—90 minutes)

Part A

Time—40 minutes

YOU MAY USE YOUR CALCULATOR FOR PART A.

CLEARLY SHOW THE METhOD USED AND THE STEPS INVOLVED IN ARRIVING AT YOUR ANSWERS.It is to your advantage to do this, since you may obtain partial credit if you do and you will receive little or no credit ifyou do not. Attention should be paid to significant figures.

Be sure to write all your answers to the questions on the lined pages following each question in the booklet with thegoldenrod cover. Do NOT write your answers on the lavender insert.

Answer Question 1 below. The Section II score weighting for this question is 20 percent.

2 HI(g) H2(g) + 12(g)

1. After a 1.0 mole sample of HI(g) is placed into an evacuated 1.0 L container at 700. K, the reaction representedabove occurs. The concentration of HI(g) as a function of time is shown below.

7 0.80

C2‘ 0.60C

0.40

(a) Write the expression for the equilibrium constant. K. for the reaction.

Ib) What is tHu at equilibrium?

Copyright © 2003 by College Entrance Examination Board. All rights reserved.Available to AP professionals at apcentral.collegeboard.com and to

students and parents at www.collegeboard.com/apstudents.

GO ON TO THE NEXT PAGE.

1.00\

-

— [HI]

F —

(1 In

0

-rTZ. 4

0 Time

6

Handout Page 37 of 182 Handout Page 37 of 182

Handout Page 37 of 182 Updated 4/9/2014 Handout Page 37 of 182

Page 38: V1) I - Blue Valley Schools Chemistry/AP... · 3DJH RI 3DJH RI 2001 AP® CHEMISTRY FREE-RESPONSEQUESTIONS Answer EITHER Question 2 below OR Question 3 printed on page 8

2003 AP® CHEMISTY FREE-RESPONSE QUESTIONS (Form B)

(c) Determine the equilibrium concentrations of H2(g) and 12(g).

(d) On the graph above, make a sketch that shows how the concentration of H2(g) changes as a functionof time.

(e) Calculate the value of the following equilibrium constants at 700. K.

(I) K

(ii K,,

(f At 1,000 K. the value of K for the reaction is 2.6 x 102. In an experiment, 0.75 mole of HI(g),0.10 mole of H2(g), and 0.50 mole of 12(g) are placed in a 1.0 L container and allowed to reachequilibrium at 1,000 K. Determine whether the equilibrium concentration of HI(g) will be greater than,equal to, or less than the initial concentration of HI(g). Justify your answer,

Copyright © 2003 by College Entrance Examination Board. All rights reserved,Available to AP professionals at apcentral.collcgehoard.com and to

students and parents at www.collegehoard.eomlapstudents.

GO ON TO THE NEXT PAGE.7

Handout Page 38 of 182 Handout Page 38 of 182

Handout Page 38 of 182 Updated 4/9/2014 Handout Page 38 of 182

Page 39: V1) I - Blue Valley Schools Chemistry/AP... · 3DJH RI 3DJH RI 2001 AP® CHEMISTRY FREE-RESPONSEQUESTIONS Answer EITHER Question 2 below OR Question 3 printed on page 8

2003 AP® CI-IEMISTY FREE-RESPONSE QUESTIONS (Form B>

Answer EITHER Question 2 below OR Question 3 printed on page 9. Only one of these two questions will begraded. If you start both questions, be sure to cross out the question you do not want graded. The Section II scoreweighting for the question you choose is 20 percent.

2. Answer the following questions that relate to chemical reactions.

(a) Iron(ffl) oxide can be reduced with carbon monoxide according to the following equation.

Fe,03(s) + 3 CO(g) — 2 Fe(s) + 3 C07(g)

A 16.2 L sample of CO(g) at 1.50 atm and 200.°C is combined with 15.39 g of Fe,03(s).

(1) How many moles of CO(g) are available for the reaction?

(ii) What is the limiting reactant for the reaction? Justify your answer with calculations.

(iii) How many moles of Fe(s) are formed in the reaction?

(b) In a reaction vessel. 0.600 mol of Ba(N03)2(s) and 0.300 mol of H3P04(aq) are combined with deionizedwater to a final volume of 2.00 L. The reaction represented below occurs.

3 Ba(N03)2(aq) + 2H3PO4(aq) —* Ba3(P04)2(s) + 6 HNO3(aq)

(i) Calculate the mass of Ba3(P04),(s) formed.

(ii) Calculate the pH of the resulting solution.

(iii) What is the concentration, in mol L, of the nitrate ion, NO3-(aq), after the reaction reachescompletion?

Cops right © 2003 by College Entrance Examination Board. All rights reserved.Asatlable to AP professionals at apcentraLcoflegeboard.com and to

students and parents at wwwcollegeboardcomIapseudents.

GO ON TO THE NEXT PAGE.8

Handout Page 39 of 182 Handout Page 39 of 182

Handout Page 39 of 182 Updated 4/9/2014 Handout Page 39 of 182

Page 40: V1) I - Blue Valley Schools Chemistry/AP... · 3DJH RI 3DJH RI 2001 AP® CHEMISTRY FREE-RESPONSEQUESTIONS Answer EITHER Question 2 below OR Question 3 printed on page 8

2003 AP® CHEMISTY FREE-RESPONSE QUESTIONS (Form B)

3. In an experiment, a sample of an unknown, pure gaseous hydrocarbon was analyzed. Results showed that thesample contained 6.000 g of carbon and 1.344 g of hydrogen.

(a) Determine the empirical formula of the hydrocarbon.

(b) The density of the hydrocarbon at 25°C and 1.09 atm is 1.96 g L’.

(i) Calculate the molar mass of the hydrocarbon.

(ii) Determine the molecular formula of the hydrocarbon.

In another experiment, liquid heptane. C7H16(1. is completely combusted to produce C02(g) and H,O(!), asrepresented by the following equation.

C7H16(/) + 11 O4g) —* 7 C02(g) + 8 H20(1)

The heat of combustion, AH,,,h, for one mole of C7H16(/) is —4.85 x l0 ki.

(c) Using the information in the table below, calculate the value of for C7H16(O in kJ mol*

Compound J AH (kJ mol )CO(g) —393.5

H,O(1 —285.8

(d) A 0.0 108 mol sample of C7H16(l) is combusted in a bomb calorimeter.

(i) Calculate the amount of heat released to the calorimeter.

(ii) Given that the total heat capacity of the calorimeter is 9.273 kJ °C ‘, calculate the temperature changeof the calorimeter.

Copyright © 2003 by College Entrance Examination Board, Al) rights reserved.Available to AP professionals at apcentral.collegeboard.com and to

students and parents at www.collegeboard.com/apstudents.

GO ON TO THE NEXT PAGE.9

Handout Page 40 of 182 Handout Page 40 of 182

Handout Page 40 of 182 Updated 4/9/2014 Handout Page 40 of 182

Page 41: V1) I - Blue Valley Schools Chemistry/AP... · 3DJH RI 3DJH RI 2001 AP® CHEMISTRY FREE-RESPONSEQUESTIONS Answer EITHER Question 2 below OR Question 3 printed on page 8

2003 AP® CHEMISTY FREE-RESPONSE QUESTIONS (Form B)

CHEMISTRY

Part B

Time—50 minutes

NO CALCULATORS MAY BE USED FOR PART B.

Answer Question 4 below. The Section II score weighting for this question is 15 percent.

4. Write the formulas to show the reactants and the products for any FIVE of the laboratory situations describedbelow. Answers to more than five choices will not be graded. In all cases, a reaction occurs. Assume thatsolutions are aqueous unless otherwise indicated. Represent substances in solution as ions if the substances areextensively ionized. Omit formulas for any ions or molecules that are unchanged by the reaction. You need notbalance the equations.

Example: A strip of magnesium is added to a solution of silver nitrate.

. + +

(a) Hot hydrogen gas is passed over heated copper(II) oxide solid.

(b) Solid sodium hydride is added to water.

(c) Propanone is burned in air.

(d) A solution of lead(lI) nitrate is added to a solution of potassium sulfate.

(e) Ammonia gas is mixed with hydrogen chloride gas.

(f) Sulfur trioxide gas is bubbled into water.

(g) Excess concentrated potassium hydroxide solution is added to a solution of nickel(II) chloride.

(h) Solid sodium acetate is added to 1.0 M hydrobromic acid.

Copyright 2003 by College Entrance Examination Board. All rights resersed.Available to AP professionals at apcentral.collegeboard.com and to

students asid parents at www.collegeboard.com/apstudents.

GO ON TO THE NEXT PAGE.10

Handout Page 41 of 182 Handout Page 41 of 182

Handout Page 41 of 182 Updated 4/9/2014 Handout Page 41 of 182

Page 42: V1) I - Blue Valley Schools Chemistry/AP... · 3DJH RI 3DJH RI 2001 AP® CHEMISTRY FREE-RESPONSEQUESTIONS Answer EITHER Question 2 below OR Question 3 printed on page 8

2003 AP® CHEMISTY FREE-RESPONSE QUESTIONS (Form B)

Your responses to the rest of the questions in this part of the examination will be graded on the basis of the accuracyand relevance of the information cited. Explanations should be clear and well organized. Examples and equationsmay be included in your responses where appropriate. Specific answers are preferable to broad, diffuse responses.

Answer BOTH Question 5 below AND Question 6 printed on page 12. Both of these questions will be graded. TheSection II score weighting for these questions is 30 percent (15 percent each).

5. Oxalic acid, H2C204,is a primary standard used to determine the concentration of potassium permanganate,KMnO4 , in solution. The equation for the reaction is as follows.

2 KMnO4(aq) + 5 H,C204(aq) + 3H2S04(aq) — 2 MnSO3(aq) + 10 C02(’g) + 8 H20(l) + K7S04(aq)

A student dissolves a sample of oxalic acid in a flask with 30 mL of water and 2.00 mL of 3.00 M H,S04 . TheKMnO3 solution of unknown concentration is in a 25.0 mL buret. In the titration, the KMnO4 solution is addedto the solution containing oxalic acid.

(a) What chemical species is being oxidized in the reaction?

(b) What substance indicates the observable endpoint of the titration? Describe the observation that shows theendpoint has been reached.

(c) What data must be collected in the titration in order to determine the molar concentration of the unknownKMnO4 solution?

(d) Without doing any calculations, explain how to determine the molarity of the unknown KMnO4 solution.

(e) How would the calculated concentration of the KMnO4 solution be affected if 40 mL of water was added tothe oxalic acid initially instead of 30 mL? Explain your reasoning.

Copyright @ 2003 by College Entrance Examination Board, All rights reserved.Available to AP professionals at apcentral.collegeboard.corn and to

students and parents at www.collegehoard.com/apstudents.

GO ON TO THE NEXT PAGE.11

Handout Page 42 of 182 Handout Page 42 of 182

Handout Page 42 of 182 Updated 4/9/2014 Handout Page 42 of 182

Page 43: V1) I - Blue Valley Schools Chemistry/AP... · 3DJH RI 3DJH RI 2001 AP® CHEMISTRY FREE-RESPONSEQUESTIONS Answer EITHER Question 2 below OR Question 3 printed on page 8

2003 AP® CHEMISTY FREE-RESPONSE QUESTIONS (Form B)

6. Answer the following questions about electrochemistry.

(a) Several different electrochemical cells can be constructed using the materials shown below. Write thebalanced net-ionic equation for the reaction that occurs in the cell that would have the greatest positivevalue of E.11.

1.0 M Cu(N03)2 1.0 M Fe(N03)2

Al Metal Cu Metal Fe MetalStrip Strip Strip Solution to

Fill Salt Bridge

eterwith Wire

(b) Calculate the standard cell potential, Eeii. for the reaction written in part (a).

(c) A cell is constructed based on the reaction in part (a) above. Label the metal used for the anode on the cellshown in the figure below.

Anode

(Anode Metal)

Copyright © 2003 by College Entrance Examination Board. All rights reserved.Available to AP professionals at apcentral.collegeboard.corn and to

students and parents at www.eollegeboard.com/apstudents.

GO ON TO THE NEXT PAGE.

1.0 M Al(N03)3

Salt Bridge

12

Handout Page 43 of 182 Handout Page 43 of 182

Handout Page 43 of 182 Updated 4/9/2014 Handout Page 43 of 182

Page 44: V1) I - Blue Valley Schools Chemistry/AP... · 3DJH RI 3DJH RI 2001 AP® CHEMISTRY FREE-RESPONSEQUESTIONS Answer EITHER Question 2 below OR Question 3 printed on page 8

2003 AP® CHEMISTY FREE-RESPONSE QUESTIONS (Form B)

(d) Of the compounds NaOH. CuS, and NaNO, which one is appropriate to use in a salt bridge? Brieflyexplain your answer, and for each of the other compounds, include a reason why it is not appropriate.

(e) Another standard cell is based on the following reaction.

Zn + Pb2 —* Zn2 + Pb

If the concentration of Zn2 is decreased from 1.0 M to 0.25 M, what effect does this have on the cellpotential? Justify your answer.

Answer EITHER Question 7 below OR Question 8 printed on page 14. Only one of these two questions will begraded. If you start both questions, be sure to cross out the question you do not want graded. The Section II scoreweighting for the question you choose is 15 percent.

7. Account for the following observations using principles of atomic structure andlor chemical bonding. In eachpart, your answer must include specific information about both substances.

(a) The Ca2 and C ions are isoelectronic, but their radii are not the same. Which ion has the larger radius?Explain.

(b) Carbon and lead are in the same group of elements, but carbon is classified as a nonmetal and lead isclassified as a metal.

(c) Compounds containing Kr have been synthesized, but there are no known compounds that contain He.

(d) The first ionization energy of Be is 900 kJ moL1.but the first ionization energy of B is 800 kJ moLt.

Copyright 2003 by College Entrance Examination Board. All rights reserved,Available to AP professionals at apcentral.collegehoard.com and to

students and parents at www.collegeboard.com/apstudents.

GO ON TO THE NEXT PAGE.13

Handout Page 44 of 182 Handout Page 44 of 182

Handout Page 44 of 182 Updated 4/9/2014 Handout Page 44 of 182

Page 45: V1) I - Blue Valley Schools Chemistry/AP... · 3DJH RI 3DJH RI 2001 AP® CHEMISTRY FREE-RESPONSEQUESTIONS Answer EITHER Question 2 below OR Question 3 printed on page 8

2003 AP® CHEMISTY FREE-RESPONSE QUESTIONS (Form B)

8. The decay of the radioisotope 1-131 was studied in a laboratory. 1-131 is known to decay by beta ( e) emission.

(a) Write a balanced nuclear equation for the decay of 1-131.

(b) What is the source of the beta particle emitted from the nucleus?

The radioactivity of a sample of 1-131 was measured. The data collected are plotted on the graph below.

NESEEZE

O 4 8 12 16 20 24Time (days)

(c) Determine the half-life, t12, of 1-131 using the graph above.

(d) The data can be used to show that the decay of 1-131 is a first-order reaction, as indicated on the graphbelow.

0 4 8 12 16 20 24Time (days)

(i) Label the vertical axis of the graph above.

(ii) What are the units of the rate constant, k. for the decay reaction?

(iii) Explain how the half-life of 1-131 can be calculated using the slope of the line plotted on the graph.

(e) Compare the value of the half-life of 1-131 at 25°C to its value at 50°C,

END OF EXAMINATIONCopyright © 2003 by College Entrance Examination Board All rights reserved.

Asailable to AP professionals at apcentralcollegeboardcom and tostudents and parents at wwwcollegeboard.com/apstudents.

0

— >-.

:

I 6,000

14.000

12,000

10,000

8000

6,000

4,000I (Inn

14

Handout Page 45 of 182 Handout Page 45 of 182

Handout Page 45 of 182 Updated 4/9/2014 Handout Page 45 of 182

Page 46: V1) I - Blue Valley Schools Chemistry/AP... · 3DJH RI 3DJH RI 2001 AP® CHEMISTRY FREE-RESPONSEQUESTIONS Answer EITHER Question 2 below OR Question 3 printed on page 8

2004 AP® CHEMISTRY FREE-RESPONSE QUESTIONS

CHEMISTRYSection II

(Total time—90 minutes)

Part A

Time—40 minutesYOU MAY USE YOUR CALCULATOR FOR PART A.

CLEARLY SHOW THE METHOD USED AND THE STEPS INVOLVED IN ARRIVING AT YOUR ANSWERS.It is to your advantage to do this, since you may obtain partial credit if you do and you will receive little or no credit ifyou do not. Attention should be paid to significant figures.

Be sure to write all your answers to the questions on the lined pages following each question in the booklet with thepink cover. Do NOT write your answers on the green insert.

Answer Question 1 below. The Section II score weighting for this question is 20 percent.

1. Answer the following questions relating to the solubilities of two silver compounds, Ag2CrO4 and Ag3PO4.

Silver chromate dissociates in water according to the equation shown below.

Ag2CrO4(s) 2 Ag(aq) + CrO42(aq) = 2.6 x l02 at 25°C

(a) Write the equilibrium-constant expression for the dissolving of Ag2CrO4(s).

(b) Calculate the concentration, in mol L’. of Ag(aq) in a saturated solution of Ag2CrO4 at 25°C.

(c) Calculate the maximum mass, in grams, of Ag,Cr03 that can dissolve in 100. mL of water at 25°C.

(d) A 0.100 mol sample of solid AgNO3 is added to a 1.00 L saturated solution of Ag2CrO4.Assumingno volume change, does [Cr042jincrease, decrease, or remain the same? Justify your answer.

In a saturated solution of Ag3PO3 at 25°C. the concentration of Ag(aq) is 5.3 x l0 M. The equilibrium-constant expression for the dissolving of Ag.PO4(s) in water is shown below.

Kcp = [Ag]3[P043J

(e) Write the balanced equation for the dissolving of Ag3PO4 in water.

f) Calculate the value of K,, for AgPO3 at 25°C.

(g) A 1.00 L sample of saturated Ag3PO4 solution is allowed to evaporate at 25°C to a final volumeof 500. mL. What is [Ag} in the solution? Justify your answer.

Copyright © 2004 by College Entrance Examination Board. All rights reserved.Visit apcentralcollegeboardcom (for AP professionals) and wwwcollegeboardcom/apstudents (for AP students and parents).

GO ON TO THE NEXT PAGE.6

Handout Page 46 of 182 Handout Page 46 of 182

Handout Page 46 of 182 Updated 4/9/2014 Handout Page 46 of 182

Page 47: V1) I - Blue Valley Schools Chemistry/AP... · 3DJH RI 3DJH RI 2001 AP® CHEMISTRY FREE-RESPONSEQUESTIONS Answer EITHER Question 2 below OR Question 3 printed on page 8

2004 AP® CHEMISTRY FREE-RESPONSE QUESTIONS

Answer EITHER Question 2 below OR Question 3 printed on page 8. Only one of these two questions will begraded. If you start both questions, be sure to cross out the question you do not want graded. The Section II scoreweighting for the question you choose is 20 percent.

2 Fe(s) + 02(g) — Fe203(s) LHJ = —824 kJ mo11

2. Iron reacts with oxygen to produce iron(ffl) oxide, as represented by the equation above. A 75.0 g sample ofFe(s) is mixed with 11.5 L of O,(g) at 2.66 atm and 298 K.

(a) Calculate the number of moles of each of the following before the reaction begins.

(i) Fe(s)

(ii) 02(g)

(b) Identify the limiting reactant when the mixture is heated to produce Fe203(s). Support your answer withcalculations.

(c) Calculate the number of moles of Fe203(s) produced when the reaction proceeds to completion.

(d) The standard free energy of formation. tG1 , of Fe103(s) is —740. kJ mol at 298 K.

(i) Calculate the standard entropy of formation, , of Fe203(s) at 298 K. Include units with your

answer.

(ii) Which is more responsible for the spontaneity of the formation reaction at 298 K, the standard enthalpyof formation. , or the standard entropy of formation, ? Justify your answer.

The reaction represented below also produces iron(III) oxide. The value of Ml for the reaction is —280. kJ per

mole of Fe203(s) formed.

2 FeO(s) + 07(g) - Fe,03(s)

(e) Calculate the standard enthaipy of formation, of FeO(s).

Copyright © 2004 by College Entrance Examination Board, All rights reserved.Visit apcentral.collegeboard.com (for AP professwnals) and www.collegeboard.com/apstudents (for AP students and parents).

GO ON TO THE NEXT PAGE.7

Handout Page 47 of 182 Handout Page 47 of 182

Handout Page 47 of 182 Updated 4/9/2014 Handout Page 47 of 182

Page 48: V1) I - Blue Valley Schools Chemistry/AP... · 3DJH RI 3DJH RI 2001 AP® CHEMISTRY FREE-RESPONSEQUESTIONS Answer EITHER Question 2 below OR Question 3 printed on page 8

2004 AP® CHEMISTRY FREE-RESPONSE QUESTIONS

3. The first-order decomposition of a colored chemical species, X, into colorless products is monitored with aspectrophotometer by measuring changes in absorbance over time. Species X has a molar absorptivity constantof 5.00 x cm M and the path length of the cuvette containing the reaction mixture is 1.00 cm. The datafrom the experiment are given in the table below.

[Xl TimeAbsorbance

iv, (mm)7 0.600 0.0

4.00x l0 0.200 35.03.OOx i0 0.150 44.2

l.50x 10 0.075 7

(a) Calculate the initial concentration of the colored species.

(b) Calculate the rate constant for the first-order reaction using the values given for concentration and time.Include units with your answer.

(c) Calculate the number of minutes it takes for the absorbance to drop from 0.600 to 0.075.

(d) Calculate the half-life of the reaction. Include units with your answer.

(e) Experiments were performed to determine the value of the rate constant for this reaction at varioustemperatures. Data from these experiments were used to produce the graph below, where T is temperature.This graph can be used to determine the activation energy, E0, of the reaction.

(i) Label the vertical axis of the graph.

(ii) Explain how to calculate the activation energy from this graph.

STOPIf you finish before time is called, you may check your work on this part only.

Do not turn to the other part of the test until you are told to do so.

Copyright © 2OfJ4 by College Entrance Examination Board. All rights reseied,Visit apcentral.collegeboard.com (for AP professionals) and www.collegeboard.com/apstudents (for AP ctudents and parents).

8

Handout Page 48 of 182 Handout Page 48 of 182

Handout Page 48 of 182 Updated 4/9/2014 Handout Page 48 of 182

Page 49: V1) I - Blue Valley Schools Chemistry/AP... · 3DJH RI 3DJH RI 2001 AP® CHEMISTRY FREE-RESPONSEQUESTIONS Answer EITHER Question 2 below OR Question 3 printed on page 8

2004 AP® CHEMISTRY FREE-RESPONSE QUESTIONS

CHEMISTRYPart B

Time—SO minutes

NO CALCULATORS MAY BE USED FOR PART B.

Answer Question 4 below. The Section II score weighting for this question is 15 percent.

4. Write the formulas to show the reactants and the products for any FIVE of the laboratory situations describedbelow. Answers to more than five choices will not be graded. In all cases, a reaction occurs. Assume thatsolutions are aqueous unless otherwise indicated. Represent substances in solution as ions if the substances areextensively ionized. Omit formulas for any ions or molecules that are unchanged by the reaction. You need notbalance the equations.

Example: A strip of magnesium is added to a solution of silver nitrate.

£. + +

(a) A solution of copper(II) sulfate is spilled onto a sheet of freshly polished aluminum metal.

(b) Dimethyl ether is burned in air.

(C) A 0.1 M nitrous acid solution is added to the same volume of a 0.1 M sodium hydroxide solution.

(d) Hydrogen iodide gas is bubbled into a solution of lithium carbonate.

(e) An acidic solution of potassium dichromate is added to a solution of iron(U) nitrate.

(0 Excess concentrated aqueous ammonia is added to a solution of nickel(1I) bromide.

(g) A solution of sodium phosphate is added to a solution of aluminum nitrate.

(h) Concentrated hydrochloric acid is added to a solution of sodium sulfide.

Copydght © 2fK)4 b College Entrance Examination Board. All dghts reserved.Visit apcentral.collegeboard,com (for AP professionals) and www.collegeboard.com/apstudents (for AP students and parents).

GO ON TO THE NEXT PAGE.9

Handout Page 49 of 182 Handout Page 49 of 182

Handout Page 49 of 182 Updated 4/9/2014 Handout Page 49 of 182

Page 50: V1) I - Blue Valley Schools Chemistry/AP... · 3DJH RI 3DJH RI 2001 AP® CHEMISTRY FREE-RESPONSEQUESTIONS Answer EITHER Question 2 below OR Question 3 printed on page 8

2004 AP® CHEMISTRY FREE-RESPONSE QUESTIONS

Your responses to the rest of the questions in this part of the examination will be graded on the basis of the accuracyand relevance of the information cited. Explanations should be clear and well organized. Examples and equationsmay be included in your responses where appropriate. Specific answers are preferable to broad, diffuse responses.

Answer BOTH Question 5 below AND Question 6 printed on page 11. Both of these questions will be graded. TheSection II score weighting for these questions is 30 percent (15 percent each).

5. In a laboratory class, a student is given three flasks that are labeled Q, R, and S. Each flask contains one of thefollowing solutions: 1.0 M Pb(N03)2,1.0 M NaCl * or 1.0 M K,C03.The student is also given two flasks thatare labeled X and Y. One of these flasks contains 1.0 M AgNO3,and the other contains 1.0 M BaCI2.Thisinformation is summarized in the diagram below.

Each flask Each flaskcmsoof {K2C01

(a) When the student combined a sample of solution Q with a sample of solution X, a precipitate formed. Aprecipitate also formed when samples of solutions Q and Y were combined.

(i) Identify solution Q.(ii) Write the chemical formulas for each of the two precipitates.

(b) When solution Q is mixed with solution R, a precipitate forms. However, no precipitate forms whensolution Q is mixed with solution S.

(i) Identify solution R and solution S.

(ii) Write the chemical formula of the precipitate that forms when solution Q is mixed with solution R,

(c) The identity of solution X and solution Y are to be determined using only the following solutions: l .0 MPb(N03)2,1.0 M NaCl, and 1.0 M K2C03,

(i) Describe a procedure to identify solution X and solution Y.

(ii) Describe the observations that would allow you to distinguish between solution X and solution Y.

(iii) Explain how the observations would enable you to distinguish between solution X and solution Y.

Copyright © 2004 by College Entrance Examination Board. All rights reserved,Visit apcentralcollegeboardcom (for AP professionals) and www.collegeboard.com/apstudents (for AP students and parents).

GO ON TO THE NEXT PAGE.10

Handout Page 50 of 182 Handout Page 50 of 182

Handout Page 50 of 182 Updated 4/9/2014 Handout Page 50 of 182

Page 51: V1) I - Blue Valley Schools Chemistry/AP... · 3DJH RI 3DJH RI 2001 AP® CHEMISTRY FREE-RESPONSEQUESTIONS Answer EITHER Question 2 below OR Question 3 printed on page 8

2004 AP® CHEMISTRY FREE-RESPONSE QUESTIONS

6. An electrochemical cell is constructed with an open switch, as shown in the diagram above. A strip of Sn and astrip of an unknown metal. X, are used as electrodes. When the switch is closed, the mass of the Sn electrodeincreases. The half-reactions are shown below.

Sn2(aq) + 2 e —* Sn(s)

X3(aq) + 3 e —* X(s)

= —o 14 V

=

(a) In the diagram above, label the electrode that is the cathode. Justify your answer.

(b) In the diagram above, draw an arrow indicating the direction of the electron flow in the external circuit whenthe switch is closed.

(c) If the standard cell potential, Eceit. is +0.60 V. what is the standard reduction potential, in volts, for theX3÷IX electrode?

(d) Identify metal X.

(e) Write a balanced net-ionic equation for the overall chemical reaction occurring in the cell.

(f) In the cell, the concentration of Sn2 is changed from 1.0 Al to 0.50 Al, and the concentration of X3 ischanged from 1.0 Al to 0.10 Al.

(i) Substitute all the appropriate values for determining the cell potential, into the Nemst equation.(Do do any calculations.)

(ii) On the basis of your response in part (f (i), will the cell potential. E11. be greater than, less than, or

equal to the original E11 ? Justify your answer.

Copyright © 2004 by College Entrance Examination Board, All rights resered.Visit apcentral.collegeboardcom (for AP professionals) and www.collegeboard.com/apstudents (for AP students and parents).

GO ON TO THE NEXT PAGE.

Sn x

1,0 Al Sn(N03)2 1,0 Al X(N01)3

11

Handout Page 51 of 182 Handout Page 51 of 182

Handout Page 51 of 182 Updated 4/9/2014 Handout Page 51 of 182

Page 52: V1) I - Blue Valley Schools Chemistry/AP... · 3DJH RI 3DJH RI 2001 AP® CHEMISTRY FREE-RESPONSEQUESTIONS Answer EITHER Question 2 below OR Question 3 printed on page 8

2004 AP® CHEMISTRY FREE-RESPONSE QUESTIONS

Answer EITHER Question 7 below OR Question 8 printed on page 13. Only one of these two questions will begraded. If you start both questions, be sure to cross out the question you do not want graded. The Section II scoreweighting for the question you choose is 15 percent.

7. Use appropriate chemical principles to account for each of the following observations. In each part, yourresponse include specific information about bth substances.

(a) At 25°C and I atm, F, is a gas, whereas 12 is a solid.

(b) The melting point of NaF is 993°C, whereas the melting point of CsCI is 645°C.

(c) The shape of the ICl3 ion is square planar, whereas the shape of the BF4 ion is tetrahedral.

(d) Ammonia, NH3. is very soluble in water, whereas phosphine, PH3, is only moderately soluble in water.

Copyright © 2004 by College Entrance Examination Board. All rights reserved.Visit apcentral.collegehoard.com (for AP professionals) and www.collegehoard.com/apstudents (for AP students and parentsi.

GO ON TO THE NEXT PAGE.12

Handout Page 52 of 182 Handout Page 52 of 182

Handout Page 52 of 182 Updated 4/9/2014 Handout Page 52 of 182

Page 53: V1) I - Blue Valley Schools Chemistry/AP... · 3DJH RI 3DJH RI 2001 AP® CHEMISTRY FREE-RESPONSEQUESTIONS Answer EITHER Question 2 below OR Question 3 printed on page 8

2004 AP® CHEMISTRY FREE-RESPONSE QUESTIONS

8. Answer the following questions about carbon monoxide, CO(g). and carbon dioxide. CO,(g). Assume that bothgases exhibit ideal behavior.

(a) Draw the complete Lewis structure (electron-dot diagram) for the CO molecule and for the CO2 molecule.

(b) Identify the shape of the CO2 molecule.

(c) One of the two gases dissolves readily in water to form a solution with a pH below 7. Identify the gas andaccount for this observation by writing a chemical equation.

(d) A 1.0 mole sample of CO(g) is heated at constant pressure. On the graph below, sketch the expected plot ofvolume versus temperature as the gas is heated.

I (kelv ins)

(e) Samples of CO(g) and C02(g) are placed in 1 L containers at the conditions indicated in the diagrambelow.

(i) Indicate whether the average kinetic energy of the CO7(g) molecules is greater than, equal to, or lessthan the average kinetic energy of the CO(g) molecules. Justify your answer.

(ii) Indicate whether the root-mean-square speed of the CO-(g) molecules is greater than, equal to, or lessthan the root-mean square speed of the CO(g) molecules. Justify your answer.

(iii) Indicate whether the number of CO2(g) molecules is greater than, equal to, or less than the number ofCO(g) molecules. Justify your answer.

END OF EXAMINATION

Copyright © 1(X)4 by College Entrance Examination Board All rights reserved.Visit apcentral.collegehoardcom cfor AP professionals) and wwwcollegeboardcomJapstudems d’or AP students and parents).

2 atm I atm25°C 25C

13

Handout Page 53 of 182 Handout Page 53 of 182

Handout Page 53 of 182 Updated 4/9/2014 Handout Page 53 of 182

Page 54: V1) I - Blue Valley Schools Chemistry/AP... · 3DJH RI 3DJH RI 2001 AP® CHEMISTRY FREE-RESPONSEQUESTIONS Answer EITHER Question 2 below OR Question 3 printed on page 8

2004 AP® CHEMISTRY FREE-RESPONSE QUESTIONS (Form B)

CHEMISTRY

Section II(Total time—90 minutes)

Part ATime—40 minutes

YOU MAY USE YOUR CALCULATOR FOR PART A.

CLEARLY SHOW THE METHOD USED AND THE STEPS INVOLVED IN ARRIVING AT YOUR ANSWERS.It is to your advantage to do this, since you may obtain partial credit if you do and you will receive little or no credit ifyou do not. Attention should be paid to significant figures.

Be sure to write all your answers to the questions on the lined pages following each question in the booklet with thegoldenrod cover. Do NOT write your answers on the lavender insert.

Answer Question I below, The Section II score weighting for this question is 20 percent.

N,(g) + 3 H2(g) 2 NH3(g)

I. For the reaction represented above, the value of the equilibrium constant, K, is 3.1 x 10 ‘ at 700. K.

(a) Write the expression for the equilibrium constant, K, for the reaction.

(b) Assume that the initial partial pressures of the gases are as follows:

PN7 = 0.411 atm. = 0.903 atm, and PNH3 = 0.224 atm.

i) Calculate the value of the reaction quotient, Q. at these initial conditions.

(ii) Predict the direction in which the reaction will proceed at 700. K if the initial partial pressures arethose given above. Justify your answer.

(c) Calculate the value of the equilibrium constant, K, given that the value of K for the reaction at 700. K is3.1 x

(d) The value of K,, for the reaction represented below is 8.3 x iO at 700. K.

NH(g) + HS(g) NH4HS(g)

Calculate the value of K,, at 700. K for each of the reactions represented below.

(i) NH4HS(g) ± NH(g) + H2S(g)

Il) 2 HS(g) + ) + 3 H(t,’) 2 ‘H3HS()

Copyright 0 2004 by College Entrance Examination Board. All rights reserved.Visit apcentralcol1egeboardcom (for AP professionals) and wwwcollegeboard.com/apstudents (for AP students and parents).

GO ON TO THE NEXT PAGE.6

Handout Page 54 of 182 Handout Page 54 of 182

Handout Page 54 of 182 Updated 4/9/2014 Handout Page 54 of 182

Page 55: V1) I - Blue Valley Schools Chemistry/AP... · 3DJH RI 3DJH RI 2001 AP® CHEMISTRY FREE-RESPONSEQUESTIONS Answer EITHER Question 2 below OR Question 3 printed on page 8

2004 AP® CHEMISTRY FREE-RESPONSE QUESTIONS (Form B)

Answer EITHER Question 2 below OR Question 3 printed on page 8. Only one of these two questions will begraded. If you start both questions, be sure to cross out the question you do not want graded. The Section II scoreweighting for the question you choose is 20 percent.

2. Answer the following questions related to hydrocarbons.

(a) Determine the empirical formula of a hydrocarbon that contains 85.7 percent carbon by mass.

(b) The density of the hydrocarbon in part (a) is 2.0 g L at 50°C and 0.948 atm.

(i) Calculate the molar mass of the hydrocarbon.

iii) Determine the molecular formula of the hydrocarbon.

(c) Two flasks are connected by a stopcock as shown below. The 5.0 L flask contains CH4 at a pressure of 3.0atm. and the I .0 L flask contains C,H6 at a pressure of 0.55 atm. Calculate the total pressure of the systemafter the stopcock is opened. Assume that the temperature remains constant.

l.() L0.55 atm

(d) Octane, C8H18(1), has a density of 0.703 g mL at 20°C. A 255 mL sample of C8H18(t) measured at 20°Creacts completely with excess oxygen as represented by the equation below.

2C5H18(1) + 25 O2(g) —> 16 CO,(g) + 18 H-,O(g)

Calculate the total number of moles of gaseous products formed.

CopYright © OO4 by College Entrance Examination Board. All rights reserved.Visit apcentral.collegeboardcom for AP professionals) and wwwcoliegeboard.com!apstudents (for AP students and parents).

GO ON TO THE NEXT PAGE.

5.OL3.Oatm

7

Handout Page 55 of 182 Handout Page 55 of 182

Handout Page 55 of 182 Updated 4/9/2014 Handout Page 55 of 182

Page 56: V1) I - Blue Valley Schools Chemistry/AP... · 3DJH RI 3DJH RI 2001 AP® CHEMISTRY FREE-RESPONSEQUESTIONS Answer EITHER Question 2 below OR Question 3 printed on page 8

2004 AP® CHEMISTRY FREE-RESPONSE QUESTIONS (Form B)

2 H,02(q) —* 2 1-120(1) + 02(g)

3. Hydrogen peroxide decomposes according to the equation above.

(a) An aqueous solution of H202 that is 6.00 percent H,02 by mass has a density of 1.03 g mL’. Calculateeach of the following.

(i) The original number of moles of FLO2 in a 125 mL sample of the 6.00 percent 1-1,02 solution

(ii) The number of moles of 0,(g) that are produced when all of the 1-1202 in the 125 mL sampledecomposes

(b) The graphs below show results from a study of the decomposition of H,02.

- -1.00

Write the rate law for the reaction. Justify your answer.

Determine the halflife of the reaction.

Calculate the value of the rate constant. k. Include appropriate units in your answer.

Determine [H,011after 2.00() minutes elapse from the time the reaction began.

1.00

0.80

0.60

0.40

0 800 1.600 2.400 3.200

Time (mm)

(i )

(ii)

(111)

(iv)

0 800 1,600 2,3(X) 3.200

Time (mm)

Cops right ) 2004 by College Entrance Examination Board. All rights resered.Visit apcentralcollegeboard.com tor AP professionals) and wwwcollegeboardcom/apstudents (for AP students and parents).

8

Handout Page 56 of 182 Handout Page 56 of 182

Handout Page 56 of 182 Updated 4/9/2014 Handout Page 56 of 182

Page 57: V1) I - Blue Valley Schools Chemistry/AP... · 3DJH RI 3DJH RI 2001 AP® CHEMISTRY FREE-RESPONSEQUESTIONS Answer EITHER Question 2 below OR Question 3 printed on page 8

2004 AP® CHEMISTRY FREE-RESPONSE QUESTIONS (Form B)

CHEIiIISTRY

Part B

Time—50 minutesNO CALCULATORS MAY BE USED FOR PART B.

Answer Question 4 below. The Section II score weighting for this question is 15 percent.

4. Write the formulas to show the reactants and the products for any FIVE of the laboratory situations describedbelow. Answers to more than tive choices will not be graded. In all cases, a reaction occurs. Assume thatsolutions are aqueous unless otherwise indicated. Represent substances in solution as ions if the substances areextensively ionized. Omit formulas for any ions or molecules that are unchanged by the reaction. You need notbalance the equations.

Example: A strip of magnesium is added to a solution of silver nitrate.

.

+ A + A

(a) Cadmium metal is placed in a solution of tin(ll) chloride.

(b) Magnesium pellets are placed in 1.0 M hydrochloric acid.

(c) Sulfur in its standard state is burned in air.

(d) Solutions of silver nitrate and sodium chloride are combined.

(e) Solid iron(H) sulfite is heated strongly.

(f) Powdered barium oxide is mixed with water.

(g) Excess saturated sodium fluoride solution is added to a solution of aluminum sulfate.

(h) Solid potassium carbonate is added to 1.0 M sulfuric acid.

Copright 2003 by College Entrance Examination Board All rights reserved.Visit apcentral.collegeboard.com (for A? professionals and www.collegehoard.com/apstudents (for AP students and parents).

GO ON TO THE NEXT PAGE.9

Handout Page 57 of 182 Handout Page 57 of 182

Handout Page 57 of 182 Updated 4/9/2014 Handout Page 57 of 182

Page 58: V1) I - Blue Valley Schools Chemistry/AP... · 3DJH RI 3DJH RI 2001 AP® CHEMISTRY FREE-RESPONSEQUESTIONS Answer EITHER Question 2 below OR Question 3 printed on page 8

2004 AP® CHEMISTRY FREE-RESPONSE QUESTIONS (Form B)

Your responses to the rest of the questions in this part of the examination will be graded on the basis of the accuracyand relevance of the information cited. Explanations should be clear and well organized. Examples and equationsmay be included in your responses where appropriate. Specific answers are preferable to broad, diffuse responses.

Answer BOTH Question 5 below AND Question 6 printed on page 11. Both of these questions will be graded. TheSection II score weighting for these questions is 30 percent (15 percent each).

5. An experiment is performed to determine the molar mass of an unknown solid monoprotic acid, HA, by titrationwith a standardized NaOH solution.

(a) What measurement(s) must be made to determine the number of moles of NaOH used in the titration?

(b) Write a mathematical expression that can be used to determine the number of moles of NaOH used to reachthe endpoint of the titration.

(c) How can the number of moles of HA consumed in the titration be determined?

(d) In addition to the measurement(s) made in part (a), what other measurement(s) must be made to determinethe molar mass of the acid, HA?

(e) Write the mathematical expression that is used to determine the molar mass of HA.

(f The following diagram represents the setup for the titration. In the appropriate boxes below, list thechemical(s) needed to perform the titration.

Chemical(s) Needed in Buret

hemicai Needed in Flask

(g) Explain what effect each of the following would have on the calculated molar mass of HA. Justify youranswers.

i) The original solid acid. HA. was not completely dry at the beginning of the experiment.

(ii) The procedure called for 25 mL of H20 in the Erlenmeyer flask, but a student used 35 mL of H)O.Copyright © 2004 by College Entrance Examination Board, All rights reserved.

Visit apcentralcollegeboard.com (for AP professionals) and www.collegeboard.com/apstudents (for AP students and parents).

GO ON TO THE NEXT PAGE.10

Handout Page 58 of 182 Handout Page 58 of 182

Handout Page 58 of 182 Updated 4/9/2014 Handout Page 58 of 182

Page 59: V1) I - Blue Valley Schools Chemistry/AP... · 3DJH RI 3DJH RI 2001 AP® CHEMISTRY FREE-RESPONSEQUESTIONS Answer EITHER Question 2 below OR Question 3 printed on page 8

2004 AP® CHEMISTRY FREE-RESPONSE QUESTIONS (Form B)

Voltmeter

Ag

6. The following questions refer to the electrochemical cell shown in the diagram above.

(a) Write a balanced net ionic equation for the spontaneous reaction that takes place in the cell.

(b) Calculate the standard cell potential, E°, for the reaction in part (a).

(c) In the diagram above,

(i) label the anode and the cathode on the dotted lines provided, and

(ii) indicate, in the boxes below the half-cells, the concentration of AgNO3 and the concentration ofZn(N03), that are needed to generate E°.

(d) How will the cell potential be affected if KI is added to the silver half-cell? Justify your answer.

Copyright @ 2004 by College Entrance Examination Board. All rights reserved,Visit apcentralcollegeboardcom (for AP professionals> and www.collegeboard.com/apstudents (for AP students and parents).

GO ON TO THE NEXT PAGE.

Wire

MAgNO1 MZn(NO,

11

Handout Page 59 of 182 Handout Page 59 of 182

Handout Page 59 of 182 Updated 4/9/2014 Handout Page 59 of 182

Page 60: V1) I - Blue Valley Schools Chemistry/AP... · 3DJH RI 3DJH RI 2001 AP® CHEMISTRY FREE-RESPONSEQUESTIONS Answer EITHER Question 2 below OR Question 3 printed on page 8

2004 AP® CHEMISTRY FREE-RESPONSE QUESTIONS (Form B)

Answer EITHER Question 7 below OR Question 8 printed on page 13. Only one of these two questions will begraded. If you start both questions, be sure to cross out the question you do not want graded, The Section II scoreweighting for the question you choose is 15 percent.

N,(g) + 2 H2(g) N,H4(g) AH98 = +95.4 kJ mol’; AS98 = —176 J K mo1

7. Answer the following questions about the reaction represented above using principles of thermodynamics.

(a) On the basis of the thermodynamic data given above, compare the sum of the bond strengths of the reactantsto the sum of the bond strengths of the product. Justify your answer.

(b) Does the entropy change of the reaction favor the reactants or the product? Justify your answer.

(C) For the reaction under the conditions specified, which is favored, the reactants or the product? Justify youranswer.

(d) Explain how to determine the value of the equilibrium constant, Keq, for the reaction. (Do j do anycalculations.)

(e) Predict whether the value of Keq for the reaction is greater than 1, equal to 1, or less than 1. Justify youranswer.

Copyright © 2004 by College Entrance Examination Board. All rights reserved.Visit apcentral.collegehoard.com for AP professionals) and ww.coIlegehoard.comJapstudents for AP students and parents.

GO ON TO THE NEXT PAGE.12

Handout Page 60 of 182 Handout Page 60 of 182

Handout Page 60 of 182 Updated 4/9/2014 Handout Page 60 of 182

Page 61: V1) I - Blue Valley Schools Chemistry/AP... · 3DJH RI 3DJH RI 2001 AP® CHEMISTRY FREE-RESPONSEQUESTIONS Answer EITHER Question 2 below OR Question 3 printed on page 8

2004 AP® CHEMISTRY FREE-RESPONSE QUESTIONS (Form B)

CH3 CH1 CH3 H\ / _\ /C=C C=C/ / \

H H H Cl-I3

cis-2-butene trans-2-butene

8. The gas-phase conversion reaction between the geometric isomers cis-2-butene and trans-2-butene is representedby the equation above. The value of the equilibrium constant, Keq , for the reaction is 3.2 at 298 K and 1.0 atm.

(a) In a mixture of the isomers at equilibrium at 298 K and 1.0 atm, which is present at a higher concentration,cis-2-butene or trans-2-butene? Justify your answer.

(b) If 1.00 mol of pure cis-2-butene and 1.0 mol of pure trans-2-butene were introduced into an evacuatedcontainer at 298 K, in which direction (to the right or to the left) would the reaction proceed to establishequilibrium? Justify your answer.

(c) Given that K1 for the reaction at 400 K has the value 1.3, predict whether the reaction is endothermic orexothermic. Justify your answer.

(d) There are other structural isomers of cis-2-butene and tran.r-2-butene. Draw one of these isomers, includingall atoms, and give its IUPAC name.

END OF EXAMINATION

Copyright © 2004 by College Entrance Examination Board, All rights reserved.Visit apcentralcolleeboard.com (for AP professionals) and wssw.collegeboard.com/apstudents (tor AP students and parents).

13

Handout Page 61 of 182 Handout Page 61 of 182

Handout Page 61 of 182 Updated 4/9/2014 Handout Page 61 of 182

Page 62: V1) I - Blue Valley Schools Chemistry/AP... · 3DJH RI 3DJH RI 2001 AP® CHEMISTRY FREE-RESPONSEQUESTIONS Answer EITHER Question 2 below OR Question 3 printed on page 8

2005 AP® CHEMISTRY FREE-RESPONSE QUESTIONS

CHEMISTRYSection II

(Total time—90 minutes)

Part A

Tinze—40 minutesYOU MAY USE YOUR CALCULATOR FOR PART A.

CLEARLY SHOW THE METHOD USED AND THE STEPS INVOLVED IN ARRIVING AT YOUR ANSWERS.It is to your advantage to do this, since you may obtain partial credit if you do and you will receive little or no credit ifyou do not. Attention should be paid to significant figures.

Be sure to write all your answers to the questions on the lined pages following each question in the booklet with thepink cover. Do NOT write your answers on the green insert.

Answer Question I below. The Section II score weighting for this question is 20 percent.

HC3H5O2(aq) C3H50, (aq) + H(aq) K0 = 1.34 x 10

I. Propanoic acid. HC3H5O,, ionizes in water according to the equation above.

(a) Write the equilibrium-constant expression for the reaction.

(b) Calculate the pH of a 0.265 M solution of propanoic acid.

(c) A 0.496 g sample of sodium propanoate, NaC3H5O7,is added to a 50.0 mL sample of a 0.265 M solutionof propanoic acid. Assuming that no change in the volume of the solution occurs, calculate each of thefollowing.

(i) The concentration of the propanoate ion,C3H50, (aq), in the solution

(ii) The concentration of the H(aq) ion in the solution

The methanoate ion. HCO(aq), reacts with water to form methanoic acid and hydroxide ion, as shown in thefollowing equation.

HCO,(aq) ÷ 1120(1) HCO,H(aq) + OW(aq)

(d) Given that [OW] is 4.18 x l0 6 M in a 0.309 Al solution of sodium methanoate. calculate each of thefollowing.

(i) The value of K6 for the methanoate ion, HCO,(aq)

(ii) The value of K0 for methanoic acid. HCO,H

(e) Which acid is stronger, propanoic acid or methanoic acid? Justify your answer.

Copyright 2005 by College Entrance Examination Board. .Al1 rights reserved.Visit apcentral.collegeboard.com (for AP professionals) and www.collegeboard.com/apstudents (for AP students and parents).

GO ON TO THE NEXT PAGE.6

Handout Page 62 of 182 Handout Page 62 of 182

Handout Page 62 of 182 Updated 4/9/2014 Handout Page 62 of 182

Page 63: V1) I - Blue Valley Schools Chemistry/AP... · 3DJH RI 3DJH RI 2001 AP® CHEMISTRY FREE-RESPONSEQUESTIONS Answer EITHER Question 2 below OR Question 3 printed on page 8

2005 AP® CHEMISTRY FREE-RESPONSE QUESTIONS

Answer EITHER Question 2 below OR Question 3 printed on page 8-9. Only one of these two questions will begraded. If you start both questions. be sure to cross out the question you do not want graded. The Section II scoreweighting for the question you choose is 20 percent.

2. Answer the following questions about a pure compound that contains only carbon, hydrogen, and oxygen.

(a) A 0.7549 g sample of the compound bums in O(g) to produce 1.9061 g of C02(g) and0.3370 g of H2O(g).

(i) Calculate the individual masses of C, H, and 0 in the 0,7549 g sample.

(ii) Determine the empirical formula for the compound.

(b) A 0.5246 g sample of the compound was dissolved in 10.0012 g of lauric acid, and it was determined thatthe freezing point of the lauric acid was lowered by 1.68°C. The value of of lauric acid is 3.90°C m*Assume that the compound does not dissociate in lauric acid.

(i) Calculate the molality of the compound dissolved in the lauric acid.

(ii) Calculate the molar mass of the compound from the information provided.

(c) Without doing any calculations, explain how to determine the molecular formula of the compound based onthe answers to parts (a)(ii) and (b)(ii).

(d) Further tests indicate that a 0.10 M aqueous solution of the compound has a pH of 2.6. Identify the organicfunctional group that accounts for this pH.

Copyright • 201)5 by College Entrance Examination Board. All rights reserved.Visit apcentral.collegeboard.com Jor AP professionals) and www.collegeboard.com/apstudents (tor AP students and parentsL

GO ON TO THE NEXT PAGE.7

Handout Page 63 of 182 Handout Page 63 of 182

Handout Page 63 of 182 Updated 4/9/2014 Handout Page 63 of 182

Page 64: V1) I - Blue Valley Schools Chemistry/AP... · 3DJH RI 3DJH RI 2001 AP® CHEMISTRY FREE-RESPONSEQUESTIONS Answer EITHER Question 2 below OR Question 3 printed on page 8

2005 AP CHEMISTRY FREE-RESPONSE QUESTIONS

3. Answer the following questions related to the kinetics of chemical reactions.

L(aq) + C10(aq) OH 10(aq) + Cl(aq)

Iodide ion, I , is oxidized to hypoiodite ion, 10-, by hypochiorite, Cl0, in basic solution according to theequation above. Three initial-rate experiments were conducted; the results are shown in the following table.

ICIOI lnitialRateofExperiment Formation ot 10

( mol L (mol L )(molL s

1 0.017 0.015 0.156

2 0.052 0.015 0.476

3 0.016 0.061 0.596

(a) Determine the order of the reaction with respect to each reactant listed below. Show your work.

(i) I (aq)

(ii) ClO(aq)

(b) For the reaction,

(i) write the rate law that is consistent with the calculations in part (a);

(ii) calculate the value of the specific rate constant, k, and specify units.

The catalyzed decomposition of hydrogen peroxide, H02(aq). is represented by the following equation.

catalyst2H202(aq) 2 H-0(h + 01(g)

The kinetics of the decomposition reaction were studied and the analysis of the results show that it is afirst-order reaction. Some of the experimental data are shown in the table be1ow.

IFI)21 Time

(fliul L. ) t

I .0()

0.7 5.0

0,6! 10.0

Copyright 2OO by College Entrance Examination Board. All rights reserved.Visit apcentral.collegeboard.com for AP professionals) and www.collegeboard.com/apstudents (for AP students and parents).

GO ON TO THE NEXT PAGE.B

Handout Page 64 of 182 Handout Page 64 of 182

Handout Page 64 of 182 Updated 4/9/2014 Handout Page 64 of 182

Page 65: V1) I - Blue Valley Schools Chemistry/AP... · 3DJH RI 3DJH RI 2001 AP® CHEMISTRY FREE-RESPONSEQUESTIONS Answer EITHER Question 2 below OR Question 3 printed on page 8

2005 AP® CHEMISTRY FREE-RESPONSE QUESTIONS

(c) During the analysis of the data, the graph below was produced.

NTime (minutes)

(i) Label the vertical axis of the graph.

(ii) What are the units of the rate constant. k, for the decomposition of l-{,02(aq)?

(iii) On the graph, draw the line that represents the plot of the uncatalyzed first-order decompositionof 1.00 M H-,07(aq).

STOPIf you finish before time is called, you may check your work on this part only.

Do not turn to the other part of the test until you are told to do so.

Copyright OO5 by College Entrance Examination Board. All rights reserved.Visit apcentral.collegehoard.com for AP professionals) and www.eollegeboard.com/apstudents for AP students and parents).

9

Handout Page 65 of 182 Handout Page 65 of 182

Handout Page 65 of 182 Updated 4/9/2014 Handout Page 65 of 182

Page 66: V1) I - Blue Valley Schools Chemistry/AP... · 3DJH RI 3DJH RI 2001 AP® CHEMISTRY FREE-RESPONSEQUESTIONS Answer EITHER Question 2 below OR Question 3 printed on page 8

2005 AP CHEMISTRY FREE-RESPONSE QUESTIONS

CHEMISTRYPart B

Time—50 minutes

NO CALCULATORS MAY BE USED FOR PART B.

Answer Question 4 below. The Section II score weighting for this question is 15 percent.

4. Write the formulas to show the reactants and the products for any FIVE of the laboratory situations describedbelow. Answers to more than five choices will not be graded. In all cases, a reaction occurs. Assume thatsolutions are aqueous unless otherwise indicated. Represent substances in solution as ions if the substances areextensively ionized. Omit formulas for any ions or molecules that are unchanged by the reaction. You need notbalance the equations.

Example: A strip of magnesium is added to a solution of silver nitrate.

L +

(a) A strip of zinc is placed in a solution of nickel(II) nitrate.

(b) Solid aluminum hydroxide is added to a concentrated solution of potassium hydroxide.

(c) Ethyne (acetylene) is burned in air.

(d) Solid calcium carbonate is added to a solution of ethanoic (acetic) acid.

(e) Lithium metal is strongly heated in nitrogen gas.

(f) Boron tritluoride gas is added to ammonia gas.

(g) Sulfur trioxide gas is bubbled into a solution of sodium hydroxide.

(h) Equal volumes of 0. 1 M solutions of lead(II) nitrate and magnesium iodide are combined.

Copyright © 2005 by College Entrance Examination Board. All rights reserved.Visit apcentraLcollegeboardcom (for AP professionals) and wwwcollegeboardcom/apstudents (for AP students and parents).

GO ON TO THE NEXT PAGE.10

Handout Page 66 of 182 Handout Page 66 of 182

Handout Page 66 of 182 Updated 4/9/2014 Handout Page 66 of 182

Page 67: V1) I - Blue Valley Schools Chemistry/AP... · 3DJH RI 3DJH RI 2001 AP® CHEMISTRY FREE-RESPONSEQUESTIONS Answer EITHER Question 2 below OR Question 3 printed on page 8

2005 AP CHEMISTRY FREE-RESPONSE QUESTIONS

Your responses to the rest of the questions in this part of the examination will be graded on the basis of the accuracyand relevance of the information cited. Explanations should be clear and well organized. Examples and equationsmay be included in your responses where appropriate. Specific answers are preferable to broad, diffuse responses.

Answer BOTH Question 5 below AND Question 6 printed on page 12. Both of these questions will be graded. TheSection II score weighting for these questions is 30 percent (15 percent each).

5. Answer the following questions that relate to laboratory observations and procedures.

(a) An unknown gas is one of three possible gases: nitrogen, hydrogen, or oxygen. For each of the threepossibilities, describe the result expected when the gas is tested using a glowing splint (a wooden stick withone end that has been ignited and extinguished. but still contains hot, glowing, partially burned wood).

(b) The following three mixtures have been prepared: CaO plus water, Si02 plus water, and CO2 plus water.For each mixture, predict whether the pH is less than 7, equal to 7, or greater than 7. Justify your answers.

(c) Each of three beakers contains a 0.1 M solution of one of the following solutes: potassium chloride, silvernitrate, or sodium sulfide. The three beakers are labeled randomly as solution I, solution 2, and solution 3.Shown below is a partially completed table of observations made of the results of combining small amountsof different pairs of the solutions.

Solution I Solution 2 Solution 3

,4%>,,/zSolution I black precipitate

///

/

Solution 2/ // no reaction

,

//‘ /-

/Solution 3 ;II

(i) Write the chemical formula of the black precipitate.

(ii) Describe the expected results of mixing solution I with solution 3.

(iii) Identify each of the solutions 1, 2, and 3.

Copyright © 2005 by College Entrance Examination Board. All rights reserved.Visit apcentraEcollegeboard.com (for AP professionals) and wwwcollegeboard.com/apstudents (tor AP students and parents).

GO ON TO THE NEXT PAGE.11

Handout Page 67 of 182 Handout Page 67 of 182

Handout Page 67 of 182 Updated 4/9/2014 Handout Page 67 of 182

Page 68: V1) I - Blue Valley Schools Chemistry/AP... · 3DJH RI 3DJH RI 2001 AP® CHEMISTRY FREE-RESPONSEQUESTIONS Answer EITHER Question 2 below OR Question 3 printed on page 8

2005 AP CHEMISTRY FREE-RESPONSE QUESTIONS

6. Answer the following questions that relate to chemical bonding.

(a) In the boxes provided, draw the complete Lewis structure (electron-dot diagram) for each of the threemolecules represented below.

CF4 PF5 SF4

(b) On the basis of the Lewis structures drawn above, answer the following questions about the particularmolecule indicated.

(i) What is the F—C—F bond angle in CF4?

(ii) What is the hybridization of the valence orbitals of P in PF5 ?

(iii) What is the geometric shape formed by the atoms in SF4?

(c) Two Lewis structures can be drawn for the OPF1 molecule, as shown below.

II .. .. I:F—P—F:

Structure I Structure 2

(i) How many sigma bonds and how many pi bonds are in structure I ?

(ii) Which one of the two structures best represents a molecule of OPF3 ? Justify your answer in terms offormal charge.

Copyright © 2005 by College Entrance Ecamination Board. All rights reserved.Visit apcentral.collegeboard.eom (for AP professionals) and www.collegeboard.com/apstudents (for AP students and parents).

GO ON TO THE NEXT PAGE.12

Handout Page 68 of 182 Handout Page 68 of 182

Handout Page 68 of 182 Updated 4/9/2014 Handout Page 68 of 182

Page 69: V1) I - Blue Valley Schools Chemistry/AP... · 3DJH RI 3DJH RI 2001 AP® CHEMISTRY FREE-RESPONSEQUESTIONS Answer EITHER Question 2 below OR Question 3 printed on page 8

2005 AP® CHEMISTRY FREE-RESPONSE QUESTIONS

Answer EITHER Question 7 below OR Question 8 printed on page 14. Only one of these two questions will begraded. If you start both questions, be sure to cross out the question you do not want graded. The Section H scoreweighting for the question you choose is 15 percent.

7. Use principles of atomic structure, bonding, and/or intermolecular forces to respond to each of the following.Your responses include specific information about li substances referred to in each question.

(a) At a pressure of I atm, the boiling point of NH3(I) is 240 K, whereas the boiling point of NF3(1)is 144K.

(i) Identify the intermolecular force(s) in each substance.

(ii) Account for the difference in the boiling points of the substances.

(h) The melting point of KC1(s) is 776°C, whereas the melting point of NaCl(s) is 801°C.

(i) Identify the type of bonding in each substance.

(ii) Account for the difference in the melting points of the substances.

(C) As shown in the table below, the first ionization energies of Si, P. and Cl show a trend.

First Ionization EnergyElement

(kJ moI’)

Si 786

P 1,012

Cl 1,251

(i) For each of the three elements, identify the quantum level (e.g., n = I, n = 2, etc.) of the valenceelectrons in the atom.

(ii) Explain the reasons for the trend in first ionization energies.

(d) A certain element has two stable isotopes. The mass of one of the isotopes is 62.93 amu and the mass ofthe other isotope is 64.93 amu.

(i) Identify the element. Justify your answer.

(ii) Which isotope is more abundant? Justify your answer.

Copyright 2003 by College Entrance Examination Board. All rights reserved.\‘jsit apcentrahcollegeboard.com (for AP professionals) and www.col1egeboardcomJapstudents (for AP students and parents).

GO ON TO THE NEXT PAGE.13

Handout Page 69 of 182 Handout Page 69 of 182

Handout Page 69 of 182 Updated 4/9/2014 Handout Page 69 of 182

Page 70: V1) I - Blue Valley Schools Chemistry/AP... · 3DJH RI 3DJH RI 2001 AP® CHEMISTRY FREE-RESPONSEQUESTIONS Answer EITHER Question 2 below OR Question 3 printed on page 8

2005 AP® CHEMISTRY FREE-RESPONSE QUESTIONS

AgNO3(s) — Ag(aq) + N03(aq)

8. The dissolving of AgNO3(s) in pure water is represented by the equation above.

(a) Is AG for the dissolving of AgNO3(s) positive, negative, or zero? Justify your answer.

(b) Is AS for the dissolving of AgNO3(s) positive, negative, or zero? Justify your answer.

(c) The solubility of AgNO3(s) increases with increasing temperature.

(i) What is the sign of All for the dissolving process? Justify your answer.

(ii) Is the answer you gave in part (a) consistent with your answers to parts (b) and (c) (i) ? Explain.

The compound Nal dissolves in pure water according to the equation NaT(s) —* Na(aq) + r(aq) . Some ofthe information in the table of standard reduction potentials given below may be useful in answering thequestions that follow.

Half-reaction E° (V)02(g) + 4 H + 4 e — 2 H,O(1) 1.23

1,(s) + 2 e —* 2 r 0.53

2 H70(1) + 2 e H2(g) + 2 OH- —0.83

Na + e — Na(s) —2.71

(d) An electric current is applied to a 1 .0 M Na! solution.

(i) Write the balanced oxidation half-reaction for the reaction that takes place.

(ii) Write the balanced reduction half-reaction for the reaction that takes place.

(iii) Which reaction takes place at the anode, the oxidation reaction or the reduction reaction?

(iv) All electrolysis reactions have the same sign for AG°. IS the sign positive or negative? Justify youranswer.

END OF EXAM

Copyright 2005 by College Entrance Examination Board. All rights reserved.Visit apcentral.collegeboardcom (for AP professionals) and www.collegeboard.com/apstudents (for AP students and parents).

14

Handout Page 70 of 182 Handout Page 70 of 182

Handout Page 70 of 182 Updated 4/9/2014 Handout Page 70 of 182

Page 71: V1) I - Blue Valley Schools Chemistry/AP... · 3DJH RI 3DJH RI 2001 AP® CHEMISTRY FREE-RESPONSEQUESTIONS Answer EITHER Question 2 below OR Question 3 printed on page 8

2005 AP® CHEMISTRY FREE-RESPONSE QUESTIONS (Form B)

CHEMISTRY

Section II

(Total time—90 minutes)

Part A

Time—40 minutes

YOU MAY USE YOUR CALCULATOR FOR PART A.

CLEARLY SHOW THE METHOD USED AND THE STEPS INVOLVED IN ARRIVING AT YOUR ANSWERS.It is to your advantage to do this, since you may obtain partial credit if you do and you will receive little or no credit ifyou do not. Attention should be paid to significant figures.

Be sure to write all your answers to the questions on the lined pages following each question in this booklet.Do NOT write your answers on the lavender insert.

Answer Question I below. The Section II score weighting for this question is 20 percent.

K[H30j[oCrj

— 3 2 x l08- [HOCJI -

1. Hypochlorous acid, HOC!, is a weak acid in water. The K(1 expression for HOC1 is shown above.

(a) Write a chemical equation showing how HOCI behaves as an acid in water.

(b) Calculate the pH of a 0.175 M solution of 1-IOCI.

(c) Write the net ionic equation for the reaction between the weak acid HOCI(aq) and the strong baseNaOH(aq).

(d) In an experiment. 20.00 mL of 0.175 M HOCI(aq) is placed in a flask and titrated with 6.55 mL of0.435 M NaOH(aq).

(i Calculate the number of moles of NaOH(aq) added.

(ii) Calculate [H3OJ in the flask after the NaOH(aq) has been added.

(iii Calculate [OWl in the flask after the NaOH(aq) has been added.

Copyright © 2005 by College Entrance Examination Board, All rights reserved.Visit apcentral.collegeboardcom (for AP professionals) and www.collegeboard.com/apstudents (for AP students and parents).

GO ON TO THE NEXT PAGE.6

Handout Page 71 of 182 Handout Page 71 of 182

Handout Page 71 of 182 Updated 4/9/2014 Handout Page 71 of 182

Page 72: V1) I - Blue Valley Schools Chemistry/AP... · 3DJH RI 3DJH RI 2001 AP® CHEMISTRY FREE-RESPONSEQUESTIONS Answer EITHER Question 2 below OR Question 3 printed on page 8

2005 AP® CHEMISTRY FREE-RESPONSE QUESTIONS (Form B)

Answer EITHER Question 2 below OR Question 3 printed on pages 8 and 9. Only one of these two questions will begraded. If you start both questions, be sure to cross out the question you do not want graded. The Section II scoreweighting for the question you choose is 20 percent.

Collected O,(g)

Wire

2. Water was electrolyzed, as shown in the diagram above, for 5.61 minutes using a constant current of0.5 13 ampere. A small amount of nonreactive electrolyte was added to the container before the electrolysisbegan. The temperature was 298 K and the atmospheric pressure was 1.00 atm.

(a) Write the balanced equation for the half reaction that took place at the anode.

(b) Calculate the amount of electric charge, in coulombs, that passed through the solution.

(c) Why is the volume of O2(g) collected different from the volume of H(g) collected, as shown in thediagram?

(d) Calculate the number of moles of H,(g) produced during the electrolysis.

(e) Calculate the volume, in liters, at 298 K and 1.00 atm of dry H2(g) produced during the electrolysis.

(f) After the hydrolysis reaction was over, the vertical position of the tube containing the collected H2(g) wasadjusted until the water levels inside and outside the tube were the same. as shown in the diagram below.The volume of gas in the tube was measured under these conditions of 298 K and 1.00 atm, and its volumewas greater than the volume calculated in part (e). Explain.

Levels Arethe S me

Copyright © 2005 by College Entrance Examination Board .Al1 rights reserved,Visit apcentraLcollegeboardcom (for AP professionals) and wwwcollegeboardcomJapstudents (for AP students and parents).

GO ON TO THE NEXT PAGE.

Collected H,(g)

7

Handout Page 72 of 182 Handout Page 72 of 182

Handout Page 72 of 182 Updated 4/9/2014 Handout Page 72 of 182

Page 73: V1) I - Blue Valley Schools Chemistry/AP... · 3DJH RI 3DJH RI 2001 AP® CHEMISTRY FREE-RESPONSEQUESTIONS Answer EITHER Question 2 below OR Question 3 printed on page 8

2005 AP® CHEMISTRY FREE-RESPONSE QUESTIONS (Form B)

X —> 2Y + Z

3. The decomposition of gas X to produce gases Y and Z is represented by the equation above. In a certainexperiment, the reaction took place in a 5.00 L flask at 428 K. Data from this experiment were used to producethe information in the table below, which is plotted in the graphs that follow.

Time [XlXI [XJ’’

(minutes) (mol L’)n

(L mol )0 0.00633 —5.062 158

10. 0.00520 —5.259 19220. 0.00427 —5.456 23430. 0.00349 —5.658 28750. 0.00236 —6.049 42470. 0.00160 —6.438 625

100. 0.000900 —7.013 { 1,110

—5.0

—5.4

—.3. 4.,

-6.2

cS.

cJrr[ iTr -

ZE0. 0. 20. 30. 40. 50. 60, 70. 50. 90. 100.

Time (mm)

..I),::j:z600

— —c—i— —t- — — —

010. 20.30. 30. 50. 60. 70. SO. 90. 100).

‘fime (mm)

Copyright © 2005 by College Entrance Examination Board .M1 rights reserved,Visit apcentral.collegeboard.com (for AP professionals) and www.collegeboard.com/apstudents (for AP students and parents).

GO ON TO THE NEXT PAGE.

—7.0

—7 ..4

10, 2t). 30. 44). 50. 60. 10,Time (mm)

1.2i)G

I .0or’,’

B

Handout Page 73 of 182 Handout Page 73 of 182

Handout Page 73 of 182 Updated 4/9/2014 Handout Page 73 of 182

Page 74: V1) I - Blue Valley Schools Chemistry/AP... · 3DJH RI 3DJH RI 2001 AP® CHEMISTRY FREE-RESPONSEQUESTIONS Answer EITHER Question 2 below OR Question 3 printed on page 8

2005 AP® CHEMISTRY FREE-RESPONSE QUESTIONS (Form B)

(a) How many moles of X were initially in the flask?

(b) How many molecules of Y were produced in the first 20. minutes of the reaction?

(C) What is the order of this reaction with respect to X 7 Justify your answer.

(d) Write the rate law for this reaction.

(e) Calculate the specific rate constant for this reaction. Specify units.

(f) Calculate the concentration of X in the flask after a total of 150. minutes of reaction.

STOPIf you finish before time is called, you may check your work on this part only.

Do not turn to the other part of the test until you are told to do so.

(;opyright © 2005 bvCollege Entrance Examination Board. All rights reserved.Visit apcentraLcollegeboard.com (for AP professionals) and www.coliegeboard.com/apstudents (for AP students and parents).

9

Handout Page 74 of 182 Handout Page 74 of 182

Handout Page 74 of 182 Updated 4/9/2014 Handout Page 74 of 182

Page 75: V1) I - Blue Valley Schools Chemistry/AP... · 3DJH RI 3DJH RI 2001 AP® CHEMISTRY FREE-RESPONSEQUESTIONS Answer EITHER Question 2 below OR Question 3 printed on page 8

2005 AP® CHEMISTRY FREE-RESPONSE QUESTIONS (Form B)

CHEMISTRY

Part BTime—50 minutes

NO CALCULATORS MAY BE USED FOR PART B.

Answer Question 4 below. The Section II score weighting for this question is 15 percent.

4. Write the formulas to show the reactants and the products for any FIVE of the laboratory situations describedbelow. Answers to more than five choices will not be graded. In all cases, a reaction occurs. Assume thatsolutions are aqueous unless otherwise indicated. Represent substances in solution as ions if the substances areextensively ionized. Omit formulas for any ions or molecules that are unchanged by the reaction. You need notbalance the equations.

Example: A strip of magnesium is added to a solution of silver nitrate.

+ +

(a) A solution of potassium carbonate is added to a solution of strontium chloride.

(b) Propene is burned in air.

(c) Excess ammonia is added to a solution of zinc nitrate.

(d) Ethanoic acid (acetic acid) is added to a solution of barium hydroxide.

(e) A small piece of potassium is added to water.

(f) Powdered iron metal is strongly heated with powdered sulfur.

(g) A solution of sodium fluoride is added to a solution of hydrochloric acid.

(h) A strip of lead metal is added to a solution of silver nitrate.

Copyright QO5 by College Entrance Examination Board. All rights reserved.Visit apcentral.collegeboard.com (for AP professionals) and wwwcollegeboard.com/apstudents (for AP students and parents).

GO ON TOTHE NEXT PAGE.10

Handout Page 75 of 182 Handout Page 75 of 182

Handout Page 75 of 182 Updated 4/9/2014 Handout Page 75 of 182

Page 76: V1) I - Blue Valley Schools Chemistry/AP... · 3DJH RI 3DJH RI 2001 AP® CHEMISTRY FREE-RESPONSEQUESTIONS Answer EITHER Question 2 below OR Question 3 printed on page 8

2005 AP CHEMISTRY FREE-RESPONSE QUESTIONS (Form B)

Your responses to the rest of the questions in this part of the examination will be graded on the basis of the accuracyand relevance of the information cited. Explanations should be clear and well organized. Examples and equationsmay be included in your responses where appropriate. Specific answers are preferable to broad, diffuse responses.

Answer BOTH Question 5 below AND Question 6 printed on page 12. Both of these questions will be graded. TheSection II score weighting for these questions is 30 percent (15 percent each).

2 Al(s) + 2 KOHaq) + 4 HSO4(aq) + 22 HOUI —* 2 Kl SO4)2 12H,O(c) + 3 H2(’)

5. In an experiment, a student synthesizes alum. KAI(SO4)2l2H,O(s), by reacting aluminum metal with potassiumhydroxide and sulfuric acid, as represented in the balanced equation above.

(a) In order to synthesize alum, the student must prepare a 5.0 M solution of sulfuric acid. Describe theprocedure for preparing 50.0 mL of 5.0 M H2S04 using any of the chemicals and equipment listed below.Indicate specific amounts and equipment where appropriate.

10.0 M H7504 50.0 mL volumetric flask

Distilled water 50.0 mL buret

100 mL graduated cylinder 25.0 mL pipet

100 mL beaker 50 mL beaker

(b) Calculate the minimum volume of 5.0 M H2S04 that the student must use to react completely with2.7 g of aluminum metal.

(c) As the reaction solution cools, alum crystals precipitate. The student filters the mixture and dries thecrystals, then measures their mass.

(i) If the student weighs the crystals before they are completely dry, would the calculated percent yield begreater than, less than, or equal to the actual percent yield? Explain.

(ii) Cooling the reaction solution in an ice bath improves the percent yield obtained. Explain.

(d) The student heats crystals of pure alum. KAI(S04), 121-12O(s), in an open crucible to a constant mass. Themass of the sample after heating is less than the mass before heating. Explain.

Copyright © 2005 by College Entrance Examination Board. All rights reserved,Visit apcentral.collegeboardcom (for AP professionals) and www.coElegeboard.com/apstudents (for AP students and parents).

GO ON TO THE NEXT PAGE.11

Handout Page 76 of 182 Handout Page 76 of 182

Handout Page 76 of 182 Updated 4/9/2014 Handout Page 76 of 182

Page 77: V1) I - Blue Valley Schools Chemistry/AP... · 3DJH RI 3DJH RI 2001 AP® CHEMISTRY FREE-RESPONSEQUESTIONS Answer EITHER Question 2 below OR Question 3 printed on page 8

2005 AP® CHEMISTRY FREE-RESPONSE QUESTIONS (Form B)

298 K

6. Consider two containers of volume 1.0 L at 298 K. as shown above. One container holds 0.10 mol N2(g) and

the other holds 0.10 mol H,(g). The average kinetic energy of the N,(g) molecules is 6.2 x l021 J. Assume

that the N2(g) and the H,(g) exhibit ideal behavior.

(a) Is the pressure in the container holding the H2(g) less than, greater than, or equal to the pressure in thecontainer holding the N2(g) ? Justify your answer.

(b) What is the average kinetic energy of the H,(g) molecules?

(c) The molecules of which gas, N, or H2, have the greater average speed? Justify your answer.

(d) What change could be made that would decrease the average kinetic energy of the N,(g) molecules in thecontainer?

(e) If the volume of the container holding the H2(g) was decreased to 0,50 L at 298 K, what would be thechange in each of the following variables? In each case, justify your answer.

(i) The pressure within the container

(ii) The average speed of the H,(g) molecules

Copyright © 2005 by College Entrance Examination Board. All rights reserved.Visit apcentral.collegehoard.com (for AP professionals) and www.collegeboard.com/apstudents (for AP students and parents).

GO ON TO THE NEXT PAGE.

1.0 L 1.0 L298 K

12

Handout Page 77 of 182 Handout Page 77 of 182

Handout Page 77 of 182 Updated 4/9/2014 Handout Page 77 of 182

Page 78: V1) I - Blue Valley Schools Chemistry/AP... · 3DJH RI 3DJH RI 2001 AP® CHEMISTRY FREE-RESPONSEQUESTIONS Answer EITHER Question 2 below OR Question 3 printed on page 8

2005 AP® CHEMISTRY FREE-RESPONSE QUESTIONS (Form B)

Answer EITHER Question 7 below OR Question 8 printed on page 14. Only one of these two questions will begraded. If you start both questions, be sure to cross out the question you do not want graded. The Section II scoreweighting for the question you choose is 15 percent.

7. Answer the following questions about thermodynamics.

Enthalpy of Combustion,Substance Combustion Reaction -1iXBcomb , at 298 K (kJ mol )

H1(g) H2(g) + O2(g) — H20(1) —290

C(s) C(s) + 02(g) -4 C01(g) —390

CH3OH(1) —730

(a) In the empty box in the table above, write a balanced chemical equation for the complete combustion ofmole of CH3OH(/). Assume products are in their standard states at 298 K. Coefficients do not need

to be whole numbers.

(b) On the basis of your answer to part (a) and the information in the table, determine the enthalpy change forthe reaction C(s) + H2(g) + H20(1) - CH3OH(1).

(c) Write the balanced chemical equation that shows the reaction that is used to determine the enthalpy offormation for one mole of CH3OH(1).

(d) Predict the sign of IS° for the combustion of H,(g). Explain your reasoning.

(e) On the basis of bond energies, explain why the combustion of H1(g) is exothermic.

Copyright @ 2005 by College Entrance Examination Board, All rights reserved.Visit apcentral.collegeboard.com (for AP professionals) and www.collegeboard.comlapstudents (for AP students and parents).

GO ON TO THE NEXT PAGE.13

Handout Page 78 of 182 Handout Page 78 of 182

Handout Page 78 of 182 Updated 4/9/2014 Handout Page 78 of 182

Page 79: V1) I - Blue Valley Schools Chemistry/AP... · 3DJH RI 3DJH RI 2001 AP® CHEMISTRY FREE-RESPONSEQUESTIONS Answer EITHER Question 2 below OR Question 3 printed on page 8

2005 AP® CHEMISTRY FREE-RESPONSE QUESTIONS (Form B)

8. Use principles of atomic structure, bonding, and intermolecular forces to answer the following questions. Yourresponses jflt include specific information about Ji substances referred to in each part.

(a) Draw a complete Lewis electron-dot structure for the CS2 molecule, Include all valence electrons in yourstructure.

(b) The carbon-to-sulfur bond length in CS, is 160 picometers. Is the carbon-to-selenium bond length in CSe2expected to be greater than, less than, or equal to this value? Justify your answer.

(c) The bond energy of the carbon-to-sulfur bond in CS2 is 577 kJ mo[. Is the bond energy of the carbon-to-selenium bond in CSe2 expected to be greater than, less than, or equal to this value? Justify your answer.

HHHI I

H—C—C—C—HI IHH H

0

H

Methanoic acid 374 K

Copyright © 2005 by College Entrance Examination Board. All rights reserved.Visit apcentral.collegeboard.com (for AP professionals) and www.collegeboard.com/apstudents (for AP students and parents).

Propane Methanoic Acid

(d) The complete structural formulas of propane, C3H8,and methanoic acid, HCOOH, are shown above. In thetable below, write the type(s) of intermolecular attractive force(s) that occur in each substance.

Propane 229 K

(e) Use principles of intermolecular attractive forces to explain why methanoic acid has a higher boiling pointthan propane.

END OF EXAM

14

Handout Page 79 of 182 Handout Page 79 of 182

Handout Page 79 of 182 Updated 4/9/2014 Handout Page 79 of 182

Page 80: V1) I - Blue Valley Schools Chemistry/AP... · 3DJH RI 3DJH RI 2001 AP® CHEMISTRY FREE-RESPONSEQUESTIONS Answer EITHER Question 2 below OR Question 3 printed on page 8

2006 AP® CHEMISTRY FREE-RESPONSE QUESTIONS

CHEMISTRYSection II

(Total time—90 minutes)

Part A

Time—40 minutes

YOU MAY USE YOUR CALCULATOR FOR PART A.

CLEARLY SHOW THE METHOD USED AND THE STEPS INVOLVED IN ARRIVING AT YOUR ANSWERS.It is to your advantage to do this, since you may obtain partial credit if you do and you will receive little or no credit ifyou do not. Attention should be paid to significant figures.

Be sure to write all your answers to the questions on the lined pages following each question in the booklet with thepink cover. Do NOT write your answers on the green insert,

Answer Question I below. The Section II score weighting for this question is 20 percent.

I. Answer the following questions that relate to solubility of salts of lead and barium.

(a) A saturated solution is prepared by adding excess Pb12(s) to distilled water to form I .0 L of solution at

25°C. The concentration of Pb2(aq) in the saturated solution is found to be 1.3 x l0 M. The chemicalequation for the dissolution of PbI,(s) in water is shown below.

PbI.,(s) Pb2(aq) + 2 1(aq)

(i) Write the equilibrium-constant expression for the equation.

(ii) Calculate the molar concentration of U(aq) in the solution.

(iii) Calculate the value of the equilibrium constant, Kçj,.

(b) A saturated solution is prepared by adding Pb12(s) to distilled water to form 2.0 L of solution at 25°C. Whatare the molar concentrations of Pb2(aq) and L(aq) in the solution? Justify your answer.

(c) Solid Na! is added to a saturated solution of Pb12 at 25°C. Assuming that the volume of the solution doesnot change. does the molar concentration of Pb2(aq) in the solution increase, decrease, or remain thesame’? Justify your answer.

(d) The value of Kçp for the salt BaCrO4 is 1.2 x l0’°. When a 500. mL sample of 8.2 x 10.6 M Ba(NO3),

is added to 500. mL of 8.2 x l06 M Na2CrO4,no precipitate is observed.

(i) Assuming that volumes are additive, calculate the molar concentrations of Ba2(aq) and CrO42(aq)in the 1.00 L of solution.

(ii) Use the molar concentrations of Ba2(aq) ions and Cr032(aq) ions as determined above to showwhy a precipitate does not form. You must include a calculation as part of your answer.

© 2006 The College Board. All rights reserved.Visit apcentral.collegeboard.com (for AP professionals) and www.collegeboard.com/apstudents (for students and parents).

GO ON TO THE NEXT PAGE.6

Handout Page 80 of 182 Handout Page 80 of 182

Handout Page 80 of 182 Updated 4/9/2014 Handout Page 80 of 182

Page 81: V1) I - Blue Valley Schools Chemistry/AP... · 3DJH RI 3DJH RI 2001 AP® CHEMISTRY FREE-RESPONSEQUESTIONS Answer EITHER Question 2 below OR Question 3 printed on page 8

2006 AP CHEMISTRY FREE-RESPONSE QUESTIONS

Answer EITHER Question 2 below OR Question 3 printed on page 8. Only one of these two questions will begraded. If you start both questions, be sure to cross out the question you do not want graded. The Section 11 scoreweighting for the question you choose is 20 percent.

CO(g) + O2(g) C02(g)

2. The combustion of carbon monoxide is represented by the equation above.

(a> Determine the value of the standard enthalpy change, AH, for the combustion of CO(g) at 298 K usingthe following information.

C(s) + -O(g) — CO(g) ATh98 = — 110.5 kJ mol’

C(s) +02(g) —* CO(g) All298 = —393.5 kJ mo1

(b) Determine the value of the standard entropy change, AS, for the combustion of CO(g) at 298 K usingthe information in the following table.

S,98Substance

(J mol K)

CO(g) 197.7

CO,(g) 213.7

02(g) 205.1

(c) Determine the standard free energy change, for the reaction at 298 K. Include units withyour answer.

(d) Is the reaction spontaneous under standard conditions at 298 K? Justify your answer.

(e) Calculate the value of the equilibrium constant, Keq, for the reaction at 298 K.

© 2006 The College Board, All rights reserved.Visit apcentral.coliegeboard.com (for AP professionals) and wwwcollegeboard.com/apstudenes (for students and parents).

GO ON TO THE NEXT PAGE.7

Handout Page 81 of 182 Handout Page 81 of 182

Handout Page 81 of 182 Updated 4/9/2014 Handout Page 81 of 182

Page 82: V1) I - Blue Valley Schools Chemistry/AP... · 3DJH RI 3DJH RI 2001 AP® CHEMISTRY FREE-RESPONSEQUESTIONS Answer EITHER Question 2 below OR Question 3 printed on page 8

2006 AP® CHEMISTRY FREE-RESPONSE QUESTIONS

3. Answer the following questions that relate to the analysis of chemical compounds.

(a) A compound containing the elements C. H, N. and 0 is analyzed. When a 1.2359 g sample is burnedin excess oxygen, 2.241 g of C02(g) is formed. The combustion analysis also showed that the samplecontained 0.0648 g of H.

(i) Determine the mass, in grams, of C in the 1.2359 g sample of the compound.

(ii) When the compound is analyzed for N content only, the mass percent of N is foundto be 28.84 percent. Determine the mass, in grams, of N in the original 1.2359 g sample ofthe compound.

(iii) Determine the mass, in grams, of 0 in the original 1.2359 g sample of the compound.

(iv) Determine the empirical formula of the compound.

(b) A different compound, which has the empirical formula CH9Br, has a vapor density of 6,00 g L’ at 375 Kand 0.983 atm. Using these data, determine the following.

i) The molar mass of the compound

(ii) The molecular formula of the compound

STOPIf you finish before time is called, you may check your work on this part only.

Do not turn to the other part of the test until you are told to do so.

© 2006 The College Board, All rights reserved.Visit apcentral.collegehoard.com (for AP professionals) and vww.collegeboard.coin/apstudents (6r students and parents).

8

Handout Page 82 of 182 Handout Page 82 of 182

Handout Page 82 of 182 Updated 4/9/2014 Handout Page 82 of 182

Page 83: V1) I - Blue Valley Schools Chemistry/AP... · 3DJH RI 3DJH RI 2001 AP® CHEMISTRY FREE-RESPONSEQUESTIONS Answer EITHER Question 2 below OR Question 3 printed on page 8

2006 AP® CHEMISTRY FREE-RESPONSE QUESTIONS

CHEMISTRY

Part B

Time—50 minutesNO CALCULATORS MAY BE USED FOR PART B.

Answer Question 4 below. The Section 11 score weighting for this question is 15 percent.

4. Write the formulas to show the reactants and the products for any FIVE of the laboratory situations describedbelow. No more than five choices will be graded. In all cases, a reaction occurs. Assume that solutions areaqueous unless otherwise indicated. Represent substances in solution as ions if the substances are extensivelyionized. Omit formulas for any ions or molecules that are unchanged by the reaction. You need not balance theequations.

Example: A strip of magnesium is added to a solution of silver nitrate.

£. + A + 1(a) Solid potassium chlorate is strongly heated.

(b) Solid silver chloride is added to a solution of concentrated hydrochloric acid.

(c) A solution of ethanoic (acetic) acid is added to a solution of barium hydroxide.

(d) Ammonia gas is bubbled into a solution of hydrotluoric acid.

(e) Zinc metal is placed in a solution of copper(II) sulfate.

(0 Hydrogen phosphide (phosphine) gas is added to boron trichloride gas.

(g) A solution of nickel(II) bromide is added to a solution of potassium hydroxide.

(h) Hexane is combusted in air.

2006 The College Board. All rights reserved.Visit apcentral.collegeboardcom (for AP professionals) and wwwcollegeboard,com/apstudents (for students and parents).

GO ON TO THE NEXT PAGE.9q«

Handout Page 83 of 182 Handout Page 83 of 182

Handout Page 83 of 182 Updated 4/9/2014 Handout Page 83 of 182

Page 84: V1) I - Blue Valley Schools Chemistry/AP... · 3DJH RI 3DJH RI 2001 AP® CHEMISTRY FREE-RESPONSEQUESTIONS Answer EITHER Question 2 below OR Question 3 printed on page 8

2006 AP® CHEMISTRY FREE-RESPONSE QUESTIONS

Your responses to the rest of the questions in this part of the examination will be graded on the basis of the accuracyand relevance of the information cited. Explanations should be clear and well organized. Examples and equationsmay be included in your responses where appropriate, Specific answers are preferable to broad, diffuse responses.

Answer BOTH Question 5 below AND Question 6 printed on pages 11-12. Both of these questions will be graded.The Section II score weighting for these questions is 30 percent (15 percent each).

5. Three pure, solid compounds labeled X, Y. and Z are placed on a lab bench with the objective of identifyingeach one. It is known that the compounds (listed in random order) are KC1, Na2CO3,and MgSO4 . A studentperforms several tests on the compounds; the results are summarized in the table below.

Result of AddingpH of an Aqueous 1.0 M NaOH to a Result of Adding

Compound Solution of the Solution of the 1.0 M HCl Dropwise toCompound Compound the Solid Compound

x > 7 No observed reaction Evolution of a gas

y 7 No observed reaction No observed reaction

Formation of a whiteZ 7 No observed reactionprecipitate

(a) Identify each compound based on the observations recorded in the table.

Compound X

_____________________

Compound Y_____________________

Compound Z_____________________

(b) Write the chemical formula for the precipitate produced when 1.0 M NaOH is added to a solution ofcompound Z.

(C) Explain why an aqueous solution of compound X has a pH value greater than 7. Write an equation as partof your explanation.

(d) One of the testing solutions used was 1.0 M NaOH. Describe the steps for preparing 100. mL of1.0 M NaOH from a stock solution of 3.0 M NaOH using a 50 mL buret, a 100 mL volumetric flask.distilled water, and a small dropper.

(e) Describe a simple laboratory test that you could use to distinguish between Na2CO3(s) and CaCO3(s).In your description, specify how the results of the test would enable you to determine which compound wasNa,CO(s) and which compound was CaCO3s).

2006 The College Board. All rights reserved.Visit apcentral.collegeboar&com (for A)’ protessonals) and wwwcollegeboarcicom/apstudents (for students and parents).

GO ON TO THE NEXT PAGE.10

Handout Page 84 of 182 Handout Page 84 of 182

Handout Page 84 of 182 Updated 4/9/2014 Handout Page 84 of 182

Page 85: V1) I - Blue Valley Schools Chemistry/AP... · 3DJH RI 3DJH RI 2001 AP® CHEMISTRY FREE-RESPONSEQUESTIONS Answer EITHER Question 2 below OR Question 3 printed on page 8

2006 AP® CHEMISTRY FREE-RESPONSE QUESTIONS

6. Answer each of the following in terms of principles of molecular behavior and chemical concepts.

(a) The structures for glucose, C6H1206,and cyclohexane, C6H12, are shown below.

C[1OH [1 K\_ \ /C—o C—C

H\/I \/H H\//

H/\*

H

Identify the type(s) of intermolecular attractive forces in

(i) pure glucose

(ii) pure cyclohexane

(b) Glucose is soluble in water but cyclohexane is not soluble in water. Explain.

(c) Consider the two processes represented below.

Process I: H,O(l) — H20(g) = +44.OkJ mol

Process 2: H10(i) —÷ H,(g)+ -O,(g) H° = +286kJmol

(i) For each of the two processes, identify the type(s) of intermolecular or intramolecular attractive forcesthat must be overcome for the process to occur.

(ii) Indicate whether you agree or disagree with the statement in the box below. Support your answer witha short explanation.

When water boils, H20 molecules break apa toform hydrogen molecules and oxygen molecules.

© 2006 The College Board. All rights reserved.Visit apcentral.collegeboard.corn (for AP professionals) and www.collegeboard.com/apstudents (for students and parents).

GO ON TO THE NEXT PAGE.11

Handout Page 85 of 182 Handout Page 85 of 182

Handout Page 85 of 182 Updated 4/9/2014 Handout Page 85 of 182

Page 86: V1) I - Blue Valley Schools Chemistry/AP... · 3DJH RI 3DJH RI 2001 AP® CHEMISTRY FREE-RESPONSEQUESTIONS Answer EITHER Question 2 below OR Question 3 printed on page 8

2006 AP® CHEMISTRY FREE-RESPONSE QUESTIONS

(d) Consider the four reaction-energy profile diagrams shown below.

Reaction Progress Reaction Progress Reaction Progress Reaction Progress

Diagram I Diagram 2 l)iagram 3 1)iagram 4

(i) Identify the two diagrams that could represent a catalyzed and an uncatalyzed reaction pathway for thesame reaction. Indicate which of the two diagrams represents the catalyzed reaction pathway for thereaction.

(ii) Indicate whether you agree or disagree with the statement in the box below. Support your answer witha short explanation.

Adding a catalyst to a reaction mixture adds energythat causes the reaction to proceed more quickly.

2006 The College Board. All rights reserved.Visit apcentral.collegeboard.corn (for AP professionals) and svwwcollegeboard.com/apstudents (for students and parents).

GO ON TO THE NEXT PAGE.12

Handout Page 86 of 182 Handout Page 86 of 182

Handout Page 86 of 182 Updated 4/9/2014 Handout Page 86 of 182

Page 87: V1) I - Blue Valley Schools Chemistry/AP... · 3DJH RI 3DJH RI 2001 AP® CHEMISTRY FREE-RESPONSEQUESTIONS Answer EITHER Question 2 below OR Question 3 printed on page 8

2006 AP CHEMISTRY FREE-RESPONSE QUESTIONS

Answer EITHER Question 7 below OR Question 8 printed on page 14. Only one of these two questions will begraded. If you start both questions, be sure to cross Out the question you do not want graded. The Section II scoreweighting for the question you choose is 15 percent.

7. Answer the following questions about the structures of ions that contain only sulfur and fluorine.

(a) The compounds SF4 and BF3 react to form an ionic compound according to the following equation.

SF4 + BF3 —* SFBF4

(i) Draw a complete Lewis structure for the SF3 cation in SF1BF4.

(ii) Identify the type of hybridization exhibited by sulfur in the SF3 cation.

(iii) Identify the geometry of the SF3 cation that is consistent with the Lewis structure drawnin part (a)(i).

(iv) Predict whether the F—S—F bond angle in the SF3 cation is larger than, equal to, or smallerthan 109.5°. Justify your answer.

(b) The compounds SF4 and CsF react to form an ionic compound according to the following equation.

SF4 + CsF — CsSF5

(i) Draw a complete Lewis structure for the SF5 anion in CsSF5.

(ii) Identify the type of hybridization exhibited by sulfur in the SF5 anion.

(iii) Identify the geometry of the SF5 anion that is consistent with the Lewis structure drawnin part (b)(i).

(iv) Identify the oxidation number of sulfur in the compound CsSF5.

© 2006 The College Board. All rights reserved.Visit apentral.collegeboard.corn (for AP professionals) and www.collegeboard.com/apstudents (for students and parents).

GO ON TO THE NEXT PAGE.13

Handout Page 87 of 182 Handout Page 87 of 182

Handout Page 87 of 182 Updated 4/9/2014 Handout Page 87 of 182

Page 88: V1) I - Blue Valley Schools Chemistry/AP... · 3DJH RI 3DJH RI 2001 AP® CHEMISTRY FREE-RESPONSEQUESTIONS Answer EITHER Question 2 below OR Question 3 printed on page 8

2006 AP® CHEMISTRY FREE-RESPONSE QUESTIONS

8. Suppose that a stable element with atomic number 119, symbol Q, has been discovered.

(a) Write the ground-state electron configuration for Q, showing only the valence-shell electrons.

(b) Would Q be a metal or a nonmetal? Explain in terms of electron configuration,

(c) On the basis of periodic trends, would Q have the largest atomic radius in its group or would it have thesmallest? Explain in terms of electronic structure.

(d) What would be the most likely charge of the Q ion in stable ionic compounds?

(e) Write a balanced equation that would represent the reaction of Q with water.

(f) Assume that Q reacts to form a carbonate compound.

(i) Write the formula for the compound formed between Q and the carbonate ion, C032.

(ii) Predict whether or not the compound would be soluble in water. Explain your reasoning.

STOP

END OF EXAM

2006 The College Board. All rights reserved.Visit apcentral.collegeboard.om (for AP professionals) and www.collegeboard.com/apstudents (for students and parents).

14

Handout Page 88 of 182 Handout Page 88 of 182

Handout Page 88 of 182 Updated 4/9/2014 Handout Page 88 of 182

Page 89: V1) I - Blue Valley Schools Chemistry/AP... · 3DJH RI 3DJH RI 2001 AP® CHEMISTRY FREE-RESPONSEQUESTIONS Answer EITHER Question 2 below OR Question 3 printed on page 8

2006 AP® CHEMISTRY FREE-RESPONSE QUESTIONS (Form B)

CHEMISTRYSection II

(Total time—90 minutes)

Part A

Time—40 minutesYOU MAY USE YOUR CALCULATOR FOR PART A.

CLEARLY SHOW THE METHOD USED AND THE STEPS INVOLVED IN ARRIVING AT YOUR ANSWERS.It is to your advantage to do this, since you may obtain partial credit if you do and you will receive little or no credit ifyou do not. Attention should be paid to significant figures.

Be sure to write all your answers to the questions on the lined pages following each question in the goldenrodbooklet. Do NOT write your answers on the lavender insert.

Answer Question 1 below. The Section II score weighting for this question is 20 percent.

C6H5COOH(s) C6H5C00 (aq) + H(aq) K 6.46 x I 0

I. Benzoic acid,C6H5COOH, dissociates in water as shown in the equation above. A 25.0 mL sample of anaqueous solution of pure benzoic acid is titrated using standardized 0.150 M NaOH.

(a) After addition of 15.0 mL of the 0.150 M NaOH, the pH of the resulting solution is 4.37. Calculate eachof the following.

(1) [Hj in the solution

(ii) [0H] in the solution

(iii) The number of moles of NaOH added

(iv) The number of moles ofC6H5COO(aq) in the solution

(v) The number of moles of C6H5COOH in the solution

(b) State whether the solution at the equivalence point of the titration is acidic, basic, or neutral. Explain yourreasoning.

In a different titration, a 0.7529 g sample of a mixture of solid C6H5COOH and solid NaCI is dissolved inwater and titrated with 0.150 M NaOH. The equivalence point is reached when 24.78 inL of the base solutionis added.

(c) Calculate each of the following.

(i) The mass, in grams. of benzoic acid in the solid sample

(ii) The mass percentage of benzoic acid in the solid sample

© 2006 The College Board. All rights reserved.Visit apcentraLcollegeboaricom (for AP professionals) and wwwcollegeboard.com/apstudents (for students and parents).

GO ON TO THE NEXT PAGE.6

Handout Page 89 of 182 Handout Page 89 of 182

Handout Page 89 of 182 Updated 4/9/2014 Handout Page 89 of 182

Page 90: V1) I - Blue Valley Schools Chemistry/AP... · 3DJH RI 3DJH RI 2001 AP® CHEMISTRY FREE-RESPONSEQUESTIONS Answer EITHER Question 2 below OR Question 3 printed on page 8

2006 AP CHEMISTRY FREE-RESPONSE QUESTIONS (Form B)

Answer EITHER Question 2 OR Question 3 below. Only one of these two questions will be graded. If you start bothquestions, be sure to cross out the question you do not want graded. The Section II score weighting for the questionyou choose is 20 percent.

2. Answer the following questions about voltaic cells.

(a) A voltaic cell is set up using Al/Al3 as one half-cell and SnfSn2 as the other half-cell. The half-cellscontain equal volumes of solutions and are at standard conditions.

(i) Write the balanced net-ionic equation for the spontaneous cell reaction.

(ii) Determine the value, in volts, of the standard potential, E°. for the spontaneous cell reaction.

(iii) Calculate the value of the standard free-energy change, G°, for the spontaneous cell reaction.Include units with your answer.

(iv) If the cell operates until [A13j is 1.08 M in the Al /Al3 half-cell, what is ISn21 in theSn ISn2 half-cell?

(b) In another voltaic cell with Al /Al and Sn /Sn2 half-cells, [Sn21 is 0.010 M and [Al 3+j is 1.00 M.Calculate the value, in volts, of the cell potential, Eceti, at 25°C.

3. Answer the following questions about the thermodynamics of the reactions represented below.

Reaction X: -I,(s) + Cl2(g) ICI(g) = 18 kJ mol’, LS = 78 J K mol’

Reaction Y: 4-12(s) + --Br1(1) lBr(g) AH. = 41 kJ mol, tS = l24J K’ mol1

(a) Is reaction X, represented above, spontaneous under standard conditions? Justify your answerwith a calculation.

(b) Calculate the value of the equilibrium constant, Keq for reaction X at 25°C.

(c) What effect will an increase in temperature have on the equilibrium constant for reaction X?Explain your answer.

(d) Explain why the standard entropy change is greater for reaction Y than for reaction X.

(e) Above what temperature will the value of the equilibrium constant for reaction Y be greater than 1.0?Justify your answer with calculations.

(0 For the vaporization of solid iodine, l(s) — I(g), the value of zV-L is 62 kJ mol. Using this

information, calculate the value of for the reaction represented below.

U(g) + C12(g) 2 IC1(g.

STOPIf you finish before time is called, you may check your work on this part only.

Do not turn to the other part of the test until you are told to do so.

© 2006 The College Board. All rights reserved.Visit apcentral.collegeboard.com (for AP professionals) and www.collegeboard.com/apstudents (for students and parents).

7

Handout Page 90 of 182 Handout Page 90 of 182

Handout Page 90 of 182 Updated 4/9/2014 Handout Page 90 of 182

Page 91: V1) I - Blue Valley Schools Chemistry/AP... · 3DJH RI 3DJH RI 2001 AP® CHEMISTRY FREE-RESPONSEQUESTIONS Answer EITHER Question 2 below OR Question 3 printed on page 8

2006 AP® CHEMISTRY FREE-RESPONSE QUESTIONS (Form B)

CHEMISTRYPart B

Time—SO minutesNO CALCULATORS MAY BE USED FOR PART B.

Answer Question 4 below. The Section II score weighting for this question is 15 percent.

4. Write the formulas to show the reactants and the products for any FIVE of the laboratory situations describedbelow. Answers to more than five choices will not be graded. In all cases, a reaction occurs. Assume thatsolutions are aqueous unless otherwise indicated, Represent substances in solution as ions if the substances areextensively ionized. Omit formulas for any ions or molecules that are unchanged by the reaction. You need notbalance the equations.

Example: A strip of magnesium is added to a solution of silver nitrate.

£. + A ÷ A

(a) Solid calcium carbonate is strongly heated.

(b) A strip of magnesium metal is placed in a solution of iron(II) chloride.

(c) Boron trifluoride gas is mixed with ammonia gas.

(d) Excess concentrated hydrochloric acid is added to a soJution of nickel(ll) nitrate.

(e) Solid ammonium chloride is added to a solution of potassium hydroxide.

(fl Propanal is burned in air.

(g) A strip of aluminum foil is placed in liquid bromine.

(h) Solid copper(ll) sulfide is strongly heated in air.

© 2006 The College Board. All rights reserved.Visit apcentraLcollegeboard.com (tbr AP professionals) and w veollegeboard.corn/apstudents (for students and parents).

GO ON TO THE NEXT PAGE.8

Handout Page 91 of 182 Handout Page 91 of 182

Handout Page 91 of 182 Updated 4/9/2014 Handout Page 91 of 182

Page 92: V1) I - Blue Valley Schools Chemistry/AP... · 3DJH RI 3DJH RI 2001 AP® CHEMISTRY FREE-RESPONSEQUESTIONS Answer EITHER Question 2 below OR Question 3 printed on page 8

2006 AP® CHEMISTRY FREE-RESPONSE QUESTIONS (Form B)

Your responses to the rest of the questions in this part of the examination will be graded on the basis of the accuracyand relevance of the information cited. Explanations should be clear and well organized. Examples and equationsmay be included in your responses where appropriate. Specific answers are preferable to broad, diffuse responses.

Answer BOTH Question 5 below AND Question 6 printed on page II. Both of these questions will be graded. TheSection II score weighting for these questions is 30 percent (15 percent each).

5. A student carries out an experiment to determine the equilibrium constant for a reaction by colorimetric(spectrophotometric) analysis. The production of the red-colored species FeSCN2(aq) is monitored.

(a) The optimum wavelength for the measurement of [FeSCN2j must first be detennined. The plot ofabsorbance, A, versus wavelength, X, for FeSCN2(aq) is given below. What is the optimum wavelengthfor this experiment? Justify your answer.

(b) A calibration plot for the concentration of FeSCN2(aq) is prepared at the optimum wavelength. The databelow give the absorbances measured for a set of solutions of known concentration of FeSCN2(aq).

Concentration-I Absorbance

(molLl.lx103 0.030

3.0x103 0.0658.0x104 0.160

12xl04 0.239

l8x104 ] 0.340

© 2006 The College Board. All rights reserved.Visit apcentraLcollegeboardcom (for AP professionals) and www.collegeboardcom/apstudents (for students and parents).

GO ON TO THE NEXT PAGE.

36t) 400 44() 480 52() 56() (400Wavelength (nrn)

9

Handout Page 92 of 182 Handout Page 92 of 182

Handout Page 92 of 182 Updated 4/9/2014 Handout Page 92 of 182

Page 93: V1) I - Blue Valley Schools Chemistry/AP... · 3DJH RI 3DJH RI 2001 AP® CHEMISTRY FREE-RESPONSEQUESTIONS Answer EITHER Question 2 below OR Question 3 printed on page 8

2006 AP® CHEMISTRY FREE-RESPONSE QUESTIONS (Form B)

(i) Draw a Beer’s law calibration plot of all the data on the grid below. Indicate the scale on the horizontalaxis by labeling it with appropriate values.

04O-

OJO

020

0. l()

0(3

Concentration

(ii) An FeSCN2(aq) solution of unknown concentration has an absorbance of 0.300. Use the plot youdrew in part (j) to determine the concentration, in moles per liter, of this solution.

(c) The purpose of the experiment is to determine the equilibrium constant for the reaction represented below.

Fe3(aq) + SCN(aq) FeSCN2(aq)

(i) Write the equilibrium-constant expression for K.

(ii) The student combines solutions of Fe(N03)3 and KSCN to produce a solution in which the initialconcentrations of Fe3(aq) and SCN(aq) are both 6.0 x l0 M. The absorbance of this solution ismeasured, and the equilibrium FeSCN2(aq) concentration is found to be 1.0 x 10 M. Determine thevalue of

(d) If the student’s equilibrium FeSCN2(aq) solution of unknown concentration fades to a lighter color beforethe student measures its absorbance, will the calculated value of K be too high, too low, or unaffected?Justify your answer.

2006 The College Board. All rights reserved.Visit apcentralcollegeboard.coni (for AP professionals) and www.collegeboard.com/apstudents (for students and parents).

GO ON TO THE NEXT PAGE.10

Handout Page 93 of 182 Handout Page 93 of 182

Handout Page 93 of 182 Updated 4/9/2014 Handout Page 93 of 182

Page 94: V1) I - Blue Valley Schools Chemistry/AP... · 3DJH RI 3DJH RI 2001 AP® CHEMISTRY FREE-RESPONSEQUESTIONS Answer EITHER Question 2 below OR Question 3 printed on page 8

2006 AP CHEMISTRY FREE-RESPONSE QUESTIONS (Form B)

GeCI4 SeC!4 ICl4 1Cl4

6. The species represented above all have the same number of chlorine atoms attached to the central atom.

(a) Draw the Lewis structure (electron-dot diagram) of each of the four species. Show all valence electrons inyour structures.

(b) On the basis of the Lewis structures drawn in part (a), answer the following questions about the particularspecies indicated.

(i) What is the Cl — Ge — Cl bond angle in GeCl4?

(ii) Is SeCl3 polar? Explain.

(iii) What is the hybridization of the I atom in Id4?

(iv) What is the geometric shape formed by the atoms in ICl3?

2006 The College Board. All rights reserved.Visit apcentral.collegeboardcorn (for AP professionals) and www.collegeboardcom/apstudents (for students and parents).

GO ON TO THE NEXT PAGE.11

Handout Page 94 of 182 Handout Page 94 of 182

Handout Page 94 of 182 Updated 4/9/2014 Handout Page 94 of 182

Page 95: V1) I - Blue Valley Schools Chemistry/AP... · 3DJH RI 3DJH RI 2001 AP® CHEMISTRY FREE-RESPONSEQUESTIONS Answer EITHER Question 2 below OR Question 3 printed on page 8

2006 AP® CHEMISTRY FREE-RESPONSE QUESTIONS (Form B)

Answer EITHER Question 7 OR Question 8 below. Only one of these two questions will be graded. If you start bothquestions, be sure to cross out the question you do not want graded. The Section II score weighting for the questionyou choose is 15 percent.

7. Account for each of the following observations in terms of atomic theory and/or quantum theory.

(a) Atomic size decreases from Na to Cl in the periodic table.

(b) Boron commonly forms molecules of the type BX3. These molecules have a trigonal planar structure.

(c) The first ionization energy of K is less than that of Na.

(d) Each element displays a unique gas-phase emission spectrum.

8. Use chemical and physical principles to account for each of the following.

(a) An aluminum container filled with an aqueous solution of CuSO4 eventually developed a leak.Include a chemical equation with your answer.

(b) The inside of a metal container was cleaned with steam and immediately sealed. Later, the containerimploded.

(c) Skin feels cooler after rubbing alcohol has been applied to it.

(d) The redness and itching of the skin caused by ant bites (injections of methanoic acid, HCO2H) canbe relieved by applying a paste made from water and baking soda (solid sodium hydrogen carbonate).Include a chemical equation with your answer.

STOP

END OF EXAM

© 2006 The College Board. All rights reserved.Visit apcentral.ollegeboard.eorn (for AP professionals) and www.collegeboardcom!apstudents (for students and parents).

12

Handout Page 95 of 182 Handout Page 95 of 182

Handout Page 95 of 182 Updated 4/9/2014 Handout Page 95 of 182

Page 96: V1) I - Blue Valley Schools Chemistry/AP... · 3DJH RI 3DJH RI 2001 AP® CHEMISTRY FREE-RESPONSEQUESTIONS Answer EITHER Question 2 below OR Question 3 printed on page 8

2007 AP® CHEMISTRY FREE-RESPONSE QUESTIONS

CHEMISTRYSection II

(Total time—95 minutes)

Part A

Time—55 minutesYOU MAY USE YOUR CALCULATOR FOR PART A.

CLEARLY SHOW THE METHOD USED AND THE STEPS INVOLVED IN ARRIVING AT YOUR ANSWERS.It is to your advantage to do this, since you may obtain partial credit if you do and you will receive little or no credit ifyou do not, Attention should be paid to significant figures.

Be sure to write all your answers to the questions on the lined pages following each question in the booklet with thepink cover. Do NOT write your answers on the green insert.

Answer Questions 1, 2, and 3. The Section II score weighting for each question is 20 percent.

HF(aq) + 1120(1) H3O(aq) + F(aq) K0 = 7.2 x l0

I. Hydrofluoric acid, HF(aq), dissociates in water as represented by the equation above.

(a) Write the equilibriumconstant expression for the dissociation of HF(aq) in water.

(b) Calculate the molar concentration of H3O in a 0.40 M HF(aq) solution.

HF(aq) reacts with NaOH(aq) according to the reaction represented below.

HF(aq) + OW(aq) —* H,O(1) + F(aq)

A volume of 15 mL of 0.40 M NaOH(aq) is added to 25 mL of 0.40 M HF(aq) solution. Assume thatvolumes are additive.

(C) Calculate the number of moles of HF(aq) remaining in the solution.

(d) Calculate the molar concentration of F(aq) in the solution.

(e) Calculate the pH of the solution.

) 2007 The College Board. All rights reserved.Visit apcentralcollegeboard.com (for AP professionals) and www.collegeboard.com/apstudents (for students and parents).

GO ON TO THE NEXT PAGE.-6-

Handout Page 96 of 182 Handout Page 96 of 182

Handout Page 96 of 182 Updated 4/9/2014 Handout Page 96 of 182

Page 97: V1) I - Blue Valley Schools Chemistry/AP... · 3DJH RI 3DJH RI 2001 AP® CHEMISTRY FREE-RESPONSEQUESTIONS Answer EITHER Question 2 below OR Question 3 printed on page 8

2007 AP® CHEMISTRY FREE-RESPONSE QUESTIONS

N-(g) + 3 Fg) —> 2 NF(g) AH198 = — 264 kJ mol: AS°98 = — 278 1 K1 mol’

2. The following questions relate to the synthesis reaction represented by the chemical equation in the box above.

(a) Calculate the value of the standard free energy change, AG, for the reaction.

(b) Determine the temperature at which the equilibrium constant, Kç(/. for the reaction is equal to 1.00.(Assume that Aft° and AS° are independent of temperature.)

(c) Calculate the standard enthalpy change, All0, that occurs when a 0.256 mol sample of NF3(g) is formedfrom N7(g) and F,(g) at 1.00 atm and 298 K.

The enthalpy change in a chemical reaction is the difference between energy absorbed in breaking bonds in thereactants and energy released by bond formation in the products.

(d) How many bonds are formed when two molecules of NF3 are produced according to the equation in thebox above?

(e) Use both the information in the box above and the table of average bond enthalpies below to calculate theaverage enthalpy of the F — F bond.

BondAverage Bond Enthalpy

(kJ mor’)

NN 946

t—F 272

F-F

2007 The College Board. All rights reserved.Visit apcentraLcollegeboardcom (for AP professionals) and www.collegeboardcorn/apstudents (for students and parents).

GO ON TO THE NEXT PAGE.-7-

Handout Page 97 of 182 Handout Page 97 of 182

Handout Page 97 of 182 Updated 4/9/2014 Handout Page 97 of 182

Page 98: V1) I - Blue Valley Schools Chemistry/AP... · 3DJH RI 3DJH RI 2001 AP® CHEMISTRY FREE-RESPONSEQUESTIONS Answer EITHER Question 2 below OR Question 3 printed on page 8

2007 AP® CHEMISTRY FREE-RESPONSE QUESTIONS

Cu2q)+ 2 e Cu(s)

3. An external direct-current power supply is connected to two platinum electrodes immersed in a beakercontaining 1.0 M CuSO4(aq) at 25°C, as shown in the diagram above. As the cell operates, copper metal isdeposited onto one electrode and 0,(g) is produced at the other electrode. The two reduction half-reactions forthe overall reaction that occurs in the cell are shown in the table below.

Half-Reaction E°(V)

O,(g) + 4 H(aq) + 4 e —* 2 1120(1) + 1.23

Cu2(aq) + 2 e — Cu(s) +0.34

(a) On the diagram, indicate the direction of electron flow in the wire.

(b) Write a balanced net ionic equation for the electrolysis reaction that occurs in the cell.

(c) Predict the algebraic sign of AG° for the reaction. Justify your prediction.

(d) Calculate the value of AG° for the reaction.

An electric current of 1.50 amps passes through the cell for 40.0 minutes.

(e) Calculate the mass, in grams, of the Cu(s) that is deposited on the electrode.

(fi Calculate the dry volume, in liters measured at 25°C arid 1.16 atm, of the 0,(g) that is produced.

STOPIf you finish before time is called, you may check your work on this part only.

Do not turn to the other part of the test until you are told to do so.

© 2007 The College Board. All rights reserved.Visit apcentral.coliegeboard.com (for AP professionals) and www.collegeboard.com/apstudents (for students and parents).

-8-

Handout Page 98 of 182 Handout Page 98 of 182

Handout Page 98 of 182 Updated 4/9/2014 Handout Page 98 of 182

Page 99: V1) I - Blue Valley Schools Chemistry/AP... · 3DJH RI 3DJH RI 2001 AP® CHEMISTRY FREE-RESPONSEQUESTIONS Answer EITHER Question 2 below OR Question 3 printed on page 8

2007 AP® CHEMISTRY FREE-RESPONSE QUESTIONS

CHEMISTRYPart B

Time—40 minutesNO CALCULATORS MAY BE USED FOR PART B.

Answer Question 4 below. The Section II score weighting for this question is 10 percent.

4. For each of the following three reactions, in part (i) write a balanced equation for the reaction and in part (ii)answer the question about the reaction. In part (i), coefficients should be in terms of lowest whole numbers.Assume that solutions are aqueous unless otherwise indicated. Represent substances in solutions as ions if thesubstances are extensively ionized. Omit formulas for any ions or molecules that are unchanged by the reaction.You may use the empty space at the bottom of the next page for scratch work, but only equations that are writtenin the answer boxes provided will be graded.

EXAMPLE:

A strip of magnesium metal is added to a solution of silver(I) nitrate.

(i) Balanced equation:

M — ‘ zA

(ii) Which substance is oxidized in the reaction?

,. :ot4..

(a) A solution of sodium hydroxide is added to a solution of lead(II) nitrate.

(i) Balanced equation:

(ii) If 1.0 L volumes of 1.0 M solutions of sodium hydroxide and lead(II) nitrate are mixed together, howmany moles of product(s) will be produced? Assume the reaction goes to completion.

OO7 The College Board. All rights reserved.Visit apcentral.collegeboard.com (for AP professionals) and www.collegeboard.com/apstudents (for students and parents).

GO ON TO THE NEXT PAGE.-9-

Handout Page 99 of 182 Handout Page 99 of 182

Handout Page 99 of 182 Updated 4/9/2014 Handout Page 99 of 182

Page 100: V1) I - Blue Valley Schools Chemistry/AP... · 3DJH RI 3DJH RI 2001 AP® CHEMISTRY FREE-RESPONSEQUESTIONS Answer EITHER Question 2 below OR Question 3 printed on page 8

2007 AP® CHEMISTRY FREE-RESPONSE QUESTIONS

(b) Excess nitric acid is added to solid calcium carbonate.

(i) Balanced equation:

(ii) Briefly explain why statues made of marble (calcium carbonate) displayed outdoors in urban areas aredeteriorating.

(c) A solution containing silver(I) ion (an oxidizing agent) is mixed with a solution containing iron(II) ion(a reducing agent).

(i) Balanced equation:

(ii) If the contents of the reaction mixture described above are filtered, what substance(s), if any, wouldremain on the filter paper?

© 2007 The College Board. All rights reserved.Visit apcentraLcol1egeboardcom (for AP professionals) and www.collegeboarcLcom/apstudents (for students and parents).

GO ON TO THE NEXT PAGE.-10-

Handout Page 100 of 182 Handout Page 100 of 182

Handout Page 100 of 182 Updated 4/9/2014 Handout Page 100 of 182

Page 101: V1) I - Blue Valley Schools Chemistry/AP... · 3DJH RI 3DJH RI 2001 AP® CHEMISTRY FREE-RESPONSEQUESTIONS Answer EITHER Question 2 below OR Question 3 printed on page 8

2007 AP® CHEMISTRY FREE-RESPONSE QUESTIONS

Answer Question 5 and Question 6. The Section II score weighting for these questions is 15 percent each.

Your responses to these questions will be graded on the basis of the accuracy and relevance of the information cited.Explanations should be clear and well organized. Examples and equations may be included in your responses whereappropriate. Specific answers are preferable to broad, diffuse responses.

5 Fe2aq) + Mn04(aq) + 8 H(aq) —* 5 Fe3(aq) + Mn2(aq) + 4 H20(1)

5. The mass percent of iron in a soluble iron(ll) compound is measured using a titration based on the balancedequation above.

(a) What is the oxidation number of manganese in the permanganate ion, Mn04(aq)?

(b) Identify the reducing agent in the reaction represented above.

The mass of a sample of the iron(ll) compound is carefully measured before the sample is dissolved in distilledwater. The resulting solution is acidified with H,S04(aq). The solution is then titrated with MnO4(aq) untilthe end point is reached.

(c) Describe the color change that occurs in the flask when the end point of the titration has been reached.Explain why the color of the solution changes at the end point.

(d) Let the variables g, M, and V be defined as follows:

g = the mass, in grams, of the sample of the iron(ll) compound

M = the molarity of the Mn04(aq) used as the titrant

V = the volume, in liters, of MnO4(aq) added to reach the end point

In tenns of these variables, the number of moles of MnO4(aq) added to reach the end point of the titrationis expressed as M x V. Using the variables defined above, the molar mass of iron (55.85 g mo[), and thecoefficients in the balanced chemical equation, write the expression for each of the following quantities.

(i) The number of moles of iron in the sample

(ii) The mass of iron in the sample, in grams

(iii) The mass percent of iron in the compound

(e) What effect will adding too much titrant have on the experimentally determined value of the mass percent ofiron in the compound’? Justify your answer.

© 2007 The College Board 11 rights reserved.Visit apcentral.collegeboardcom (for AP professionals> and www.collegeboardcorn/apstuden ts (for students and parents).

GO ON TO THE NEXT PAGE.•-11-

Handout Page 101 of 182 Handout Page 101 of 182

Handout Page 101 of 182 Updated 4/9/2014 Handout Page 101 of 182

Page 102: V1) I - Blue Valley Schools Chemistry/AP... · 3DJH RI 3DJH RI 2001 AP® CHEMISTRY FREE-RESPONSEQUESTIONS Answer EITHER Question 2 below OR Question 3 printed on page 8

2007 AP® CHEMISTRY FREE-RESPONSE QUESTIONS

6. Answer the following questions, which pertain to binary compounds.

(a) In the box provided below, draw a complete Lewis electron-dot diagram for the IF3 molecule.

(b) On the basis of the Lewis electron-dot diagram that you drew in part (a), predict the molecular geometry ofthe IF3 molecule.

(c) In the SO, molecule, both of the bonds between sulfur and oxygen have the same length. Explain thisobservation, supporting your explanation by drawing in the box below a Lewis electron-dot diagram (ordiagrams) for the SO, molecule.

(d) On the basis of your Lewis electron-dot diagram(s) in part (c). identify the hybridization of the sulfur atomin the SO2 molecule.

The reaction between S02(g) and O,(g) to form S03(g) is represented below.

2 SO.,g) + O(g) 2 SO3(g)

The reaction is exothermic. The reaction is slow at 25°C; however, a catalyst will cause the reaction to proceedfaster.

(e) Using the axes provided on the next page, draw the complete potential-energy diagram for both thecatalyzed and uncatalyzed reactions. Clearly label the curve that represents the catalyzed reaction.

2007 The College Board. All rights reserved.Visit apcentral.collegehoardcom for AP professionals) and s vwcoilegeboardcomJapstsidents (for students and parents).

GO ON TO THE NEXT PAGE.-12-

Handout Page 102 of 182 Handout Page 102 of 182

Handout Page 102 of 182 Updated 4/9/2014 Handout Page 102 of 182

Page 103: V1) I - Blue Valley Schools Chemistry/AP... · 3DJH RI 3DJH RI 2001 AP® CHEMISTRY FREE-RESPONSEQUESTIONS Answer EITHER Question 2 below OR Question 3 printed on page 8

2007 AP CHEMISTRY FREE-RESPONSE QUESTIONS

>‘jj

Reaction Pro2ress

(f) Predict how the ratio of the equilibrium pressures, would change when the temperature of thePso3

uncatalyzed reaction mixture is increased. Justify your prediction.

(g) How would the presence of a catalyst affect the change in the ratio described in part (f)? Explain.

STOP

END OF EXAM

© 2007 The College Board. All rights reserved.Visit apcentraLcollegeboard.com (for AP professionals) and www.collegeboard.com/apstudents (for students and parents).

-13-

Handout Page 103 of 182 Handout Page 103 of 182

Handout Page 103 of 182 Updated 4/9/2014 Handout Page 103 of 182

Page 104: V1) I - Blue Valley Schools Chemistry/AP... · 3DJH RI 3DJH RI 2001 AP® CHEMISTRY FREE-RESPONSEQUESTIONS Answer EITHER Question 2 below OR Question 3 printed on page 8

2007 AP® CHEMISTRY FREE-RESPONSE QUESTIONS (Form B)

CHEMISTRYSection II

(Total time—95 minutes)

Part ATime—55 minutes

YOU MAY USE YOUR CALCULATOR FOR PART A.

CLEARLY SHOW THE METHOD USED AND THE STEPS INVOLVED IN ARRIVING AT YOUR ANSWERS.it is to your advantage to do this, since you may obtain partial credit if you do and you will receive little or no credit ifyou do not. Attention should be paid to significant figures.

Be sure to write all your answers to the questions on the lined pages following each question in the goldenrodbooklet. Do NOT write your answers on the lavender insert.

Answer Questions 1, 2, and 3. The Section II score weighting for each question is 20 percent.

1. A sample of solid U308 is placed in a rigid 1.500 L flask. Chlorine gas, Cl,(g), is added, and the flask is heatedto 862°C. The equation for the reaction that takes place and the equilibrium-constant expression for the reactionare given below.

U3O5(s) + 3 Cl,(g) 3 UO,Cl2(g) + O,(g) K,,= 22)3(Po2)

(Pci,)

When the system is at equilibrium, the partial pressure of C12(g) is 1.007 atm and the partial pressureof UO,C12(g) is 9.734 x iO4 atm.

(a) Calculate the partial pressure of 02(g) at equilibrium at 862°C.

(b) Calculate the value of the equilibrium constant, K,,, for the system at 862°C.

(c) Calculate the Gibbs free-energy change, AG°, for the reaction at 862°C.

(d) State whether the entropy change, AS°. for the reaction at 862°C is positive, negative, or zero. Justifyyour answer.

(e) State whether the enthalpy change, AK°, for the reaction at 862°C is positive, negative, or zero. Justifyyour answer.

(1) After a certain period of time, 1.000 mol of O,(g) is added to the mixture in the flask. Does the mass ofU308(s) in the flask increase, decrease, or remain the same? Justify your answer.

( 2007 The College Board, All rights reserved.Visit apcent-ral.collegeboard.com (for AP professionals) and www.collegeboard.com/apstudents (for students and parents).

GO ON TO THE NEXT PAGE.-6-

Handout Page 104 of 182 Handout Page 104 of 182

Handout Page 104 of 182 Updated 4/9/2014 Handout Page 104 of 182

Page 105: V1) I - Blue Valley Schools Chemistry/AP... · 3DJH RI 3DJH RI 2001 AP® CHEMISTRY FREE-RESPONSEQUESTIONS Answer EITHER Question 2 below OR Question 3 printed on page 8

2007 AP CHEMISTRY FREE-RESPONSE QUESTIONS (Form B)

2. Answer the following problems about gases.

(a) The average atomic mass of naturally occurring neon is 20.18 amu. There are two common isotopes ofnaturally occurring neon as indicated in the table below.

Isotope Mass (amu)

Ne-20 19.99

L Ne-22 21.99

(i) Using the information above, calculate the percent abundance of each isotope.

(ii) Calculate the number of Ne-22 atoms in a 12.55 g sample of naturally occurring neon.

(b) A major line in the emission spectrum of neon corresponds to a frequency of 4.34 x 1014 s1. Calculate thewavelength, in nanometers, of light that corresponds to this line.

(c) In the upper atmosphere, ozone molecules decompose as they absorb ultraviolet (UV) radiation, as shown bythe equation below. Ozone serves to block harmful ultraviolet radiation that comes from the Sun.

O3(g) ) 02(g) + 0(g)

A molecule of 03(g) absorbs a photon with a frequency of 1.00 x 10’3s’.

(i) How much energy, in joules, does the 03(g) molecule absorb per photon?

(ii) The minimum energy needed to break an oxygen-oxygen bond in ozone is 387 kJ mol* Does aphoton with a frequency of 1.00 x 1015 _l have enough energy to break this bond? Support youranswer with a calculation.

© 2007 The College Board. All rights reserved.Visit apcentraLcollegeboard.com (for AP professionals and www.collegeboard.com/apstudents (for students and parents).

GO ON TO THE NEXT PAGE.-7-

Handout Page 105 of 182 Handout Page 105 of 182

Handout Page 105 of 182 Updated 4/9/2014 Handout Page 105 of 182

Page 106: V1) I - Blue Valley Schools Chemistry/AP... · 3DJH RI 3DJH RI 2001 AP® CHEMISTRY FREE-RESPONSEQUESTIONS Answer EITHER Question 2 below OR Question 3 printed on page 8

2007 AP® CHEMISTRY FREE-RESPONSE QUESTIONS (Form B)

2 H,(g) + 07(g) —> 2 H20(1)

3. In a hydrogen-oxygen fuel cell, energy is produced by the overall reaction represented above.

(a) When the fuel cell operates at 25°C and 1.00 atm for 78.0 minutes, 0.0746 mol of O,(g) is consumed.Calculate the volume of H7(g) consumed during the same time period. Express your answer in litersmeasured at 25°C and 1.00 atm.

(b) Given that the fuel cell reaction takes place in an acidic medium,

(i) write the two half reactions that occur as the cell operates,

(ii) identify the half reaction that takes place at the cathode, and

(iii) determine the value of the standard potential, E °, of the cell.

(c) Calculate the charge, in coulombs, that passes through the cell during the 78.0 minutes of operation asdescribed in part (a).

STOPIf you finish before time is called, you may check your work on this part only.

Do not turn to the other part of the test until you are told to do so.

© 2007 The College Board, All rights reserved.Visit apcentral.collegeboard.com (for AP professionals) and www.collegeboard.com/apstudents (for students and parents).

-8-

Handout Page 106 of 182 Handout Page 106 of 182

Handout Page 106 of 182 Updated 4/9/2014 Handout Page 106 of 182

Page 107: V1) I - Blue Valley Schools Chemistry/AP... · 3DJH RI 3DJH RI 2001 AP® CHEMISTRY FREE-RESPONSEQUESTIONS Answer EITHER Question 2 below OR Question 3 printed on page 8

2007 AP CHEMISTRY FREE-RESPONSE QUESTIONS (Form B)

CHEMISTRYPart B

Time—40 minutesNO CALCULATORS MAY BE USED FOR PART B.

Answer Question 4 below. The Section II score weighting for this question is 10 percent.

4. For each of the following three reactions, in part (i) write a balanced equation for the reaction and in part (ii)answer the question about the reaction. In part (i), coefficients should be in terms of lowest whole numbers.Assume that solutions are aqueous unless otherwise indicated. Represent substances in solutions as ions if thesubstances are extensively ionized. Omit formulas for any ions or molecules that are unchanged by the reaction.You may use the empty space at the bottom of the next page for scratch work, but only equations that are writtenin the answer boxes provided will be graded.

EXAMPLE:

A strip of magnesium metal is added to a solution of silver(I) nitrate.

(i) Balanced equation;

(ii) Which substance is oxidized in the reaction?

bt

(a) Solid ammonium carbonate decomposes as it is heated.

(i) Balanced equation:

(ii) Predict the algebraic sign of S° for the reaction. Explain your reasoning.

© 2007 The College Board. All rights reserved.Visit apcen tral.collegeboard.com (for AP professionals) and www.collegeboard.com/apstudents (for students and parents).

GO ON TO THE NEXT PAGE.-9-

Handout Page 107 of 182 Handout Page 107 of 182

Handout Page 107 of 182 Updated 4/9/2014 Handout Page 107 of 182

Page 108: V1) I - Blue Valley Schools Chemistry/AP... · 3DJH RI 3DJH RI 2001 AP® CHEMISTRY FREE-RESPONSEQUESTIONS Answer EITHER Question 2 below OR Question 3 printed on page 8

2007 AP8 CHEMISTRY FREE-RESPONSE QUESTIONS (Form B)

(b) Chlorine gas, an oxidizing agent, is bubbled into a solution of potassium bromide.

(i) Balanced equation:

(ii) What is the oxidation number of chlorine before the reaction occurs? What is the oxidation number ofchlorine fir the reaction occurs?

(c) A small piece of sodium is placed in a beaker of distilled water.

(i) Balanced equation:

(ii) The reaction is exothermic, and sometimes small flames are observed as the sodium reacts with thewater. Identify the product of the reaction that bums to produce the flames.

© 2007 The College Board. All rights reserved.Visit apcentral.collegeboard.com (for AP professonalsJ and www.collegeboard.com/apstudents (for students and parents).

GO ON TO THE NEXT PAGE.-10-

Handout Page 108 of 182 Handout Page 108 of 182

Handout Page 108 of 182 Updated 4/9/2014 Handout Page 108 of 182

Page 109: V1) I - Blue Valley Schools Chemistry/AP... · 3DJH RI 3DJH RI 2001 AP® CHEMISTRY FREE-RESPONSEQUESTIONS Answer EITHER Question 2 below OR Question 3 printed on page 8

2007 AP® CHEMISTRY FREE-RESPONSE QUESTIONS (Form B)

Answer Question 5 and Question 6. The Section II score weighting for these questions is 15 percent each.

Your responses to these questions will be graded on the basis of the accuracy and relevance of the information cited.Explanations should be clear and well organized. Examples and equations may be included in your responses whereappropriate. Specific answers are preferable to broad, diffuse responses.

5. Answer the following questions about laboratory situations involving acids, bases, and buffer solutions.

(a) Lactic acid, HC3H503,reacts with water to produce an acidic solution. Shown below are thecomplete Lewis structures of the reactants.

H

H :0: :0:I I II

H—-C—C—-C—0—H + [1—0:I I I

H H K

In the space provided above, complete the equation by drawing the complete Lewis structures of thereaction products.

(b) Choosing from the chemicals and equipment listed below, describe how to prepare 100.00 mL of a 1.00 Maqueous solution of NH4C1 (molar mass 53.5 g mold). Include specific amounts and equipment whereappropriate.

NH4CI(s)

Distilled water

50 mL buret 100 mL graduated cylinder100 mL beaker 100 mL volumetric flask

100 mL pipet

Balance

(c) Two buffer solutions, each containing acetic acid and sodium acetate, are prepared. A student adds 0.10 molof HCI to 1.0 L of each of these buffer solutions and to 1.0 L of distilled water. The table below shows thepH measurements made before and after the 0.10 mol of HCI is added.

pH BeforeHC1 Added

pH AfterHCI Added

ti) ‘rite the balanced net-ionic equation for the reaction that takes place when the HCI is added tobuffer I or buffer 2.

(ii) Explain why the pH of buffer 1 is different from the pH of buffer 2 after 0.10 mol of 1-ICI is added.(iii) Explain why the pH of buffer I is the same as the pH of buffer 2 before 0.10 mol of HCI is added.

© OO7 The College Board. All rights reserved.Visit apcentraLcollegeboard.com (for AP professionals) and www.coflegeboard.com/apstudents (for students and parents>.

GO ON TO THE NEXT PAGE.

Distilled water 7.0 1.0

, Buffer I 4.7 2.7

Buffer 2 4.7 4.3

—f 1—

Handout Page 109 of 182 Handout Page 109 of 182

Handout Page 109 of 182 Updated 4/9/2014 Handout Page 109 of 182

Page 110: V1) I - Blue Valley Schools Chemistry/AP... · 3DJH RI 3DJH RI 2001 AP® CHEMISTRY FREE-RESPONSEQUESTIONS Answer EITHER Question 2 below OR Question 3 printed on page 8

2007 AP® CHEMISTRY FREE-RESPONSE QUESTIONS (Form B)

First Second ThirdIonization Energy Ionization Energy Ionization Energy

(kJ mo1) (kJ mo1) (kJ mo1’)

Element 1 1.251 2,300 3,820

Element 2 496 4,560 6,9 10

Element 3 738 1.450 7,730

Element 4 1,000 2,250 3,360

6. The table above shows the first three ionization energies for atoms of four elements from the third period of theperiodic table. The elements are numbered randomly. Use the information in the table to answer the followingquestions.

(a) Which element is most metallic in character? Explain your reasoning.

(b) Identify element 3. Explain your reasoning.

(c) Write the complete electron configuration for an atom of element 3.

(d) What is the expected oxidation state for the most common ion of element 2?

(e) What is the chemical symbol for element 2?

(f) A neutral atom of which of the four elements has the smallest radius?

STOP

END OF EXAM

‘ 2007 The College Board. All rights reserved.Visit apcenrral.coliegeboard.com (for AP professionais) and www.collegeboard.com/apstudenrs (for students and parents).

-12-

Handout Page 110 of 182 Handout Page 110 of 182

Handout Page 110 of 182 Updated 4/9/2014 Handout Page 110 of 182

Page 111: V1) I - Blue Valley Schools Chemistry/AP... · 3DJH RI 3DJH RI 2001 AP® CHEMISTRY FREE-RESPONSEQUESTIONS Answer EITHER Question 2 below OR Question 3 printed on page 8

2008 AP® CHEMISTRY FREE-RESPONSE QUESTIONS

CHEMISTRYSection 11

(Total time—95 minutes)

Part A

‘rinie—55 minutes

YOU MAY USE YOUR CALCULATOR FOR PART A.

CLEARLY SHOW THE METHOD USED AND THE STEPS INVOLVED IN ARRIVING AT YOUR ANSWERS.It is to your advantage to do this, since you may obtain partial credit if you do and you will receive little or no credit ifyou do not Attention should be paid to significant figures.

Be sure to write all your answers to the questions on the lined pages following each question in the booklet with thepink cover. Do NOT write your answers on the green insert.

Answer Questions 1. 2. and 3. The Section 11 score weighting for each question is 20 percent.

C(s) + CO,(g) 2 CO(g)

1. Solid carbon and carbon dioxide gas at 1,160 K were placed in a rigid 2.00 L container, and the reactionrepresented above occurred. As the reaction proceeded, the total pressure in the container was monitored.When equilibrium was reached, there was still some C(s) remaining in the container. Results are recordedin the table below.

. PressureofGasesfirne.in Container at 1,160 K

(atm)I

0.0 5.00

2.0 6.26

4.0 7.09

6.0 7.75

8.0 8.37—-

Li__

(a) Write the expression for the equilibrium constant. K1, . for the reaction.

(b) Calculate the number of moles of C02(g) initially placed in the container, (Assume that the volume of thesolid carbon is negligible.)

© 2008 The College Board. All rights reserved.Visit .spcentral.eollegehoard.corn for Al’ professionals and wsvw.eoiiegeboard.corn/apstudents for students and parents.

GO ON TO THE NEXT PAGE.-6-

Handout Page 111 of 182 Handout Page 111 of 182

Handout Page 111 of 182 Updated 4/9/2014 Handout Page 111 of 182

Page 112: V1) I - Blue Valley Schools Chemistry/AP... · 3DJH RI 3DJH RI 2001 AP® CHEMISTRY FREE-RESPONSEQUESTIONS Answer EITHER Question 2 below OR Question 3 printed on page 8

2008 AP® CHEMISTRY FREE-RESPONSE QUESTIONS

(c) For the reaction mixture at equilibrium at 1,160 K, the partial pressure of the C02g) is 1.63 atm. Calculate

(i) the partial pressure of CO(g). and

(ii) the value of the equilibrium constant, K.

(d) If a suitable solid catalyst were placed in the reaction vessel, would the final total pressure of the gases at

equilibrium be greater than, less than, or equal to the final total pressure of the gases at equilibrium withoutthe catalyst? Justify your answer. (Assume that the volume of the solid catalyst is negligible.)

In another experiment involving the same reaction, a rigid 2.00 L container initially contains 10.0 g of C(s).plus CO(g) and C01(g), each at a partial pressure of 2.00 atm at 1,160 K.

(e) Predict whether the partial pressure of CO,tg) will increase, decrease, or remain the same as this system

approaches equilibrium. Justify your prediction with a calculation.

© 2008 The College Board. All rights reserved.Visit apeentraleollegetoordeorn for AP professionals and wwwollegehoard.corn apoudents for students and parenr©.

GO ON TO THE NEXT PAGE.-7-

Handout Page 112 of 182 Handout Page 112 of 182

Handout Page 112 of 182 Updated 4/9/2014 Handout Page 112 of 182

Page 113: V1) I - Blue Valley Schools Chemistry/AP... · 3DJH RI 3DJH RI 2001 AP® CHEMISTRY FREE-RESPONSEQUESTIONS Answer EITHER Question 2 below OR Question 3 printed on page 8

2008 AP® CHEMISTRY FREE-RESPONSE QUESTIONS

2. Answer the flllowing questions relating to gravimetric analysis.

In the first of two experiments, a student is assigned the task of determining the number of moles of water in onemole of MgCI2 n H20. The student collects the data shown in the following table.

Mass of empty container 22.347 g

Initial mass of sample and container 25.825 g

Mass of sample and container after first heating 23.982 g

Mass of sample and container after second heating 23976 g

Mass of sample and container after third heating 23.977

(a) Explain why the student can correctly conclude that the hydrate was heated a sufficient number of times inthe experiment.

(b) Use the data above to

(i) calcuJ ate the total number of moles of water lost when the sample was heated, and

(ii) determine the formula of the hydrated compound.

(c) A different student heats the hydrate in an uncovered crucible, and some of the solid spatters out of thecrucible. This spattering will have what effect on the calculated mass of the water lost by the hydrate?Justify your answer.

In the second experiment, a student is given 2.94 g of a mixture containing anhydrous MgCI2 and KNO3.Todetermine the percentage by mass of MgCl, in the mixture, the student uses excess AgNO(aq) to precipitatethe chloride ion as AgChs).

(ci) Starting with the 2.93 g sample of the mixture dissolved in water, briefly describe the steps necessary toquantitatively determine the mass of the AgCl precipitate,

(e) The student determines the mass of the AgCI precipitate to be 5.48 g. On the basis of this information,calculate each of the following.

(i) The number of moles of MgCI2 in the original mixture

ii) The percent by mass of MgCl-, in the original mixture

20(J8 ‘Ihe College Board. All rights reserved.Visit apeentral.eollegehoard.eom for AP professionals) and wwwcollegeboardcomapstudents for students and parents

GO ON TO THE NEXT PAGE.-8-

Handout Page 113 of 182 Handout Page 113 of 182

Handout Page 113 of 182 Updated 4/9/2014 Handout Page 113 of 182

Page 114: V1) I - Blue Valley Schools Chemistry/AP... · 3DJH RI 3DJH RI 2001 AP® CHEMISTRY FREE-RESPONSEQUESTIONS Answer EITHER Question 2 below OR Question 3 printed on page 8

2008 AP® CHEMISTRY FREE-RESPONSE QUESTIONS

3. Answer the following questions related to chemical reactions involving nitrogen monoxide, NO(g).

The reaction between solid copper and nitric acid to form copper( ) ion, nitrogen monoxide gas. and water isrepresented by the following equation.

3 Cu(s) + 2 N03(aq) + 8 H(aq) — 3 Cu2(aq) + 2 N0g) + 4 H,O() = +t.62 V

(a) Using the information above and in the table below, calculate the standard reduction potential. E, for thereduction of NO3 in acidic solution.

Standard ReductionHalfReactionPotential. E°

Cu2(aq) + 2 e — Cu(s) +0.33 V

N03(aq) + 4 l-P(aq) + 3 e - — NO(g) + 2 H-,O(l)

(b) Calculate the value of the standard free energy change, 1G for the overall reaction between solid copperand nitric acid.

(c) Predict whether the value of the standard entropy change, zXS 0 for the overall reaction is greater than 0, lessthan 0. or equal to 0. Justify your prediction.

Nitrogen monoxide gas, a product of the reaction above, can react with oxygen to produce nitrogen dioxide gas.as represented below.

2 NO(g) + O2(g) —> 2 NO,(g)

© 2008 The College Board. All rights reserved.Visit apcentral.collegeboardcorn (for AP professionals) and www.collegehoard.com/apstudents (for students and parents).

GO ON TO THE NEXT PAGE.-9-

Handout Page 114 of 182 Handout Page 114 of 182

Handout Page 114 of 182 Updated 4/9/2014 Handout Page 114 of 182

Page 115: V1) I - Blue Valley Schools Chemistry/AP... · 3DJH RI 3DJH RI 2001 AP® CHEMISTRY FREE-RESPONSEQUESTIONS Answer EITHER Question 2 below OR Question 3 printed on page 8

2008 AP® CHEMISTRY FREERESPONSE QUESTIONS

A rate study of the reaction yielded the data recorded in the table below.

I . .Initial Rate ofInitial Concentration Initial Concentration .

Experiment Formation of N0201 NO (mol L) of 07 (mol L )

(mol L S

1 0.0200 0.0300 8.52 x l02

2 — 0.0200 0.0900 2.56xl0’

3 060O_j 7.67 x 10

(d) Determine the order of the reaction with respect to each of the following reactants. Give details of yourreasoning, clearly explaining or showing how you arrived at your answers.

(i) NO

(ii) 02

(e) Write the expression for the rate law for the reaction as determined from the experimental data.

(f) Determine the value of the rate constant for the reaction, clearly indicating the units.

STOPIf you finish before time is called, you may check your work on this part only.

Do not turn to the other part of the test until you are told to do so.

T 2008 The College Board, All rights reserved,Visit apcentralcollegeboard.com (for Al’ professiona1s and wwwcollegehoardcorn/apstudents (for students and parents).

-10-

Handout Page 115 of 182 Handout Page 115 of 182

Handout Page 115 of 182 Updated 4/9/2014 Handout Page 115 of 182

Page 116: V1) I - Blue Valley Schools Chemistry/AP... · 3DJH RI 3DJH RI 2001 AP® CHEMISTRY FREE-RESPONSEQUESTIONS Answer EITHER Question 2 below OR Question 3 printed on page 8

2008 AP® CHEMISTRY FREE-RESPONSE QUESTIONS

CHEMISTRY

Part B

Time—40 minutesNO CALCULATORS MAY BE USED FOR PART B.

Answer Question 4 below. The Section II score weighting for this question is 10 percent.

4. For each of the following three reactions, in part (i) write a balanced equation for the reaction and in part (ii)answer the question about the reaction, In part (i), coefficients should be in terms of lowest whole numbers.Assume that solutions are aqueous unless otherwise indicated. Represent substances in solutions as ions if thesubstances are extensively ionized. Omit formulas for any ions or molecules that arc unchanged by the reaction.You may use the empty space at the bottom of the next page for scratch work, but only equations that are writtenin the answer boxes provided will he graded.

EXAMPLE:

A strip of magnesium metal is added to a solution of silver(I) nitrate.

(i) Balanced equation:

A4i 2/ -, - 2.,4i%ç

(ii) Which substance is oxidized in the reaction?

A’tj

(a) Aqueous sodium hydroxide is added to a saturated solution of aluminum hydroxide. forming a complex ion.

(i) Balanced equation:

....

_

(ii) If the resulting mixture is acidified, would the concentration of the complex ion increase, decrease, orremain the same? Explain.

@ 2008 The College Board. All rights reserved,Visit apcentral.collegeboard.com (for AP professionals) and www.coflegehoard.com/apstudents (for students and parL’nts).

GO ON TO THE NEXT PAGE.—11—

Handout Page 116 of 182 Handout Page 116 of 182

Handout Page 116 of 182 Updated 4/9/2014 Handout Page 116 of 182

Page 117: V1) I - Blue Valley Schools Chemistry/AP... · 3DJH RI 3DJH RI 2001 AP® CHEMISTRY FREE-RESPONSEQUESTIONS Answer EITHER Question 2 below OR Question 3 printed on page 8

2008 AP® CHEMISTRY FREE-RESPONSE QUESTIONS

(b) Hydrogen chloride gas is oxidized by oxygen gas.

(i) Balanced equation:

ii) If three moles of hydrogen chloride gas and three moles of oxygen gas react as completely as possible.which reactant, if any, is present in excess? Justify your answer.

(c) Solid potassium oxide is added to water.

(i) Balanced equation:

(ii) If a few drops of phenolphthalein are added to the resulting solution, what would be observed?Explain.

YOU MAY USE THE SPACE BELOW FOR SCRATCH WORK, BUT ONLY EQUATIONSTHAT ARE WRITTEN IN THE ANSWER BOXES PROVIDED WILL BE GRADED.

2008 The College Board, All rights reserved,Visit apcentral.collegeboard.com (for AP professionals) and www.col1egeboardcom/apstudents (for studtnts and parents

GO ON TO THE NEXT PAGE.-12-

Handout Page 117 of 182 Handout Page 117 of 182

Handout Page 117 of 182 Updated 4/9/2014 Handout Page 117 of 182

Page 118: V1) I - Blue Valley Schools Chemistry/AP... · 3DJH RI 3DJH RI 2001 AP® CHEMISTRY FREE-RESPONSEQUESTIONS Answer EITHER Question 2 below OR Question 3 printed on page 8

2008 AP CHEMISTRY FREE-RESPONSE QUESTIONS

Answer Question 5 and Question 6. The Section II score weighting for these questions is 15 percent each.

Your responses to these questions will be graded on the basis of the accuracy and relevance of the information cited.Explanations should be clear and well organized. Examples and equations may be included in your responses whereappropriate. Specific answers are preferable to broad, diffuse responses.

5. Using principles of atomic and molecular structure and the information in the table below, answer the followingquestions about atomic fluorine, oxygen, and xenon, as well as some of their compounds.

AtomFirst Ionization Energy

(kJ mol )

F 1,681.0I_ Ul3.9

Xe

(a) Write the equation for the ionization of atomic fluorine that requires 1,681.0 kJ mol*

(b) Account for the fact that the first ionization energy of atomic fluorine is greater than that of atomic oxygen.(You must discuss bQth atoms in your response.)

(c) Predict whether the first ionization energy of atomic xenon is greater than, less than, or equal to the firstionization energy of atomic fluorine. Justify your prediction.

2008 The College Board. All rights reserved,Visit apcentralcollegehoard,com (for At’ professionals) and www.collegehoard.corn’apstudents (for students and parents).

GO ON TO THE NEXT PAGE.-13-

Handout Page 118 of 182 Handout Page 118 of 182

Handout Page 118 of 182 Updated 4/9/2014 Handout Page 118 of 182

Page 119: V1) I - Blue Valley Schools Chemistry/AP... · 3DJH RI 3DJH RI 2001 AP® CHEMISTRY FREE-RESPONSEQUESTIONS Answer EITHER Question 2 below OR Question 3 printed on page 8

2008 AP® CHEMISTRY FREE-RESPONSE QUESTIONS

(d) Xenon can react with oxygen and fluorine to form compounds such as Xe03 and XeF3. In the boxesprovided, draw the complete Lewis electron-dot diagram for each of the molecules represented below,

Xe03

(e) On the basis of the Lewis electron-dot diagrams you drew for part (d), predict the following:

(i) The geometric shape of the Xe03 molecule

(ii) The hybridization of the valence orbitals of xenon in XeF4

(0 Predict whether the Xe03 molecule is polar or nonpolar. Justify your prediction.

2008 The College Hoard, A]] rights reserved.\ isO apLeittra].collcgebuard.eonl for AP professionals and www.collegehoardconiiapstudents for students and parents

GO ON TO THE NEXT PAGE.-14-

Handout Page 119 of 182 Handout Page 119 of 182

Handout Page 119 of 182 Updated 4/9/2014 Handout Page 119 of 182

Page 120: V1) I - Blue Valley Schools Chemistry/AP... · 3DJH RI 3DJH RI 2001 AP® CHEMISTRY FREE-RESPONSEQUESTIONS Answer EITHER Question 2 below OR Question 3 printed on page 8

2008 AP® CHEMISTRY FREE-RESPONSE QUESTIONS

6. Answer the following questions by using principles of molecular structure and intermolecular forces.

(a) Structures of the pvridine molecule and the benzene molecule are shown below. Pyridine is soluble in water.whereas benzene is not soluble in water. Account for the difference in solubility. You must discuss kcgh ofthe substances in your answer.

H H H K\ / \, ,/I/(\\ //C

C—H H—C C—il\ / \ /C=C Cz=C/ / \

H 11 II H

Pyridine llenzene

(b) Structures of the dirnethyl ether molecule and the ethanol molecule are shown below. The normal boilingpoint of dimethyl ether is 250 K, whereas the normal boiling point of ethanol is 351 K. Account for thedifference in boiling points. You must discuss th of the substances in your answer.

H H H HI I

H—C—O—C—H H—C—C —0—li

H H [I H

l)iineihvl I ther ltlianol

(c) SO, melts at 201 K, whereas SiO, melts at 1,883 K. Account for the difference in melting points. Youmust discuss kth of the substances in your answer.

(d) The normal boiling point of Cl,(1) (238 K) is higher than the normal boiling point of HCI(l) (188 K.Account for the difference in normal boiling points based on the types of intermolecular forces in thesubstances. You must discuss bQth of the substances in your answer.

STOP

END OF EXAM

© 2008 The College Board. All rights reserved.Visit apcentral.collegeboard.com for AP professionals and www.collegeboard.comlapstudents for students and parents).

-15-

Handout Page 120 of 182 Handout Page 120 of 182

Handout Page 120 of 182 Updated 4/9/2014 Handout Page 120 of 182

Page 121: V1) I - Blue Valley Schools Chemistry/AP... · 3DJH RI 3DJH RI 2001 AP® CHEMISTRY FREE-RESPONSEQUESTIONS Answer EITHER Question 2 below OR Question 3 printed on page 8

2008 AP® CHEMISTRY FREE-RESPONSE QUESTIONS (Form B)

CHEMISTRYSection II

(Total time—95 minutes)

Part A

Time—55 minutesYOU MAY USE YOUR CALCULATOR FOR PART A.

CLEARLY SHOW THE METHOD USED AND THE STEPS INVOLVED IN ARRIVING AT YOUR ANSWERS.It is to your advantage to do this, since you may obtain partial credit if you do and you will receive little or no credit ifyou do not. Attention should be paid to significant figures.

Be sure to write all your answers to the questions on the lined pages following each question in the goldenrodbooklet. Do NOT write your answers on the lavender insert.

Answer Questions 1, 2, and 3. The Section II score weighting for each question is 20 percent.

1. Answer the following questions regarding the decomposition of arsenic pentafluoride, A5F5(g).

(a) A 55.8 g sample of AsF5(g) is introduced into an evacuated 10.5 L container at 105°C.

(i) What is the initial molar concentration of AsF5(g) in the container?

(ii) What is the initial pressure, in atmospheres, of the AsF5(g) in the container?

At 105°C, AsF5(g) decomposes into AsF3(g) and F,(g) according to the following chemical equation.

A5F5(g) A5F3(g) + F2(g)

(b) In terms of molar concentrations, write the equilibrium-constant expression for the decomposition ofAsF5(g).

(c) When equilibrium is established, 27.7 percent of the original number of moles of AsF5(g) has decomposed.

(i) Calculate the molar concentration of AsF5(g) at equilibrium.

(ii) Using molar concentrations, calculate the value of the equilibrium constant, Keq at 105°C.

(d> Calculate the mole fraction of F1(g in the container at equilibrium.

© 2008 The College Board. All rights reserved.Visit apcentral.collegeboard.com (for AP professionals) and www.collegeboard.com/apstudents (for students and parents).

GO ON TO THE NEXT PAGE.-6-

Handout Page 121 of 182 Handout Page 121 of 182

Handout Page 121 of 182 Updated 4/9/2014 Handout Page 121 of 182

Page 122: V1) I - Blue Valley Schools Chemistry/AP... · 3DJH RI 3DJH RI 2001 AP® CHEMISTRY FREE-RESPONSEQUESTIONS Answer EITHER Question 2 below OR Question 3 printed on page 8

2008 AP® CHEMISTRY FREE-RESPONSE QUESTIONS (Form B)

A(g) + B(g) —* C(g) + D(g)

2. For the gas-phase reaction represented above, the following experimental data were obtained.

. Initial [A] Initial [B] Initial Reaction RateExperiment

(mol L-’) (mol L) (mol L s-’)

1 0.033 0.034 6.67 x 10

2 0.034 0.137 l.08x 10-2

3 0.136 0.136 1.07 x 10-2

4 0.202 0.233 ?

(a) Determine the order of the reaction with respect to reactant A. Justify your answer.

(b) Determine the order of the reaction with respect to reactant B . Justify your answer.

(c) Write the rate law for the overall reaction.

(d) Determine the value of the rate constant, k, for the reaction. Include units with your answer.

(e) Calculate the initial reaction rate for experiment 4.

(1) The following mechanism has been proposed for the reaction.

Step 1: B + B — E + D slow

Step 2: E + A B + C fast equilibrium

Provide two reasons why the mechanism is acceptable.

(g) In the mechanism in part (I), is species E a catalyst, or is it an intermediate? Justify your answer.

© 2008 The College Board. All rights reserved.Visit apcentral.collegeboard.com (for AP professionals) and www.collegeboard.com/apstudents (for students and parents).

GO ON TO THE NEXT PAGE.-7-

Handout Page 122 of 182 Handout Page 122 of 182

Handout Page 122 of 182 Updated 4/9/2014 Handout Page 122 of 182

Page 123: V1) I - Blue Valley Schools Chemistry/AP... · 3DJH RI 3DJH RI 2001 AP® CHEMISTRY FREE-RESPONSEQUESTIONS Answer EITHER Question 2 below OR Question 3 printed on page 8

2008 AP® CHEMISTRY FREE-RESPONSE QUESTIONS (Form B)

3. A 0.150 g sample of solid lead(I1) nitrate is added to 125 mL of 0.100 M sodium iodide solution. Assume nochange in volume of the solution. The chemical reaction that takes place is represented by the followingequation.

Pb(N03)2(s) + 2 Nal(aq) — Pb12(s) + 2 NaNO3(aq)

(a) List an appropriate observation that provides evidence of a chemical reaction between the two compounds.

(b) Calculate the number of moles of each reactant.

Ic) Identify the limiting reactant. Show calculations to support your identification.

(d) Calculate the molar concentration of N03(aq) in the mixture after the reaction is complete.

(e) Circle the diagram below that best represents the results after the mixture reacts as completely as possible.Explain the reasoning used in making your choice.

/.

Solid Phi.

eSolid Phk

0

Solid Ph(NO.,

2008 The College Board. All rights reserved.Visit apcentralcollegeboardcom (for AP professionak and www.collcgeboard.com/apstudents (ftr students and parents.

GO ON TO THE NEXT PAGE.

ED0

No Precipitate Solid Phi,

I

0 0

-8-

Handout Page 123 of 182 Handout Page 123 of 182

Handout Page 123 of 182 Updated 4/9/2014 Handout Page 123 of 182

Page 124: V1) I - Blue Valley Schools Chemistry/AP... · 3DJH RI 3DJH RI 2001 AP® CHEMISTRY FREE-RESPONSEQUESTIONS Answer EITHER Question 2 below OR Question 3 printed on page 8

2008 AP CHEMISTRY FREE-RESPONSE QUESTIONS (Form B)

STOPIf you finish before time is called, you may check your work on this part only.

Do not turn to the other part of the test until you are told to do so.

2008 The College Board, All rights reserved.Visit apcentral.collegeboard.com (for AP professionals) and www.collegeboard.com/apstudents (for students and parents).

-9-

Handout Page 124 of 182 Handout Page 124 of 182

Handout Page 124 of 182 Updated 4/9/2014 Handout Page 124 of 182

Page 125: V1) I - Blue Valley Schools Chemistry/AP... · 3DJH RI 3DJH RI 2001 AP® CHEMISTRY FREE-RESPONSEQUESTIONS Answer EITHER Question 2 below OR Question 3 printed on page 8

2008 AP® CHEMISTRY FREE-RESPONSE QUESTIONS (Form B)

CHEMISTRYPart B

Time—40 minutesNO CALCULATORS MAY BE USED FOR PART B.

Answer Question 4 below. The Section II score weighting for this question is 10 percent.

4. For each of the following three reactions, in part (i) write a balanced equation for the reaction and in part (ii)answer the question about the reaction. In part (i), coefficients should be in terms of lowest whole numbers.Assume that solutions are aqueous unless otherwise indicated. Represent substances in solutions as ions if thesubstances are extensively ionized. Omit formulas for any ions or molecules that are unchanged by the reaction.You may use the empty space at the bottom of the next page for scratch work, but only equations that are writtenin the answer boxes provided will be graded.

BXAPLE:

A strip of magnesium metal is added to a solution of silver(l) nitrate.

(I) Balanced equation:

Mgi- —A4 zA

(ii) Which substance is oxidized in the reaction?

.

(a) Chlorine gas, an oxidizing agent, is bubbled into a solution of potassium bromide at 25°C.

(i) Balanced equation:

(ii) Predict the sign of AS° for the reaction at 25°C. Justify your prediction.

2008 The College Board. All rights reserved.Visit apcentral.collegeboard.com (for AP professionals) and wwwcollegeboard.comlapstudents (for students and parents).

GO ON TO THE NEXT PAGE.-10-

Handout Page 125 of 182 Handout Page 125 of 182

Handout Page 125 of 182 Updated 4/9/2014 Handout Page 125 of 182

Page 126: V1) I - Blue Valley Schools Chemistry/AP... · 3DJH RI 3DJH RI 2001 AP® CHEMISTRY FREE-RESPONSEQUESTIONS Answer EITHER Question 2 below OR Question 3 printed on page 8

2008 AP CHEMISTRY FREE-RESPONSE QUESTIONS (Form B)

(b) Solid strontium hydroxide is added to a solution of nitric acid.

(i) Balanced equation:

(ii) How many moles of strontium hydroxide would react completely with 500. mL of 0.40 M nitric acid?

(c) A solution of barium chloride is added drop by drop to a solution of sodium carbonate, causing a precipitateto form.

(i) Balanced equation:

(ii) What happens to the pH of the sodium carbonate solution as the barium chloride is added to it?

© 2008 The College Board. All rights reserved.Visit apcentraLcollegeboarcLcom (for AP professionals) and wwwcollegeboardcom/apstudents (for students and parents).

GO ON TO THE NEXT PAGE.-11-

Handout Page 126 of 182 Handout Page 126 of 182

Handout Page 126 of 182 Updated 4/9/2014 Handout Page 126 of 182

Page 127: V1) I - Blue Valley Schools Chemistry/AP... · 3DJH RI 3DJH RI 2001 AP® CHEMISTRY FREE-RESPONSEQUESTIONS Answer EITHER Question 2 below OR Question 3 printed on page 8

2008 AP® CHEMISTRY FREE-RESPONSE QUESTIONS (Form B)

Answer Question 5 and Question 6. The Section 11 score weighting for these questions is 15 percent each.

Your responses to these questions will be graded on the basis of the accuracy and relevance of the information cited.Explanations should be clear and well organized. Examples and equations may be included in your responses whereappropriate. Specific answers are preferable to broad, diffuse responses.

5. The identity of an unknown solid is to be determined. The compound is one of the seven salts in the followingtable.

Use the results of the following observations or laboratory tests to explain how each compound in the table maybe eliminated or confirmed. The tests are done in sequence from (a) through (e).

(a) The unknown compound is white. In the table below, cross out the two compounds that can be eliminatedusing this observation. Be sure to cross out these same two compounds in the tables in parts (b), (c), and (d).

Al(NO) 9H20 BaC12 2H20 CaCO3 CuSO4 5H20

NaC1 BaSO4 Ni(N03)2’6H70

(b) When the unknown compound is added to water, it dissolves readily. in the table below, cross out the twocompounds that can be eliminated using this test. Be sure to cross out these same two compounds in thetables in parts (c) and (d).

Al(NO3)9H10 BaCl2 2HO CaCO CuSO35H,O

(c) When AgNO3(aq) is added to an aqueous solution of the unknown compound, a white precipitate forms. Inthe table below, cross out each compound that can be eliminated using this test. Be sure to cross out thesame compound(s) in the table in part (d).

© 2008 The College Board. All rights reserved.Visit apcentral.collegeboard.corn (for AP professionals and www.collegeboard.com/apstudents (for students and parents).

GO ON TO THE NEXT PAGE.

NaC1 BaSO4 Ni(N03)26H20

Al(NO)3’9H,O BaCI2 2H20

NaCl f CaCO3

BaSO4 Ni(NO)16H20

CuSO35H,O

-12-

Handout Page 127 of 182 Handout Page 127 of 182

Handout Page 127 of 182 Updated 4/9/2014 Handout Page 127 of 182

Page 128: V1) I - Blue Valley Schools Chemistry/AP... · 3DJH RI 3DJH RI 2001 AP® CHEMISTRY FREE-RESPONSEQUESTIONS Answer EITHER Question 2 below OR Question 3 printed on page 8

2008 AP CHEMISTRY FREE-RESPONSE QUESTIONS (Form B)

(d) When the unknown compound is carefully heated, it loses mass. In the table below, cross out eachcompound that can be eliminated using this test.

Al(N03)3 9H20 BaCI2 2H20 CaCO3 CuSO4 5H20

NaCI BaSO4 Ni(N03)2 6H20

(e) Describe a test that can be used to confirm the identity of the unknown compound identified in part (d).Limit your confirmation test to a reaction between an aqueous solution of the unknown compound and anaqueous solution of one of the other soluble salts listed in the tables. Describe the expected results of thetest; include the formula(s) of any product(s).

© 2008 The College Board. All rights reserved.Visit apcentral.collegeboard.com tot AP prolessionats) and www.collegeboard.com/apstudents (tor students and parents).

GO ON TO THE NEXT PAGE.-13-

Handout Page 128 of 182 Handout Page 128 of 182

Handout Page 128 of 182 Updated 4/9/2014 Handout Page 128 of 182

Page 129: V1) I - Blue Valley Schools Chemistry/AP... · 3DJH RI 3DJH RI 2001 AP® CHEMISTRY FREE-RESPONSEQUESTIONS Answer EITHER Question 2 below OR Question 3 printed on page 8

2008 AP® CHEMISTRY FREE-RESPONSE QUESTIONS (Form B)

6. Use principles of thermodynamics to answer the following questions.

(a) The gas N,04 decomposes to form the gas NO2 according to the equation below.

0 0/

N—N/

() ()

() 0/

N’ + N

(i) Predict the sign of All° for the reaction. Justify your answer.

(ii) Predict the sign of AS° for the reaction. Justify your answer.

(b) One of the diagrams below best represents the relationship between AG° and temperature for the reactiongiven in part (a). Assume that MI° and S° are independent of temperature.

Draw a circle around the correct graph. Explain why you chose that graph in terms of the relationshipAG° = AH°-TAS°.

(c) A reaction mixture of N104 and NO2 is at equilibrium. Heat is added to the mixture while the mixture ismaintained at constant pressure.

(i) Explain why the concentration of N2O4 decreases.

(ii. The value of Keq at 25°C is 5.0 x 10* Will the value of K(q at 100°C be greater than, less than, orequal to this value?

(d) Using the value of Keq at 25°C given in part (c)(ii), predict whether the value of JJO is expected to begreater than, less than, or equal to the value of ThS°. Explain.

STOP

END OF EXAM

© 2008 The College Board. All rights reserved.Visit apcentral.collegeboard.com (for AP professionals) and www.collegeboard.com/apstudents (for students and parents).

T T T

-14-

Handout Page 129 of 182 Handout Page 129 of 182

Handout Page 129 of 182 Updated 4/9/2014 Handout Page 129 of 182

Page 130: V1) I - Blue Valley Schools Chemistry/AP... · 3DJH RI 3DJH RI 2001 AP® CHEMISTRY FREE-RESPONSEQUESTIONS Answer EITHER Question 2 below OR Question 3 printed on page 8

2009 AP® CHEMISTRY FREE-RESPONSE QUESTIONS

CHEMISTRY

Section II(Total time—9$ minutes)

Part A

Time—55 minutesYOU MAY USE YOUR CALCULATOR FOR PART A.

CLEARLY SHOW THE METhOD USED AND THE STEPS INVOLVED IN ARRiVING AT YOUR ANSWERS.It is to your advantage to do this, since you may obtain partial credit if you do and you will receive little or no credit ifyou do not. Attention should be paid to significant tigures.

Be sure to write all your answers to the questions on the lined pages following each question in the booklet with thepink cover. Do NOT write your answers on the green insert.

Answer Questions 1, 2, and 3. The Section II score weighting for each question is 20 percent.

I. Answer the following questions that relate to the chemistry of halogen oxoacids.

(a) Use the information in the table below to answer part (a)(i).

Acid K0 at 298 K

HOCI 2.9 x 108

HOBr 2.4 x

(i) Which of the two acids is stronger, HOCI or HOBr ? Justify your answer in terms of K1.

(ii Draw a complete Lewis electron-dot diagram for the acid that you identified in part (a)(i).

(iii) Hypoiodous acid has the formula HO1. Predict whether 1-10! is a stronger acid or a weaker acid thanthe acid that you identified in part (a)(i). Justify your prediction in terms of chemical bonding.

(h) Write the equation for the reaction that occurs between hypochiorous acid and water.

(c) A 1 .2 M NaOC1 solution is prepared by dissolving solid NaOCl in distilled water at 298 K. The hydrolysisreaction 0C1(aq) + H7O(l) HOCI(aq) + 0H(aq) occurs.

i) Write the equilibrium-constant expression for the hydrolysis reaction that occurs between OClaq)and H,0(1).

(ii Calculate the value of the equilibrium constant at 298 K for the hydrolysis reaction.

(iii) Calculate the value of [OHJ in the 1.2 M NaOC1 solution at 298 K.

© 2009 The College Board. All rights reserved.Visit the College Board on the Web: wwwcollegeboardcorn.

-6- GO ON TO THE NEXT PAGE.

Handout Page 130 of 182 Handout Page 130 of 182

Handout Page 130 of 182 Updated 4/9/2014 Handout Page 130 of 182

Page 131: V1) I - Blue Valley Schools Chemistry/AP... · 3DJH RI 3DJH RI 2001 AP® CHEMISTRY FREE-RESPONSEQUESTIONS Answer EITHER Question 2 below OR Question 3 printed on page 8

2009 AP0 CHEMISTRY FREE-RESPONSE QUESTIONS

(d) A buffer solution is prepared by dissolving some solid NaOCl in a solution of HOC! at 298 K. The pH ofthe buffer solution is determined to be 6.48.

(i) Calculate the value of [H3O1 in the buffer solution.

(ii) Indicate which of HOCI(aq) or OCl(aq) is present at the higher concentration in the buffer solution.Support your answer with a calculation.

2. A student was assigned the task of determining the molar mass of an unknown gas. The student measured themass of a sealed 843 mL rigid flask that contained dry air. The student then flushed the flask with the unknowngas, resealed it, and measured the mass again. Both the air and the unknown gas were at 23.0°C and 750. torr.The data for the experiment are shown in the table below.

Volume of sealed flask 843 mL

Mass of sealed flask and dry air 157.70 g

Mass of sealed flask and unknown gas 158.08 g

(a) Calculate the mass, in grams, of the dry air that was in the sealed flask. (The density of dry air is I .18 gat 23.0°C and 750. torr.)

(b) Calculate the mass, in grams, of the sealed flask itself (i.e., if it had no air in it).

(c) Calculate the mass, in grams, of the unknown gas that was added to the sealed flask.

(d) Using the information above, calculate the value of the molar mass of the unknown gas.

After the experiment was completed, the instructor informed the student that the unknown gas was carbondioxide (44.0 g mol’).

(e) Calculate the percent error in the value of the molar mass calculated in part (d).

(0 For each of the following two possible occurrences, indicate whether it by itself could have been responsiblefor the error in the student’s experimental result. You need not include any calculations with your answer.For each of the possible occurrences, justify your answer.

Occurrence 1: The flask was incompletely flushed with C02(g), resulting in some dry air remainingin the flask.

Occurrence 2: The temperature of the air was23.0°C. but the temperature of the CO,(g) was lower thanthe reported 23.0°C,

(gi Describe the steps of a laboratory method that the student could use to verify that the volume of the rigidflask is 843 mL at 23.0°C. You need not include any calculations with your answer.

© 2009 The College Board. All rights reserved.Visit the College Board on the Web: www.collegeboard.com.

-7- GO ON TO THE NEXT PAGE.

Handout Page 131 of 182 Handout Page 131 of 182

Handout Page 131 of 182 Updated 4/9/2014 Handout Page 131 of 182

Page 132: V1) I - Blue Valley Schools Chemistry/AP... · 3DJH RI 3DJH RI 2001 AP® CHEMISTRY FREE-RESPONSEQUESTIONS Answer EITHER Question 2 below OR Question 3 printed on page 8

2009 AP CHEMISTRY FREE-RESPONSE QUESTIONS

CH4(g) + 2 C1,(g) —* CH2C12(g) + 2 HCI(g)

3. Methane gas reacts with chlorine gas to form dichioromethane and hydrogen chloride, as represented by theequation above.

(a) A 25.0 g sample of methane gas is placed in a reaction vessel containing 2.58 mol of Cl2g).

(i) Identify the limiting reactant when the methane and chlorine gases are combined, Justify your answerwith a calculation.

(ii) Calculate the total number of moles of CH,Cl2(g) in the container after the limiting reactant has beentotally consumed.

Initiating most reactions involving chlorine gas involves breaking the Cl—Cl bond, which has a bond energyof 242 kJ mol’.

(b) Calculate the amount of energy, in joules, needed to break a single Cl—Cl bond,

(c) Calculate the longest wavelength of light, in meters, that can supply the energy per photon necessary tobreak the Cl—Cl bond.

The following mechanism has been proposed for the reaction of methane gas with chlorine gas. All species arein the gas phase.

Step I Cl, 2 Cl fast equilibrium

Step 2 CH4 + Cl —÷ CH3 + HCI slow

Step 3 CH3 + Cl2 — CHCI + Cl fist

Step 4 CH3C1 + Cl —* CH2CI, + H fast

Step 5 H + Cl —* HCI fist

(d) In the mechanism, is CH3CI a catalyst, or is it an intermediate? Justify your answer.

(e) Identify the order of the reaction with respect to each of the following according to the mechanism. In eachcase, justify your answer.

(i) CH3(g)

(ii) Cl,(g)

STOPIf you finish before time is called, you may check your work on this part only.

Do not turn to the other part of the test until you are told to do so.

© 2009 The College Board. All rights reserved.Visit the College Board on the Web: www.collegeboard.com.

-8-

Handout Page 132 of 182 Handout Page 132 of 182

Handout Page 132 of 182 Updated 4/9/2014 Handout Page 132 of 182

Page 133: V1) I - Blue Valley Schools Chemistry/AP... · 3DJH RI 3DJH RI 2001 AP® CHEMISTRY FREE-RESPONSEQUESTIONS Answer EITHER Question 2 below OR Question 3 printed on page 8

2009 AP® CHEMISTRY FREE-RESPONSE QUESTIONS

CHEMISTRY

Part BTime—40 minutes

NO CALCULATORS MAY BE USED FOR PART B.

Answer Question 4 below. The Section II score weighting for this question is 10 percent.

4. For each of the following three reactions, write a balanced equation in part (i) and answer the question inpart (ii). In part (i), coefficients should be in terms of lowest whole numbers. Assume that solutions are aqueousunless otherwise indicated. Represent substances in solutions as ions if the substances are extensively ionized.Omit formulas for any ions or molecules that are unchanged by the reaction. You may use the empty space at thebottom of the next page for scratch work, but only equations that are written in the answer boxes provided willbe graded.

EXAMPLE:

A strip of magnesium metal is added to a solution of silver(I) nitrate.

(i) Balanced equation:

M 2,/.t .,

i- aA

(ii) Which substance is oxidized in the reaction’?

bt

(a) A sample of solid iron(III) oxide is reduced completely with solid carbon.

(i) Balanced equation:

(ii) What is the oxidation number of carbon before the reaction, and what is the oxidation number ofcarbon after the reaction is complete?

© 2009 The College Board. All rights reserved.Visit the College Board on the Web: www.collegeboard.com.

-9- GO ON TO THE NEXT PAGE.

Handout Page 133 of 182 Handout Page 133 of 182

Handout Page 133 of 182 Updated 4/9/2014 Handout Page 133 of 182

Page 134: V1) I - Blue Valley Schools Chemistry/AP... · 3DJH RI 3DJH RI 2001 AP® CHEMISTRY FREE-RESPONSEQUESTIONS Answer EITHER Question 2 below OR Question 3 printed on page 8

2009 AP® CHEMISTRY FREE-RESPONSE QUESTIONS

(b) Equal volumes of equimolar solutions of ammonia and hydrochloric acid are combined.

(i) Balanced equation:

(ii) Indicate whether the resulting solution is acidic, basic, or neutral. Explain.

(c) Solid mercury(Il) oxide decomposes as it is heated in an open test tube in a fume hood.

(i) Balanced equation:

(ii) After the reaction is complete, is the mass of the material in the test tube greater than, less than, orequal to the mass of the original sample? Explain.

YOU MAY USE THE SPACE BELOW FOR SCRATCH WORK, BUT ONLY EQUATIONSTHAT ARE WRITTEN IN THE ANSWER BOXES PROVIDED WILL BE GRADED.

© 2009 The College Board. All rights reserved.Visit the College Board on the Web: www.collegeboard.com.

-10- GO ON TO THE NEXT PAGE.

Handout Page 134 of 182 Handout Page 134 of 182

Handout Page 134 of 182 Updated 4/9/2014 Handout Page 134 of 182

Page 135: V1) I - Blue Valley Schools Chemistry/AP... · 3DJH RI 3DJH RI 2001 AP® CHEMISTRY FREE-RESPONSEQUESTIONS Answer EITHER Question 2 below OR Question 3 printed on page 8

2009 AP® CHEMISTRY FREE-RESPONSE QUESTIONS

Answer Question 5 and Question 6. The Section II score weighting for these questions is 15 percent each.

Your responses to these questions will be graded on the basis of the accuracy and relevance of the information cited.Explanations should be clear and well organized. Examples and equations may be included in your responses whereappropriate. Specific answers are preferable to broad, diffuse responses.

Reaction Equation } M1, f LS tXG8

X C(s) +H10(g) CO(g) + H.,(g) +131 kJ mo1 +134 J mold K-’ +91 kJ mol

Y C02(g) + H,(g) CO(g) + 1-120(g) +41 kJ mo11 ÷42 J mol’ K-’ +29 kJ moI’

Z 2 CO(g) C(s) + C02(g) ? ? ?

5. Answer the following questions using the information related to reactions X, Y, and Z in the table above.

(a) For reaction X, write the expression for the equilibrium constant, K,,.

(b) For reaction X, will the equilibrium constant, K,,. increase, decrease, or remain the same if the temperaturerises above 298 K? Justify your answer.

(c) For reaction Y at 298 K, is the value of K, greater than 1. less than 1, or equal to 1? Justify your answer.

(d) For reaction Y at 298 K, which is larger: the total bond energy of the reactants or the total bond energy ofthe products? Explain.

(e) Is the following statement true or false? Justify your answer.

“On the basis of the data in the table, it can be predicted that reaction Y will occur more rapidly thanreaction X will occur.”

(f) Consider reaction Z at 298 K.

(1) Is AS° for the reaction positive, negative, or zero? Justify your answer.

(ii) Determine the value of AU° for the reaction.

(iii) A sealed glass reaction vessel contains only CO(g) and a small amount of C(s). If a reaction occursand the temperature is held constant at 298 K, will the pressure in the reaction vessel increase,decrease, or remain the same over time? Explain.

© 2009 The College Board. All rights reserved.Visit the College Board on the Web: www.coHegeboard.com.

-11- GO ON TO THE NEXT PAGE.

Handout Page 135 of 182 Handout Page 135 of 182

Handout Page 135 of 182 Updated 4/9/2014 Handout Page 135 of 182

Page 136: V1) I - Blue Valley Schools Chemistry/AP... · 3DJH RI 3DJH RI 2001 AP® CHEMISTRY FREE-RESPONSEQUESTIONS Answer EITHER Question 2 below OR Question 3 printed on page 8

2009 AP® CHEMISTRY FREE-RESPONSE QUESTIONS

6. Answer the following questions related to sulfur and one of its compounds.

(a) Consider the two chemical species S and S2

(i) Write the electron configuration (e.g., ls2 2s2. ..) of each species.

(ii) Explain why the radius of the S2 ion is larger than the radius of the S atom.

(iii) Which of the two species would be attracted into a magnetic field? Explain.

(b) The S2 ion is isoelectronic with the Ar atom. From which species, S2 or Ar, is it easier to remove anelectron? Explain.

(c) In the H,S molecule, the H—S—H bond angle is close to 900. On the basis of this information, which atomicorbitals of the S atom are involved in bonding with the H atoms?

(d) Two types of intermolecular forces present in liquid H2S are London (dispersion) forces and dipole-dipoleforces.

(i) Compare the strength of the London (dispersion) forces in liquid H,S to the strength of the London(dispersion) forces in liquid H,O. Explain.

(ii) Compare the strength of the dipole-dipole forces in liquid H,S to the strength of the dipole-dipoleforces in liquid H20. Explain.

STOP

END OF EXAM

2009 The College Board. Mi rights reserved.Visit the College Board on the Web; www.colJegeboard.com.

-12-

Handout Page 136 of 182 Handout Page 136 of 182

Handout Page 136 of 182 Updated 4/9/2014 Handout Page 136 of 182

Page 137: V1) I - Blue Valley Schools Chemistry/AP... · 3DJH RI 3DJH RI 2001 AP® CHEMISTRY FREE-RESPONSEQUESTIONS Answer EITHER Question 2 below OR Question 3 printed on page 8

2009 AP® CHEMISTRY FREE-RESPONSE QUESTIONS (Form B)

CHEMISTRYSection II

(Total time—95 minutes)

Part A

Time—55 minutesYOU MAY USE YOUR CALCULATOR FOR PART A.

CLEARLY SHOW THE METHOD USED AND THE STEPS INVOLVED IN ARRIVING AT YOUR ANSWERS.It is to your advantage to do this, since you may obtain partial credit if you do and you will receive little or no credit ifyou do not. Attention should be paid to significant figures.

Be sure to write all your answers to the questions on the lined pages following each question in this booklet. DoNOT write your answers on the lavender insert.

Answer Questions 1, 2, and 3. The Section 11 score weighting for each question is 20 percent.

1. A pure 14.85 g sample of the weak base ethylamine, C,H5NH,, is dissolved in enough distilled water to make500. mL of solution.

(a) Calculate the molar concentration of the C2H5NH, in the solution.

The aqueous ethylamine reacts with water according to the equation below.

C2H5NH2(aq) + H20(1) C7H5NH3(aq) + OW(aq)

(b) Write the equilibrium-constant expression for the reaction betweenC2H5NH,(aq) and water.

(c) OfC2H5NH2(aq) andC2H5NH3(aq), which is present in the solution at the higher concentration atequilibrium? Justify your answer.

(d) A different solution is made by mixing 500. rnL of 0.500 41 C-,H5NH, with 500. mL of 0.200 41 HCI.Assume that volumes are additive, The pH of the resulting solution is found to be 10.93.

(i Calculate the concentration of 0H(aq) in the solution.

ii) Write the net-ionic equation that represents the reaction that occurs when the C1H5NH2solution ismixed with the HC1 solution.

(iii) Calculate the molar concentration of theC2HNH(aq) that is formed in the reaction.

(iv) Calculate the value of Kb for C,H5NH,.

© 2009 The College Board. All rights reserved.Visit the College Board on the Web: www.collegeboard.com.

-6- GO ON TO THE NEXT PAGE.

Handout Page 137 of 182 Handout Page 137 of 182

Handout Page 137 of 182 Updated 4/9/2014 Handout Page 137 of 182

Page 138: V1) I - Blue Valley Schools Chemistry/AP... · 3DJH RI 3DJH RI 2001 AP® CHEMISTRY FREE-RESPONSEQUESTIONS Answer EITHER Question 2 below OR Question 3 printed on page 8

2009 AP® CHEMISTRY FREE-RESPONSE QUESTIONS (Form B)

HS2O3(aq) > SO(aq) + S(s)

2. A student performed an experiment to investigate the decomposition of sodium thiosulfate, Na2S,03.in acidicsolution, as represented by the equation above. In each trial the student mixed a different concentration ofsodium thiosulfate with hydrochloric acid at constant temperature and determined the rate of disappearance ofS,032(aq). Data from five trials are given below in the table on the left and are plotted in the graph on the right.

Initial Initial Rate ofConcentration Disappearance

Trial ofS2O3(aq) of S2032 (aq)(M) I (M S )

I 0.050 0.020

2 0.075 0.030

3 0.088 0.034

4 0.112 0.045

5 0.125 0.051

(a) Identify the independent variable in the experiment.

(b) Determine the order of the reaction with respect to S2032. Justify your answer by using the informationabove.

(c) Determine the value of the rate constant, k, for the reaction. Include units in your answer. Show how youarrived at your answer.

(d In another trial the student mixed 0.10 M Na2S2O2 with hydrochloric acid. Calculate the amount of time itwould take for the concentration of S2O2 to drop to 0.020 M.

(e) On the graph above, sketch the line that shows the results that would be expected if the student repeated thefive trials at a temperature lower than that during the first set of trials.

0

Initial Concentration of S,O32(iW)

© 2009 The College Board. All rights reserved.Visit the College Board on the Web: www.collegeboard.com.

-7- GO ON TO THE NEXT PAGE.

Handout Page 138 of 182 Handout Page 138 of 182

Handout Page 138 of 182 Updated 4/9/2014 Handout Page 138 of 182

Page 139: V1) I - Blue Valley Schools Chemistry/AP... · 3DJH RI 3DJH RI 2001 AP® CHEMISTRY FREE-RESPONSEQUESTIONS Answer EITHER Question 2 below OR Question 3 printed on page 8

Handout Page 139 of 182 Handout Page 139 of 182

Handout Page 139 of 182 Updated 4/9/2014 Handout Page 139 of 182

Page 140: V1) I - Blue Valley Schools Chemistry/AP... · 3DJH RI 3DJH RI 2001 AP® CHEMISTRY FREE-RESPONSEQUESTIONS Answer EITHER Question 2 below OR Question 3 printed on page 8

2009 AP® CHEMISTRY FREE-RESPONSE QUESTIONS (Form B)

CHEMISTRY

Part BTime—49 minutes

NO CALCULATORS MAY BE USED FOR PART B.

Answer Question 4 below. The Section II score weighting for this question is 10 percent.

4, For each of the following three reactions, write a balanced equation in part (i) and answer the question inpart (ii) In part (i) coefficients should be in terms of lowest whole numbers Assume that solutions are aqueousunless otherwise indicated. Represent substances in solutions as ions if the substances are extensively ionized.Omit formulas for any ions or molecules that are unchanged by the reaction. You may use the empty space at thebottom of the next page for scratch work, but only equations that are written in the answer boxes provided willbe graded.

p ‘- C)

(a) A barium nitrate solution and a potassium fluoride solution are combined and a precipitate forms.

(i) Balanced equation:

(ii) If equimolar amounts of barium nitrate and potassium fluoride are combined, which reactant, if any,is the limiting reactant? Explain.

© 2009 The College Board. All rights reserved.Visit the College Board on the Web: www.collegeboard.com.

EXAMPLE:

A strip of magnesium metal is added to a solution of silver(I) nitrate.

(I) Balanced equation:

—‘ - z,4

(ii) Which substance is oxidized in the reaction?

-9- GO ON TO THE NEXT PAGE.

Handout Page 140 of 182 Handout Page 140 of 182

Handout Page 140 of 182 Updated 4/9/2014 Handout Page 140 of 182

Page 141: V1) I - Blue Valley Schools Chemistry/AP... · 3DJH RI 3DJH RI 2001 AP® CHEMISTRY FREE-RESPONSEQUESTIONS Answer EITHER Question 2 below OR Question 3 printed on page 8

2009 AP® CHEMISTRY FREE-RESPONSE QUESTIONS (Form B)

(b) A piece of cadmium metal is oxidized by adding it to a solution of copper(ll) chloride.

Balanced equation:

(ii) List two visible changes that would occur in the reaction container as the reaction is proceeding.

(c) A hydrolysis reaction occurs when solid sodium sulfide is added to distilled water.

(i) Balanced equation:

(ii) Indicate whether the pH of the resulting solution is less than 7, equal to 7, or greater than 7. Explain.

© 2009 The College Board, All rights reserved.Visit the College Board on the Web: www.collegeboard.com.

-10- GO ON TO THE NEXT PAGE.

Handout Page 141 of 182 Handout Page 141 of 182

Handout Page 141 of 182 Updated 4/9/2014 Handout Page 141 of 182

Page 142: V1) I - Blue Valley Schools Chemistry/AP... · 3DJH RI 3DJH RI 2001 AP® CHEMISTRY FREE-RESPONSEQUESTIONS Answer EITHER Question 2 below OR Question 3 printed on page 8

2009 AP® CHEMISTRY FREE-RESPONSE QUESTIONS (Form B)

Answer Question 5 and Question 6. The Section II score weighting for these questions is 15 percent each.

Your responses to these questions will be graded on the basis of the accuracy and relevance of the information cited.Explanations should be clear and well organized. Examples and equations may be included in your responses whereappropriate. Specific answers are preferable to broad, diffuse responses.

5. Answer the following questions about nitrogen. hydrogen, and ammonia.

(a) In the boxes below, draw the complete Lewis electron-dot diagrams for N7 and NH3.

N, NH.

(b) Calculate the standard free-energy change, AG°. that occurs when 12.0 g of H,(g) reacts with excessN7(g) at 298 K according to the reaction represented below.

N1(g) + 3 H,(g) 2 NH3(g) AG98 —34 kJ mot’

(c) Given that AH98 for the reaction is —92.2 kJ mol’, which is larger, the total bond dissociation energyof the reactants or the total bond dissociation energy of the products? Explain.

(d) The value of the standard entropy change, AS98, for the reaction is —199 J mol’K’. Explain why thevalue of ASQ, is negative.

(e) Assume that AH° and L\S° for the reaction are independent of temperature.

(1) Explain why there is a temperature above 298 K at which the algebraic sign of the value of AG°changes.

(ii) Theoretically, the best yields of ammonia should be achieved at low temperatures and high pressures.Explain.

© 2009 The College Board. All rights reserved.Visit the College Board on the Web: www.collegeboard.com.

-11- GO ON TO THE NEXT PAGE.

Handout Page 142 of 182 Handout Page 142 of 182

Handout Page 142 of 182 Updated 4/9/2014 Handout Page 142 of 182

Page 143: V1) I - Blue Valley Schools Chemistry/AP... · 3DJH RI 3DJH RI 2001 AP® CHEMISTRY FREE-RESPONSEQUESTIONS Answer EITHER Question 2 below OR Question 3 printed on page 8

2009 AP® CHEMISTRY FREE-RESPONSE QUESTIONS (Form B)

6. Answer the following questions about electrochemical cells.

Voltmeter

It is observed that when silver metal is placed in aqueous thallium(I) fluoride, TIF. no reaction occurs. When theswitch is closed in the cell represented above, the voltage reading is + 1.14 V.

(a> Write the reduction half-reaction that occurs in the cell.

(b) Write the equation for the overall reaction that occurs in the cell.

(c) Identify the anode in the cell. Justify your answer.

(d) On the diagram above, use an arrow to clearly indicate the direction of electron flow as the cell operates.

(e) Calculate the value of the standard reduction potential for the TlITl half-reaction.

The standard reduction potential, E°. of the reaction Pt2 + 2 e —, Pt is 120 V.

(0 Assume that electrodes of pure Pt, Ag, and Ni are available as well as 1.00 M solutions of their salts.Three different electrochemical cells can be constructed using these materials. Identify the two metals thatwhen used to make an electrochemical cell would produce the cell with the largest voltage. Explain how youarrived at your answer.

(g) Predict whether Pt metal will react when it is placed in I 00 Al AgNO-(aq). Justify your answer.

STOP

END OF EXAM

2009 The College Board. All rights reserved.Visit the College Board on the Web: www.collegeboard.com.

-12-

1.OOMT1F l.OOMAgF

Handout Page 143 of 182 Handout Page 143 of 182

Handout Page 143 of 182 Updated 4/9/2014 Handout Page 143 of 182

Page 144: V1) I - Blue Valley Schools Chemistry/AP... · 3DJH RI 3DJH RI 2001 AP® CHEMISTRY FREE-RESPONSEQUESTIONS Answer EITHER Question 2 below OR Question 3 printed on page 8

2010 AP® CHEMISTRY FREE-RESPONSE QUESTIONS

CHEMISTRY

Section II

(Total time—95 minutes)

Part ATime— 55 minutes

YOU MAY USE YOUR CALCULATOR FOR PART A.

CLEARLY SHOW THE METHOD USED AND THE STEPS INVOLVED IN ARRIVING AT YOUR ANSWERS.It is to your advantage to do this, since you may obtain partial credit if you do and you will receive little or no credit ifyou do not. Attention should be paid to significant figures.

Be sure to write all your answers to the questions on the lined pages following each question in the booklet with thepink cover. Do NOT write your answers on the green insert.

Answer Questions I, 2, and 3. The Section II score weighting for each question is 20 percent.

1. Several reactions are carried out using AgBr, a cream-colored silver salt for which the value of the solubilityproduct constant, is 5,0 x 10—13 at 298 K.

(a) Write the expression for the solubility-product constant, of AgBr.

(b) Calculate the value of [Ag1 in 50.0 mL of a saturated solution of AgBr at 298 K.

(c) A 50.0 mL sample of distilled water is added to the solution described in part (b), which is in a beaker withsome solid AgBr at the bottom. The solution is stirred and equilibrium is reestablished. Some solid AgBrremains in the beaker. Is the value of [AgJ greater than, less than, or equal to the value you calculated inpart (b) ? Justify your answer.

(d) Calculate the minimum volume of distilled water, in liters, necessary to completely dissolve a 5.0 g sampleof AgBr(s) at 298 K. (The molar mass of AgBr is 188 g moL’.)

(e) A student mixes 10.0 mL of 1.5 l04M AgNO3 with 2.0 mL of 5.0 x 104M NaBr and stirs the resultingmixture. What will the student observe? Justify your answer with calculations.

(f) The color of another salt of silver, AgI(s), is yellow. A student adds a solution of Nal to a test tubecontaining a small amount of solid, cream-colored AgBr. After stirring the contents of the test tube, thestudent observes that the solid in the test tube changes color from cream to yellow.

(i) Write the chemical equation for the reaction that occurred in the test tube.

(ii) Which salt has the greater value of K: AgBr or AgI ‘?Justify your answer.

2010 The College Board.Visit the College Board on the Web: www.collegeboard.com.

GO ON TO THE NEXT PAGE.-6-

Handout Page 144 of 182 Handout Page 144 of 182

Handout Page 144 of 182 Updated 4/9/2014 Handout Page 144 of 182

Page 145: V1) I - Blue Valley Schools Chemistry/AP... · 3DJH RI 3DJH RI 2001 AP® CHEMISTRY FREE-RESPONSEQUESTIONS Answer EITHER Question 2 below OR Question 3 printed on page 8

2010 AP® CHEMISTRY FREE-RESPONSE QUESTIONS

2. A student performs an experiment to determine the molar enthalpy of solution of urea,H2NCONH2.The studentplaces 91.95 g of water at 25°C into a coffee-cup calorimeter and immerses a thermometer in the water.After 50 s, the student adds 5.13 g of solid urea, also at 25°C, to the water and measures the temperature of thesolution as the urea dissolves. A plot of the temperature data is shown in the graph below.

26.0

25.0U

24.0

23.0

22.0

21.0

Time (s)

(a) Determine the change in temperature of the solution that results from the dissolution of the urea.

(b) According to the data, is the dissolution of urea in water an endothermic process or an exothermic process?Justify your answer.

(c) Assume that the specific heat capacity of the calorimeter is negligible and that the specific heat capacity ofthe solution of urea and water is 4.2 J g’ °C throughout the experiment.

(i) Calculate the heat of dissolution of the urea injoules.

(ii) Calculate the molar enthalpy of solution, AH)bI, of urea in kJ moU1.

(d) Using the information in the table below, calculate the value of the molar entropy of solution, AS,,,, , ofurea at 298 K. Include units with your answer.

(e) The student repeats the experiment and this time obtains a result for AN0,,, of urea that is II percent

below the accepted value. Calculate the value of AHQ,fl that the student obtained in this second trial.

(f) The student performs a third trial of the experiment but this time adds urea that has been taken directlyfrom a refrigerator at 5°C. What effect. if any, would using the cold urea instead of urea at 25°C have onthe experimentally obtained value of 7 Justify your answer.

C 2010 The College Board.Visit the College Board on the Web: www.collegeboar&com.

GO ON TO THE NEXT PAGE.-7-

Handout Page 145 of 182 Handout Page 145 of 182

Handout Page 145 of 182 Updated 4/9/2014 Handout Page 145 of 182

Page 146: V1) I - Blue Valley Schools Chemistry/AP... · 3DJH RI 3DJH RI 2001 AP® CHEMISTRY FREE-RESPONSEQUESTIONS Answer EITHER Question 2 below OR Question 3 printed on page 8

2010 AP CHEMISTRY FREE-RESPONSE QUESTIONS

8 H(aq) +4 Cl(aq) + Mn04(aq) 2 C12(g) + Mn3(aq) + 4 WO(i)

3. Clg) can be generated in the laboratory by reacting potassium permanganate with an acidified solution ofsodium chloride. The net-ionic equation for the reaction is given above.

(a) A 25.00 mL sample of 0.250 M NaCI reacts completely with excess KMnO4(aq). The C12(g) produced isdried and stored in a sealed container. At 22°C the pressure of the C12(g) in the container is 0.950 atm.

(i) Calculate the number of moles of C1(aq) present before any reaction occurs.

(ii) Calculate the volume, in L, of the C12(g) in the sealed container.

An initial-rate study was performed on the reaction system. Data for the experiment are given in the table below.

Trial [Cl-I [Ma04] [Wi Rate of Disappearance of Mn04 in M s

1 0.0104 0.00400 3.00 2.25 x 10-8

2 0.03 12 0.00400 3.00 2.03 x l0

3 0.03 12 0.00200 3.00 1.02 x l0

(b) Using the information in the table, determine the order of the reaction with respect to each of the following.Justify your answers.

(i) C1

(ii) Mn04

(c) The reaction is known to be third order with respect to H. Using this information and your answers topart (b) above, complete both of the following:

(i) Write the rate law for the reaction.

ii) Calculate the value of the rate constant. k, for the reaction, including appropriate units.

(d) Is it likely that the reaction occurs in a single elementary step? Justify your answer.

STOPIf you finish before time is called, you may check your work on this part only.

Do not turn to the other part of the test until you are told to do so.

0 2010 The College Board.Visit the College Board on the Web: www.collegeboard.com.

-8-

Handout Page 146 of 182 Handout Page 146 of 182

Handout Page 146 of 182 Updated 4/9/2014 Handout Page 146 of 182

Page 147: V1) I - Blue Valley Schools Chemistry/AP... · 3DJH RI 3DJH RI 2001 AP® CHEMISTRY FREE-RESPONSEQUESTIONS Answer EITHER Question 2 below OR Question 3 printed on page 8

2010 AP® CHEMISTRY FREE-RESPONSE QUESTIONS

CHEMISTRY

Part BTime—40 minutes

NO CALCULATORS MAY BE USED FOR PART B.

Answer Question 4 below. The Section II score weighting for this question is 10 percent.

4. For each of the following three reactions, write a balanced equation for the reaction in part (i) and answer thequestion about the reaction in part (ii). In part (i), coefficients should be in terms of lowest whole numbers.Assume that solutions are aqueous unless otherwise indicated. Represent substances in solutions as ions if thesubstances are extensively ionized. Omit formulas for any ions or molecules that are unchanged by the reaction.You may use the empty space at the bottom of the next page for scratch work, but only equations that are writtenin the answer boxes provided will be scored.

(i) Balanced equation: 1pr

(a) A 0.2 Al potassium hydroxide solution is titrated with a 0.1 M nitric acid solution.

(i) Balanced equation:

(ii) What would be observed if the solution was titrated well past the equivalence point using bromthymolblue as the indicator? (Bromthymol blue is yellow in acidic solution and blue in basic solution.)

© 2010 The College Board.Visit the College Board on the Web: www.collegeboard.com.

GO ON TO THE NEXT PAGE.

EXAMPLE

A strip of magnesium metal is added to a solution of silver(I) nitrate.

(ii) Which substance is oxidized in the reaction?

-9-

Handout Page 147 of 182 Handout Page 147 of 182

Handout Page 147 of 182 Updated 4/9/2014 Handout Page 147 of 182

Page 148: V1) I - Blue Valley Schools Chemistry/AP... · 3DJH RI 3DJH RI 2001 AP® CHEMISTRY FREE-RESPONSEQUESTIONS Answer EITHER Question 2 below OR Question 3 printed on page 8

2010 AP CHEMISTRY FREE-RESPONSE QUESTIONS

(b) Propane is burned completely in excess oxygen gas.

(i) Balanced equation:

(ii) When the products of the reaction are bubbled through distilled water, is the resulting solution neutral,acidic. or basic? Explain.

(c) A solution of hydrogen peroxide is heated, and a gas is produced.

(i) Balanced equation:

(ii) Identify the oxidation state of oxygen in hydrogen peroxide.

YOU MAY USE THE SPACE BELOW FOR SCRATCH WORK, BUT ONLY EQUATIONSTHAT ARE WRITTEN IN THE ANSWER BOXES PROVIDED WILL BE SCORED.

2010 The College Board.Visit the College Board on the Web: www.collegeboard.com.

GO ON TO THE NEXT PAGE.-10-

Handout Page 148 of 182 Handout Page 148 of 182

Handout Page 148 of 182 Updated 4/9/2014 Handout Page 148 of 182

Page 149: V1) I - Blue Valley Schools Chemistry/AP... · 3DJH RI 3DJH RI 2001 AP® CHEMISTRY FREE-RESPONSEQUESTIONS Answer EITHER Question 2 below OR Question 3 printed on page 8

2010 AP® CHEMISTRY FREE-RESPONSE QUESTIONS

Answer Question 5 and Question 6. The Section II score weighting for these questions is 15 percent each.

Your responses to these questions will be scored on the basis of the accuracy and relevance of the information cited.Explanations should be clear and well organized. Examples and equations may be included in your responses whereappropriate. Specific answers are preferable to broad, diffuse responses.

5. Use the information in the table below to respond to the statements and questions that follow. Your answersshould be in terms of principles of molecular structure and intermolecular forces.

Compound Formula Lewis Electron-Dot Diagram

HHEthanethiol CH3CH,SH H: : : H

HH

HREthane CH3CH3 H:C:C:H

HR

HHEthanol CH3CHIOH H:C:C:0:H

HH

Ethyne CR-,

(a) Draw the complete Lewis electron-dot diagram for ethyne in the appropriate cell in the table above.

(b) Which of the four molecules contains the shortest carbon-to-carbon bond? Explain.

(c) A Lewis electron-dot diagram of a molecule of ethanoic acid is given below. The carbon atoms in themolecule are labeled .v and y, respectively.

:O:H

H:O:C:C:H

H

Identify the geometry of the arrangement of atoms bonded to each of the following.

(i) Carbon x

(ii) Carbon v

(d) Energy is required to boil ethanol. Consider the statement “As ethanol boils, energy goes into breaking C—Cbonds. C—H bonds. C—0 bonds, and 0—H bonds. Is the statement true or false? Justify your answer.

(e) Identify a compound from the table above that is nonpolar. Justify your answer.

(f Ethanol is completely soluble in water, whereas ethanethiol has limited solubility in water. Account for thedifference in solubilities between the two compounds in terms of intermolecular forces.

2010 The College Board.Visit the College Board on the Web: www.collegeboard.com.

GO ON TO THE NEXT PAGE.—11—

Handout Page 149 of 182 Handout Page 149 of 182

Handout Page 149 of 182 Updated 4/9/2014 Handout Page 149 of 182

Page 150: V1) I - Blue Valley Schools Chemistry/AP... · 3DJH RI 3DJH RI 2001 AP® CHEMISTRY FREE-RESPONSEQUESTIONS Answer EITHER Question 2 below OR Question 3 printed on page 8

2010 AP® CHEMISTRY FREE-RESPONSE QUESTIONS

2 Al(s) + 3 Zn(aq) —* 2 AlS1(aq) + 3 Zn(s)

6. Respond to the following statements and questions that relate to the species and the reaction represented above.

(a) Write the complete electron configuration (e.g.. Is’ 2s2. . .) for Zn2.

(b) Which species, Zn or Zn2, has the greater ionization energy? Justify your answer.

(c) Identify the species that is oxidized in the reaction.

The diagram below shows a galvanic cell based on the reaction. Assume that the temperature is 25°C.

1.0 M Al(N03)3 1.0 M Zn(N03),

(d) The diagram includes a salt bridge that is filled with a saturated solution of KNO3. Describe what happensin the salt bridge as the cell operates.

(e) Determine the value of the standard voltage, E°, for the cell.

(D Indicate whether the value of the standard free-energy change, AG°, for the cell reaction is positive,negative, or zero. Justify your answer.

(g) If the concentration of Al(NO3) in the AHsilAl3(aq. half-cell is lowered from l.OM to 0.01 Mat 25°C.does the cell voltage increase, decrease, or remain the same? Justify your answer.

STOP

END OF EXAM

2010 The College Board.Visit the College Board on the Web www.collegeboard.com.

-12-

Handout Page 150 of 182 Handout Page 150 of 182

Handout Page 150 of 182 Updated 4/9/2014 Handout Page 150 of 182

Page 151: V1) I - Blue Valley Schools Chemistry/AP... · 3DJH RI 3DJH RI 2001 AP® CHEMISTRY FREE-RESPONSEQUESTIONS Answer EITHER Question 2 below OR Question 3 printed on page 8

2010 AP CHEMISTRY FREE-RESPONSE QUESTIONS (Form B)

CHEMISTRY

Section II(Total time—95 minutes)

Part A

Time—55 minutesYOU MAY USE YOUR CALCULATOR FOR PART A.

CLEARLY SHOW THE METHOD USED AND THE SThPS INVOLVED IN ARRIVING AT YOUR ANSWERS.It is to your advantage to do this, since you may obtain partial credit if you do and you will receive little or no credit ifyou do not. Attention should be paid to significant figures.

Be sure to write all your answers to the questions on the lined pages following each question in the goldenrodbooklet. Do NOT write your answers on the lavender insert.

Answer Questions 1, 2, and 3. The Section II score weighting for each question is 20 percent.

1. The compound butane, C4H10,occurs in two isomeric forms, n-butane and isobutane (2-methyl propane).Both compounds exist as gases at 25°C and 1.0 atm.

(a) Draw the structural formula of each of the isomers (include all atoms). Clearly label each structure.

(b) On the basis of molecular structure, identify the isomer that has the higher boiling point. Justify youranswer.

The two isomers exist in equilibrium as represented by the equation below.

n-butane(g) isobutane(g) K 2.5 at 25°C

Suppose that a 0.0 10 mol sample of pure n-butane is placed in an evacuated 1.0 L rigid container at 25°C.

(c) Write the expression for the equilibrium constant, K, for the reaction.

(dJ Calculate the initial pressure in the container when the n-butane is first introduced (before the reactionstarts),

(e) The n-butane reacts until equilibrium has been established at 25°C.

(i) Calculate the total pressure in the container at equilibrium. Justify your answer.

(ii) Calculate the molar concentration of each species at equilibrium.

(iii) If the volume of the system is reduced to half of its original volume, what will he the newconcentration of n-butane after equilibrium has been reestablished at 25°C ? Justify your answer.

Suppose that in another experiment a 0.010 mol sample of pure isobutane is placed in an evacuated 1 .0 L rigidcontainer and allowed to come to equilibrium at 25°C.

(f) Calculate the molar concentration of each species after equilibrium has been established.

© 2010 The College Board.Visit the College Board on the Web: www.collegeboard.com.

GO ON TO THE NEXT PAGE.-6-

Handout Page 151 of 182 Handout Page 151 of 182

Handout Page 151 of 182 Updated 4/9/2014 Handout Page 151 of 182

Page 152: V1) I - Blue Valley Schools Chemistry/AP... · 3DJH RI 3DJH RI 2001 AP® CHEMISTRY FREE-RESPONSEQUESTIONS Answer EITHER Question 2 below OR Question 3 printed on page 8

2010 AP® CHEMISTRY FREE-RESPONSE QUESTIONS (Form B)

5 Fe2(aq) + Mn04(aq) + 8 H(aq) —* 5 Fe3(aq) + Mn2(aq) ÷4H20(1)

2. A galvanic cell and the balanced equation for the spontaneous cell reaction are shown above. The two reductionhalf-reactions for the overall reaction that occurs in the cell are shown in the table below.

Half-Reaction E° (V) at298 K

Fe3(aq) + e — Fe2(aq) + 0.77

Mn04(aq) + 8 H4(aq) + 5 e — Mn2(aq) + 4 H,O(1) ÷1.49

(a) On the diagram, clearly label the cathode.

(b) Calculate the value of the standard potential. E°. for the spontaneous cell reaction.

(c) How many moles of electrons are transferred when 1.0 mol of MnO4(aq) is consumed in the overallcell reaction?

(d) Calculate the value of the equilibrium constant, for the cell reaction at 25°C. Explain what themagnitude of Keq tells you about the extent of the reaction.

Three solutions, one containing Fe2(aq), one containing Mn04(aq), and one containing H(aq), are mixedin a beaker and allowed to react. The initial concentrations of the species in the mixture are 0.60 M Fe2(aq),0.10 M Mn04(aq). and 1.0 W H(aq).

(e) When the reaction mixture has come to equilibrium, which species has the higher concentration, Mn2(aq)or Mn04(aq)? Explain.

(f) When the reaction mixture has come to equilibrium, what are the molar concentrations of Fe2(aq) andFe3(aq)?

© 2010 The College Board.Visit the College Board on the Web: www.collegeboardcom.

GO ON TO THE NEXT PAGE.-7-

Handout Page 152 of 182 Handout Page 152 of 182

Handout Page 152 of 182 Updated 4/9/2014 Handout Page 152 of 182

Page 153: V1) I - Blue Valley Schools Chemistry/AP... · 3DJH RI 3DJH RI 2001 AP® CHEMISTRY FREE-RESPONSEQUESTIONS Answer EITHER Question 2 below OR Question 3 printed on page 8

2010 AP CHEMISTRY FREE-RESPONSE QUESTIONS (Form B)

3. A sample of ore containing the mineral tellurite, Te02,was dissolved in acid. The resulting solution was thenreacted with a solution of K,Cr207 to form telluric acid. H1TeO4.The unbalanced chemical equation for thereaction is given below.

TeO,(s) + . . Cr,O72(aq) + . . . H(aq) — .. . H,TeO(aq) + .. Cr(aq) + . . . H20(1)

(a) identifv the molecule or ion that is being oxidized in the reaction.

(h) Give the oxidation number of Cr in the Cr,O72(aq) ion.

(c) Balance the chemical equation given above by writing the correct lowest whole-number coefticients on thedotted lines.

In the procedure described above. 46.00 mL of 0.03109 M K2Cr,07 was added to the ore sample after itwas dissolved in acid. When the chemical reaction had progressed as completely as possible, the amount ofunreacted (excess) Cr2O72(aq) was determined by titrating the solution with 0.110 M Fe(N03)2.Thereaction that occurred during the titration is represented by the following balanced equation.

6 Fe(aq) +Cr2072(aq) + 14 H4(aq) — 2 Cr3(aq) + 6 Fe3(aq) + 7 H20(l)

A volume of 9.85 mL of 0.110 M Fe(N03)1 was required to reach the equivalence point.

(d) Calculate the number of moles of excess Cr2072(aq) that was titrated.

(e) Calculate the number of moles of Cr2O72(aq) that reacted with the tellurite.

(f) Calculate the mass, in grains, of tellurite that was in the ore sample.

STOPit you finish before time is called, you may check your work on this part only.

Do not turn to the other part of the test until you are told to do so.

© 2010 The College Board.Visit the College Board on the Web: wwwcollegeboard.corn.

-8-

Handout Page 153 of 182 Handout Page 153 of 182

Handout Page 153 of 182 Updated 4/9/2014 Handout Page 153 of 182

Page 154: V1) I - Blue Valley Schools Chemistry/AP... · 3DJH RI 3DJH RI 2001 AP® CHEMISTRY FREE-RESPONSEQUESTIONS Answer EITHER Question 2 below OR Question 3 printed on page 8

2010 APn CHEMISTRY FREE-RESPONSE QUESTIONS (Form B)

CHEMISTRYPart B

Time—40 minutesNO CALCULATORS MAY BE USED FOR PART B.

Answer Question 4 below. The Section 11 score weighting for this question is 10 percent.

4. For each of the following three reactions, in part (i) write a balanced equation and in part (ii) answer the questionabout the reaction. In part (i), coefficients should be in terms of lowest whole numbers. Assume that solutionsare aqueous unless otherwise indicated. Represent substances in solutions as ions if the substances areextensively ionized. Omit formulas for any ions or molecules that are unchanged by the reaction. You may usethe empty space at the bottom of the next page for scratch work, but only equations that are written in the answerboxes provided will be scored.

EXAMPLE:

A strip of magnesium metal is added to a solution of silver(I) nitrate.

(i) Balanced equation:

(ii) Which substance is oxidized in the reaction?

A,t ,

(a) Solid copper(ll) sulfate pentahydrate is gently heated.

(i) Balanced equation:

(ii) How many grams of water are present in 1.00 mol of copper(II) sulfate pentahydrate?

© 2010 The College Board.Visit the College Board on the Web: www.eollegeboard.com.

GO ON TO THE NEXT PAGE.-9-

Handout Page 154 of 182 Handout Page 154 of 182

Handout Page 154 of 182 Updated 4/9/2014 Handout Page 154 of 182

Page 155: V1) I - Blue Valley Schools Chemistry/AP... · 3DJH RI 3DJH RI 2001 AP® CHEMISTRY FREE-RESPONSEQUESTIONS Answer EITHER Question 2 below OR Question 3 printed on page 8

2010 AP CHEMISTRY FREE-RESPONSE QUESTIONS (Form B)

(b) Excess concentrated aqueous ammonia is added to a solution of nickel(II) nitrate, leading to the formation ofa complex ion.

i) Balanced equation:

(ii) Which of the reactants acts as a Lewis acid?

(c) Methylamine (CH3NH2) is added to a solution of hydrochloric acid.

(i) Balanced equation:

(ii) Methylamine dissolves in water to form a solution. Indicate whether this solution is acidic, basic, orneutral.

Q) 2010 The College Board.Visit the College Board on the Web: www.ollegeboard.com.

GO ON TO THE NEXT PAGE.-10-

Handout Page 155 of 182 Handout Page 155 of 182

Handout Page 155 of 182 Updated 4/9/2014 Handout Page 155 of 182

Page 156: V1) I - Blue Valley Schools Chemistry/AP... · 3DJH RI 3DJH RI 2001 AP® CHEMISTRY FREE-RESPONSEQUESTIONS Answer EITHER Question 2 below OR Question 3 printed on page 8

2010 AP CHEMISTRY FREE-RESPONSE QUESTIONS (Form B>

Answer Question 5 and Question 6. The Section II score weighting for these questions is 15 percent each.

Your responses to these questions will be scored on the basis of the accuracy and relevance of the information cited.Explanations should be clear and vel1 organized. Examples and equations may be included in your responses whereappropriate. Specific answers are preferable to broad, diffuse responses.

pH

o 20.0 40.0 60.0 80.0Volume of Titrant Added (mL)

5. A solution of 0.100 M HC1 and a solution of 0.100 M NaOH are prepared. A 40.0 mL sample of one of thesolutions is added to a beaker and then titrated with the other solution. A pH electrode is used to obtain the datathat are plotted in the titration curve shown above.

(a) Identify the solution that was initially added to the beaker. Explain your reasoning.

(b) On the titration curve above, circle the point that corresponds to the equivalence point.

(c) At the equivalence point, how many moles of titrant have been added?

(d) The same titration is to be performed again, this time using an indicator. Use the information in the tablebelow to select the best indicator for the titration. Explain your choice.

. pH Range ofIndicatorColor Change

Methyl violet 0— 1.6

Methyl red 4—6

Alizarin yellow 10— 12

(e) What is the difference between the equivalence point of a titration and the end point of a titration?

(f) On the grid provided on the next page. sketch the titration curve that would result if the solutions in thebeaker and buret were reversed (i.e., if 30.0 rnL of the solution used in the buret in the previous titrationwere titrated with the solution that was in the beaker).

T 2010 The College Board.Visit the College Board on the Web: www.collegeboard.com.

GO ON TO THE NEXT PAGE.

:

—11—

Handout Page 156 of 182 Handout Page 156 of 182

Handout Page 156 of 182 Updated 4/9/2014 Handout Page 156 of 182

Page 157: V1) I - Blue Valley Schools Chemistry/AP... · 3DJH RI 3DJH RI 2001 AP® CHEMISTRY FREE-RESPONSEQUESTIONS Answer EITHER Question 2 below OR Question 3 printed on page 8

2010 AP® CHEMISTRY FREE-RESPONSE QUESTIONS (Form B>

Volume of Titrant Added (mL)

© 2010 The College Board.Visit the College Board on the Web: www.collegeboard.com.

GO ON TO THE NEXT PAGE.

II

p’-1

12 —h J

l _i______

6-

-______

0“

0 20X) 40.0 60.0 80.0

-12-

Handout Page 157 of 182 Handout Page 157 of 182

Handout Page 157 of 182 Updated 4/9/2014 Handout Page 157 of 182

Page 158: V1) I - Blue Valley Schools Chemistry/AP... · 3DJH RI 3DJH RI 2001 AP® CHEMISTRY FREE-RESPONSEQUESTIONS Answer EITHER Question 2 below OR Question 3 printed on page 8

2010 AP® CHEMISTRY FREE-RESPONSE QUESTIONS (Form B)

H2(g) +02(g) 2 HC1(g)

6. The table below gives data for a reaction rate study of the reaction represented above.

Initial [ H- I Initial [Clj Initial Rate of FormationExperiment -

(mol L) (mol L) of HC1 (mol L s )

1 0.00100 0.000500 l.82x l0_12

2 000200 0.000500 3.64xlO12

3 0.00200 0.000250 1.82 x l0_12

(a) Determine the order of the reaction with respect to H2 and justify your answer.

(b) Determine the order of the reaction with respect to Cl2 and justify your answer.

(c) Write the overall rate law for the reaction.

(d) Write the units of the rate constant.

(e) Predict the initial rate of the reaction if the initial concentration of H2 is 0.00300 mol L and theinitial concentration of Cl2 is 0.000500 mol L.

The gas-phase decomposition of nitrous oxide has the following two-step mechanism.

Step I: N,0 —* N,+0

Step 2: 0 + N20 — N2 + 02

(f) Write the balanced equation for the overall reaction.

(g Is the oxygen atom, 0, a catalyst for the reaction or is it an intermediate? Explain.

(h) Identify the slower step in the mechanism if the rate law for the reaction was determined to herate = k [N20]. Justify your answer.

STOP

END OF EXAM

© 2010 The College Board.Visit the College Board on the Web: www.collegeboard.com.

-13-

Handout Page 158 of 182 Handout Page 158 of 182

Handout Page 158 of 182 Updated 4/9/2014 Handout Page 158 of 182

Page 159: V1) I - Blue Valley Schools Chemistry/AP... · 3DJH RI 3DJH RI 2001 AP® CHEMISTRY FREE-RESPONSEQUESTIONS Answer EITHER Question 2 below OR Question 3 printed on page 8

Handout Page 159 of 182 Handout Page 159 of 182

Handout Page 159 of 182 Updated 4/9/2014 Handout Page 159 of 182

Page 160: V1) I - Blue Valley Schools Chemistry/AP... · 3DJH RI 3DJH RI 2001 AP® CHEMISTRY FREE-RESPONSEQUESTIONS Answer EITHER Question 2 below OR Question 3 printed on page 8

Handout Page 160 of 182 Handout Page 160 of 182

Handout Page 160 of 182 Updated 4/9/2014 Handout Page 160 of 182

Page 161: V1) I - Blue Valley Schools Chemistry/AP... · 3DJH RI 3DJH RI 2001 AP® CHEMISTRY FREE-RESPONSEQUESTIONS Answer EITHER Question 2 below OR Question 3 printed on page 8

Handout Page 161 of 182 Handout Page 161 of 182

Handout Page 161 of 182 Updated 4/9/2014 Handout Page 161 of 182

Page 162: V1) I - Blue Valley Schools Chemistry/AP... · 3DJH RI 3DJH RI 2001 AP® CHEMISTRY FREE-RESPONSEQUESTIONS Answer EITHER Question 2 below OR Question 3 printed on page 8

Handout Page 162 of 182 Handout Page 162 of 182

Handout Page 162 of 182 Updated 4/9/2014 Handout Page 162 of 182

Page 163: V1) I - Blue Valley Schools Chemistry/AP... · 3DJH RI 3DJH RI 2001 AP® CHEMISTRY FREE-RESPONSEQUESTIONS Answer EITHER Question 2 below OR Question 3 printed on page 8

Handout Page 163 of 182 Handout Page 163 of 182

Handout Page 163 of 182 Updated 4/9/2014 Handout Page 163 of 182

Page 164: V1) I - Blue Valley Schools Chemistry/AP... · 3DJH RI 3DJH RI 2001 AP® CHEMISTRY FREE-RESPONSEQUESTIONS Answer EITHER Question 2 below OR Question 3 printed on page 8

Handout Page 164 of 182 Handout Page 164 of 182

Handout Page 164 of 182 Updated 4/9/2014 Handout Page 164 of 182

Page 165: V1) I - Blue Valley Schools Chemistry/AP... · 3DJH RI 3DJH RI 2001 AP® CHEMISTRY FREE-RESPONSEQUESTIONS Answer EITHER Question 2 below OR Question 3 printed on page 8

Handout Page 165 of 182 Handout Page 165 of 182

Handout Page 165 of 182 Updated 4/9/2014 Handout Page 165 of 182

Page 166: V1) I - Blue Valley Schools Chemistry/AP... · 3DJH RI 3DJH RI 2001 AP® CHEMISTRY FREE-RESPONSEQUESTIONS Answer EITHER Question 2 below OR Question 3 printed on page 8

Handout Page 166 of 182 Handout Page 166 of 182

Handout Page 166 of 182 Updated 4/9/2014 Handout Page 166 of 182

Page 167: V1) I - Blue Valley Schools Chemistry/AP... · 3DJH RI 3DJH RI 2001 AP® CHEMISTRY FREE-RESPONSEQUESTIONS Answer EITHER Question 2 below OR Question 3 printed on page 8

Handout Page 167 of 182 Handout Page 167 of 182

Handout Page 167 of 182 Updated 4/9/2014 Handout Page 167 of 182

Page 168: V1) I - Blue Valley Schools Chemistry/AP... · 3DJH RI 3DJH RI 2001 AP® CHEMISTRY FREE-RESPONSEQUESTIONS Answer EITHER Question 2 below OR Question 3 printed on page 8

Handout Page 168 of 182 Handout Page 168 of 182

Handout Page 168 of 182 Updated 4/9/2014 Handout Page 168 of 182

Page 169: V1) I - Blue Valley Schools Chemistry/AP... · 3DJH RI 3DJH RI 2001 AP® CHEMISTRY FREE-RESPONSEQUESTIONS Answer EITHER Question 2 below OR Question 3 printed on page 8

Handout Page 169 of 182 Handout Page 169 of 182

Handout Page 169 of 182 Updated 4/9/2014 Handout Page 169 of 182

Page 170: V1) I - Blue Valley Schools Chemistry/AP... · 3DJH RI 3DJH RI 2001 AP® CHEMISTRY FREE-RESPONSEQUESTIONS Answer EITHER Question 2 below OR Question 3 printed on page 8

Handout Page 170 of 182 Handout Page 170 of 182

Handout Page 170 of 182 Updated 4/9/2014 Handout Page 170 of 182

Page 171: V1) I - Blue Valley Schools Chemistry/AP... · 3DJH RI 3DJH RI 2001 AP® CHEMISTRY FREE-RESPONSEQUESTIONS Answer EITHER Question 2 below OR Question 3 printed on page 8

Handout Page 171 of 182 Handout Page 171 of 182

Handout Page 171 of 182 Updated 4/9/2014 Handout Page 171 of 182

Page 172: V1) I - Blue Valley Schools Chemistry/AP... · 3DJH RI 3DJH RI 2001 AP® CHEMISTRY FREE-RESPONSEQUESTIONS Answer EITHER Question 2 below OR Question 3 printed on page 8

Handout Page 172 of 182 Handout Page 172 of 182

Handout Page 172 of 182 Updated 4/9/2014 Handout Page 172 of 182

Page 173: V1) I - Blue Valley Schools Chemistry/AP... · 3DJH RI 3DJH RI 2001 AP® CHEMISTRY FREE-RESPONSEQUESTIONS Answer EITHER Question 2 below OR Question 3 printed on page 8

Handout Page 173 of 182 Handout Page 173 of 182

Handout Page 173 of 182 Updated 4/9/2014 Handout Page 173 of 182

Page 174: V1) I - Blue Valley Schools Chemistry/AP... · 3DJH RI 3DJH RI 2001 AP® CHEMISTRY FREE-RESPONSEQUESTIONS Answer EITHER Question 2 below OR Question 3 printed on page 8

2012 AP® CHEMISTRY FREE-RESPONSE QUESTIONS

CHEMISTRY

Section II

TotaI time—95 miiiutes

Part A

Time—55 minutes

YOU MAY USE YOUR CALCULATOR FOR PART A.

CLEARLY SHOW THE METHOD USED AND THE STEPS INVOLVED TN ARRIVING AT YOUR ANSWERS.It 1% to \ our ad ml ice to do this smnLe ‘ nu mas obtain parti ii LrLdit it s ou do md \ ou \ ill recei\ e httk or no Lredmt it

you do not. Attention should be paid to significant figures. Bc sure to write all your answers to the questions on thelined pages following each question in this booklet.

Answer Questions 1. 2. and 3. The Section II score weightins’ for each question is 20 percent.

A 1.22 g sample of a pure monoprotic acid, HA, was dissolved in distilled water. The HA solution was thentitrated with 0.250 M NaOH. The pH was measured throughout the titration, and the equivalence point wasreached when 40.0 mL of the NaOH solution had been added. The data from the titration are recorded in thetable below.

nietf0.250tWNaOHpHofT1tratedAdded (mL) 0 UtiOfl

0.00 j 7

10.0 3.72

20.0 4.20

30.0 7

40.0 8.62

[ 50.0 12.40

(a) Explain how the data in the table above provide evidence that HA is a weak acid rather than a strong acid.

(b) Write the balanced net-ionic equation for the reaction that occurs when the solution of NaOH is added to thesolution of HA.

ic) Calculate the number of moles of HA that were titrated.

(d) Calculate the molar mass of HA

The equation for the dissociation reaction of HA in water is shown below.

HA aq) ÷ H5O(h HOwq + ATh aq K,. = 6.3 x I

(e) Assume that the initial concentration of the HA solution (before any NaCFI solution was added is 0,200 M.Determine the pH of the initial HA solution.

(0 Calculate the value of [HO] in the solution after 30.0 mL of NaOH solution is added and the total volumeof the solution is 80.0 rnL.

GO ON TO THE NEXT PAGE.-6-

Handout Page 174 of 182 Handout Page 174 of 182

Handout Page 174 of 182 Updated 4/9/2014 Handout Page 174 of 182

Page 175: V1) I - Blue Valley Schools Chemistry/AP... · 3DJH RI 3DJH RI 2001 AP® CHEMISTRY FREE-RESPONSEQUESTIONS Answer EITHER Question 2 below OR Question 3 printed on page 8

2012 AP0 CHEMISTRY FREE-RESPONSE QUESTIONS

. A sample of a pure. gaseous h’.drocarhon is introduced into a previously evacuated rigid 1.1)0 1. esel. Chepressure of the gas i5 O.20t) aim at a temperature of I2C.

(a Calculate the number of moles of the hdrocarhon in the vessel.

(hi 02(C is introduced into the same vessel containing the hydrocarbon, Afterthe addition of the Ogi. thetotal pressure of the gas mixture in the essei is 1,40 atm at 127C. Calculate the partial pressure of O ç’ inthe vessel.

The mixture of the h drocarhon and oxygen is sparked so that a complete combustion reaction occurs. po siucincC02ici and I 1:O ci, The partial presures of these gases at I 27°C are (1.600 atm for C02 c and t).S0(J atm forH20(ç). There is 02(g remaining in the container after the reaction is complete.

(c) Use the partial pres’ures of C02(ç) and H2O(g) to calculate the partial pressure of the 02(g) cunumed inthe combustion.

d) On the basis of your ansv ers ahose, write the balanced chemical equation for the combustion reaction anddetermine the formula of the hydrocarbon.

idi Calculate the mass of the hdrocarbon that sas combusted.

fi As the vessel cools to room temperature. droplets of liquid water form on the inside ails of the container.Predict whether the pH of the water in the vessel is less than 7. equal to 7, or greater than 7. Explain ourprediction.

ih ll I3’,rd.\‘t.

GO ON TO THE NEXT PAGE.

Handout Page 175 of 182 Handout Page 175 of 182

Handout Page 175 of 182 Updated 4/9/2014 Handout Page 175 of 182

Page 176: V1) I - Blue Valley Schools Chemistry/AP... · 3DJH RI 3DJH RI 2001 AP® CHEMISTRY FREE-RESPONSEQUESTIONS Answer EITHER Question 2 below OR Question 3 printed on page 8

2012 AP® CHEMISTRY FREERESPONSE QUESTIONS

H H H H H HI

___

I IlU—C—C-—N-—H C=C + :N—H

H H H H H

3. A sample ot CH1CH,NH2is placed in an insulated container, where it decomposes into ethene and ammonia

according to the reaction represented abose,

in J/(molK) at 298 K

CHCH2NH2(e 284.9

CH,CH2(g) 219.3

192.8

(a) Using the data in the table above, calculate the salue, in J/(mol, K), of the standard entropy change. ‘\S

for the reaction at 298 K.

(b) Using the data in the table below, calculate the value, in kJ/molrxn. of the standard enthalpy change, AlP,for the reaction at 298 K.

Bond C-CC=CJ C-H

I Average Bond Enthalpy I348 614 413 293 391I (kJ/mol) I I

(c) Based on your answer to part (b), predict whether the temperature of the contents of the insulated containerwill increase, decrease, or remain the same as the reaction proceeds. Justify your prediction.

An experiment is carried out to measure the rate of the reaction, which is first order. A 4.70 x 10 mol sample

of CHCH,NH2is placed in a previously esacuated 2,00 1. container at 773 K. After 20.0 minutes. the

concentration of the CHCH..NH, is found to be 3.60 x 10 4molfL.

d) Calculate the rate constant for the reaction at 773 K. Include units with your answer.

(e) Calculate the initial rate, in 41 mm of the reaction at 773 K.

(r If‘CH • \ H

is plotted versus time for this reaction, would the plot result in a straight line or would it

result in a curve Lxplain your reasoning

STOPIf you finish before time is called, you may check your work on this part only.

Do not turn to the other part of the test until you are told to do so.

2 12 r ( olk5eHo rd,S sit ‘l 11 ‘g. I ,rd r 55 k,eb ard rg

GO ON TO THE NEXT PAGE.-8-

Handout Page 176 of 182 Handout Page 176 of 182

Handout Page 176 of 182 Updated 4/9/2014 Handout Page 176 of 182

Page 177: V1) I - Blue Valley Schools Chemistry/AP... · 3DJH RI 3DJH RI 2001 AP® CHEMISTRY FREE-RESPONSEQUESTIONS Answer EITHER Question 2 below OR Question 3 printed on page 8

2012 AP® CHEMISTRY FREERESPONSE QUESTIONS

CHEMISTRY

Part B

Time—40 minutes%O CALCULATORS MAY BE USED FOR PART B,

Answer Question 4 below. The Section II score weighting for this question is 10 percent.

4. For each of the following three reactions, write a balanced equation for the reaction in part (i) and answer thequestion about the reaction in part (ii). In part (i), coefficients should be in terms of lowest whole numbers,Assume that solutions are aqueous unless otherwise indicated. Represent substances in solutions as ions if thesubstances are extensively ionized. Omit formulas for any ions or molecules that are unchanged by the reaction.You may use the empty space at the bottom of the next page for scratch work, but only equations that are writtenin the answer boxes provided will be scored.

EXAMPLE:

A strip of magnesium metal is added to a solution of silver(I) nitrate.

(i) Balanced equation:

[(ii) Which substance is oxidized in the reaction?

1(’L1

(a) A piece of solid strontium carbonate is dropped into a 0,1 M solution of hydrochloric acid.

(i) Balanced equation:

I I

(ii) Indicate one thing that would be observed as the reaction occurs.

© 2012 The College Board.Visit the College Board on the Web:. wwwcoi1egeboard.org.

GO ON TO THE NEXT PAGE.

Handout Page 177 of 182 Handout Page 177 of 182

Handout Page 177 of 182 Updated 4/9/2014 Handout Page 177 of 182

Page 178: V1) I - Blue Valley Schools Chemistry/AP... · 3DJH RI 3DJH RI 2001 AP® CHEMISTRY FREE-RESPONSEQUESTIONS Answer EITHER Question 2 below OR Question 3 printed on page 8

2012 AP® CHEMISTRY FREE-RESPONSE QUESTIONS

(b) Magnesium metal is trongl heated in oxYgen gas.

(i Balanced equation:

ii What is the oxidation number of macnesium before the reaction occurs, and what is the o\idatlonnumber of magnesium ater the reaction is complete?

ci A solution of nickelill) chloride is added to a solution of sodium h droxide. formine a precipitate.

i Balanced equation:

iii If equal volumes of 1 .0 M nickel( II> chloride and 1.0 .11 sodium hydroxide are used, what ion ispresent in the solution in the highest concemration after the precipitate forms?

YOU MAY USE TIlE SPACE BELOW FOR SCRATCH WORK, BUT ONLY EQUATIONSTHAT ARE WRITTEN IN TIlE ANSWER BOXES PROVIDED WILL BE SCORED.

1h ill a B.:rJ.

GO ON TO THE NEXT PAGE.-10-

Handout Page 178 of 182 Handout Page 178 of 182

Handout Page 178 of 182 Updated 4/9/2014 Handout Page 178 of 182

Page 179: V1) I - Blue Valley Schools Chemistry/AP... · 3DJH RI 3DJH RI 2001 AP® CHEMISTRY FREE-RESPONSEQUESTIONS Answer EITHER Question 2 below OR Question 3 printed on page 8

2012 AP® CHEMISTRY FREE-RESPONSE QUESTIONS

Answer Question 5 and Question 6. The Section II score weighting for these questions is 15 percent each.

Your responses to these questions will be scored on the basis of the accuracy and relevance of the information citedExplanations should be clear and well organized. Examples and equations may be included in your responses whereappropriate. Specific answers are preferable to broad. diffuse responses.

Process (U/mo1,,

fli T

L I2i 12(g) 62.44

5. At 298 K and I atm, the standard state of Br2 is a liquid, whereas the standard state of E is a solid. The enthalpy

changes for the formation of Br(e and 12(gi from these elemental forms at 298 K and I atm are given in the

table abos e.

ta) Explain why \H for the formation of 12(gi from Is) is larger than H° for the formation of Br,(g) from

Br2(1), In your explanation identify the type of particle interactions involved and a reason for the difference

in magnitude of those interactions.

(b) Predict which of the two processes shown in the table has the greater change in entropy. Justify yourprediction.

cc) J,() and Br2(l) can react to form the compound IBr(l). Predict which would have the greater molar

enthalpy of vaporization, IBr(1) or Br2(l). Justify your prediction.

An experiment is performed to compare the solubilities of 12(s) in different solvents, water and hexane (CI114).

A student adds 2 mL of H20 and 2 mL of C6H4 to a test tube. Because 1120 and C6H4 are immiscible,

two layers are observed in the test tube. The student drops a small, purple crystal of I2() into the test tube,

which is then corked and inverted several times. The C6H14 layer becomes light purple, while the H20 layer

remains virtually colorless.

(dl Explain why the hexane layer is light purple while the water layer is virtually colorless. Your explanation

should reference the relative strengths of interactions between molecules of 12 and the sol cuts H2() and

611i4 and the reasons for the differences.

.. lhe( oI1ez B irdzi hL ( d g B rd n thL \ct v, lkBeh r Lorg.

GO ON TO THE NEXT PAGE.—11—

Handout Page 179 of 182 Handout Page 179 of 182

Handout Page 179 of 182 Updated 4/9/2014 Handout Page 179 of 182

Page 180: V1) I - Blue Valley Schools Chemistry/AP... · 3DJH RI 3DJH RI 2001 AP® CHEMISTRY FREE-RESPONSEQUESTIONS Answer EITHER Question 2 below OR Question 3 printed on page 8

2012 AP® CHEMISTRY FREE-RESPONSE QUESTIONS

te) The student then adds a small crystal of Klis) to the test tube, The test tube is corked and inverted several

times. The I ion reacts with l to form the l ion, a linear species.

ut In the box below, draw the complete Lewis electron-dot diagram for the 1 ion.

(ii) In which layer, water or hexane, would the concentration of I be higher? Explain.

The LoIiee Board,\‘sit Ia College Board on th \ab; wo.)I1egenoard.org.

GO ON TO THE NEXT PAGE.-12-

Handout Page 180 of 182 Handout Page 180 of 182

Handout Page 180 of 182 Updated 4/9/2014 Handout Page 180 of 182

Page 181: V1) I - Blue Valley Schools Chemistry/AP... · 3DJH RI 3DJH RI 2001 AP® CHEMISTRY FREE-RESPONSEQUESTIONS Answer EITHER Question 2 below OR Question 3 printed on page 8

2012 AP® CHEMISTRY FREE-RESPONSE QUESTIONS

6. In a laboratory experiment. Ph and an unknown metal Q were immersed in solutions containina aqueous ions otunknown metals Q and X. The foHowing reactions summarize the observations.

Observation 1: Phs + XDaq Pbtaq) + X(s)Observation 2; Qs ± X’ —* no reactionObservation 3: Phs; + Qiaq Phaq + Qi s

a On the basis of the reactions indicated above, arrange the three metals. Ph. Q. and N. in order from leastreactive to most reactive on the lines provided below,

least reactive metal most reactive metal

The diagram below shows an electrochemical cell that is constructed with a Pb electrode immersed

in 100. mL of 1.0 Al PbtNO3)2(aq) and an electrode made of metal X immersed in 100. mL of

1.0 M X(N03)2(aq). A salt bridge containing saturated aqueous KNO3 connects the anode compartment to the

cathode compartment. The electrodes are connected to an external circuit containing a switch, which is open.When a voltmeter is connected to the circuit as shown, the reading on the voltmeter is 0.47 V. When the switchis closed, electrons flow through the switch from the Pb electrode toward the X electrode.

b; Write the equation for the half-reaction that occurs at the anode.

The due of the si indard potLntlal for the eell L i’ 1) 4’

i Determine the standard reduction potential for the half-reaction that occurs at the cathode.

(iii Determine the identity of metal N.

I d Describe what happens to the mass of each electrode as the cell operates.

V 2OI The (.olleae Board.Visit the College Board on :he Web: wavw.collegèboard.or5.

GO ON TO THE NEXT PAGE.

\bltmeter

1.0 M Pb(NO 1.0 M X(NO3)

-1 3.

Handout Page 181 of 182 Handout Page 181 of 182

Handout Page 181 of 182 Updated 4/9/2014 Handout Page 181 of 182

Page 182: V1) I - Blue Valley Schools Chemistry/AP... · 3DJH RI 3DJH RI 2001 AP® CHEMISTRY FREE-RESPONSEQUESTIONS Answer EITHER Question 2 below OR Question 3 printed on page 8

2012 AP° CHEMISTRY FREE-RESPONSE QUESTIONS

(e) During a laboratory session, students set up the electrochemical cell shown abose, For each of the follos ingthree scenarios, choose the correct same of the cell soltage and justify your choice

i A ‘indent hump the cell cetup, reculting in the ‘alt bridge losing contact ith the solution in thecathode compartment, Is equal to 047 or is V equal to 0? Justify your choice.

(ii) A student spills a small amount of 0,5 M aSO(aq) into the compartment with the Pb eledrode.

resulting in the formation 01 a precipitate. Is V less than 0.4’ or is V greater than 0.4 ‘ Justify \uurchoice.

iii i After the laboratory session is over, a student leaves the switch closed. The next day, the student opensthe switch and reads the voltmeter, Is V less than 0.47 or is V equal to 0.47 ? Justify sour choice.

STOP

END OF EXAM

-14-

Handout Page 182 of 182 Handout Page 182 of 182

Handout Page 182 of 182 Updated 4/9/2014 Handout Page 182 of 182